• Re: Einstein rejected Galileo & Eotvos

    From whodat@21:1/5 to Dono. on Mon Dec 25 18:17:31 2023
    On 12/25/2023 5:56 PM, Dono. wrote:
    On Monday, December 25, 2023 at 2:01:59 PM UTC-8, Laurence Clark Crossen wrote:
    On Monday, December 25, 2023 at 11:57:15 AM UTC-8, Dono. wrote:
    On Monday, December 25, 2023 at 11:51:31 AM UTC-8, Laurence Clark Crossen wrote:
    Because Galileo found that all objects, regardless of mass, are affected the same by gravity,
    and Eotvos found all objects are affected the same regardless of the material,
    Einstein's claim that light is affected twice Newtonian jettison's Galileo's and Eotvos' experimental scientific advances.
    Dumbfuck,

    Light has no mass.


    Then gravity wouldn't affect it,

    Dumbfuck,

    Gravity affects massless particles, like the photons.

    What doesn't gravity affect?

    --- SoupGate-Win32 v1.05
    * Origin: fsxNet Usenet Gateway (21:1/5)
  • From Tom Roberts@21:1/5 to Laurence Clark Crossen on Tue Dec 26 00:09:27 2023
    On 12/25/23 1:51 PM, Laurence Clark Crossen wrote:
    Because Galileo found that all objects, regardless of mass, are
    affected the same by gravity, and Eotvos found all objects are
    affected the same regardless of the material,

    Yes. With the advent of GR we now know that the first is only
    approximately valid for tiny objects, small fields, and velocities << c.
    So it does not apply to light.

    GR predicts that a massive particle moving past the sun with speed >
    0.999999*c would be deflected approximately the same as a light ray. A
    massive particle moving with speed < 0.001*c would be deflected
    approximately half as much as a light ray.

    Tom Roberts

    --- SoupGate-Win32 v1.05
    * Origin: fsxNet Usenet Gateway (21:1/5)
  • From Tom Roberts@21:1/5 to Maciej Wozniak on Tue Dec 26 09:48:31 2023
    On 12/26/23 1:37 AM, Maciej Wozniak wrote:
    On Tuesday 26 December 2023 at 07:09:40 UTC+1, Tom Roberts wrote:
    On 12/25/23 1:51 PM, Laurence Clark Crossen wrote:
    Because Galileo found that all objects, regardless of mass, are
    affected the same by gravity, and Eotvos found all objects are
    affected the same regardless of the material,
    Yes. With the advent of GR we now know that the first is only
    approximately valid for tiny objects, small fields, and velocities << c.
    So it does not apply to light.

    And with the advent of GPS we now know that GR shit
    has nothing in common with real clocks, real observers
    or real anything.

    Not true. We just know that Maciej Wozniak knows nothing about GPS, GR,
    or clocks.

    Tom Roberts

    --- SoupGate-Win32 v1.05
    * Origin: fsxNet Usenet Gateway (21:1/5)
  • From whodat@21:1/5 to Dono. on Tue Dec 26 13:15:02 2023
    On 12/25/2023 6:39 PM, Dono. wrote:
    On Monday, December 25, 2023 at 4:17:38 PM UTC-8, whodat wrote:
    On 12/25/2023 5:56 PM, Dono. wrote:
    On Monday, December 25, 2023 at 2:01:59 PM UTC-8, Laurence Clark Crossen wrote:
    On Monday, December 25, 2023 at 11:57:15 AM UTC-8, Dono. wrote:

    Dumbfuck,

    Light has no mass.


    Then gravity wouldn't affect it,

    Dumbfuck,

    Gravity affects massless particles, like the photons.
    What doesn't gravity affect?

    Affects even dumbfucks

    I have to wonder whether or not you ever listen to yourself.

    --- SoupGate-Win32 v1.05
    * Origin: fsxNet Usenet Gateway (21:1/5)
  • From Volney@21:1/5 to Maciej Wozniak on Tue Dec 26 15:28:01 2023
    On 12/26/2023 11:59 AM, Maciej Wozniak wrote:
    On Tuesday 26 December 2023 at 16:48:43 UTC+1, Tom Roberts wrote:
    On 12/26/23 1:37 AM, Maciej Wozniak wrote:

    And with the advent of GPS we now know that GR shit
    has nothing in common with real clocks, real observers
    or real anything.
    Not true. We just know that Maciej Wozniak knows nothing about GPS, GR,
    or clocks.

    Spitting and ravings won't help, poor fanatic trash. the
    "improper" clocks of GPS will keep measuring t'=t, just
    like all serious clocks always did.

    And once again, you got it back asswards. t'≠t is the whole reason why
    the master clock divisor on a GPS satellite is set to 9192631774.1 and
    not 9192631770.

    --- SoupGate-Win32 v1.05
    * Origin: fsxNet Usenet Gateway (21:1/5)
  • From Tom Roberts@21:1/5 to Laurence Clark Crossen on Tue Dec 26 15:41:58 2023
    On 12/26/23 3:30 PM, Laurence Clark Crossen wrote:
    On Monday, December 25, 2023 at 10:09:40 PM UTC-8, Tom Roberts
    wrote:
    On 12/25/23 1:51 PM, Laurence Clark Crossen wrote:
    Because Galileo found that all objects, regardless of mass, are
    affected the same by gravity, and Eotvos found all objects are
    affected the same regardless of the material,
    Yes. With the advent of GR we now know that the first is only
    approximately valid for tiny objects, small fields, and velocities
    << c. So it does not apply to light.

    GR predicts that a massive particle moving past the sun with speed
    0.999999*c would be deflected approximately the same as a light
    ray. A massive particle moving with speed < 0.001*c would be
    deflected approximately half as much as a light ray.

    Tom Roberts
    So you admit photons have mass and make them an exception to the rule
    purely on experimental results without any physics cause.

    You REALLY need to learn how to read. I said no such thing, you're just
    making it up and attributing it to me.

    Tom Roberts

    --- SoupGate-Win32 v1.05
    * Origin: fsxNet Usenet Gateway (21:1/5)
  • From Volney@21:1/5 to Maciej Wozniak on Wed Dec 27 09:08:03 2023
    On 12/27/2023 8:49 AM, Maciej Wozniak wrote:

    And that it will be nothing. According to your Shit
    light [in vacuum] takes always straight/geodesic
    paths.

    Straight or geodesic, janitor? Make up your mind. They aren't the same
    except where space is flat.

    --- SoupGate-Win32 v1.05
    * Origin: fsxNet Usenet Gateway (21:1/5)
  • From Tom Roberts@21:1/5 to Laurence Clark Crossen on Wed Dec 27 08:36:36 2023
    On 12/27/23 6:12 AM, Laurence Clark Crossen wrote:
    On Tuesday, December 26, 2023 at 1:42:10 PM UTC-8, Tom Roberts
    wrote:
    On 12/26/23 3:30 PM, Laurence Clark Crossen wrote:
    On Monday, December 25, 2023 at 10:09:40 PM UTC-8, Tom Roberts
    wrote:
    On 12/25/23 1:51 PM, Laurence Clark Crossen wrote:
    Because Galileo found that all objects, regardless of mass,
    are affected the same by gravity, and Eotvos found all
    objects are affected the same regardless of the material,
    Yes. With the advent of GR we now know that the first is only
    approximately valid for tiny objects, small fields, and
    velocities << c. So it does not apply to light.

    GR predicts that a massive particle moving past the sun with
    speed
    0.999999*c would be deflected approximately the same as a
    light
    ray. A massive particle moving with speed < 0.001*c would be
    deflected approximately half as much as a light ray.

    Tom Roberts
    So you admit photons have mass and make them an exception to the
    rule purely on experimental results without any physics cause.
    You REALLY need to learn how to read. I said no such thing, you're
    just making it up and attributing it to me.

    Tom Roberts
    According to the mass-energy relationship, anything with energy has
    mass,

    This is not true. Why do you just make stuff up and pretend it is true?

    so photons must have mass.

    The current measurements of photon mass are consistent with zero, with
    an upper limit of 10^-18 eV.

    That is so incredibly small that any deviation between the actual vacuum
    speed of light and c (the invariant velocity of SR and GR) is completely unmeasurable, even over cosmic distances.

    Tom Roberts

    --- SoupGate-Win32 v1.05
    * Origin: fsxNet Usenet Gateway (21:1/5)
  • From Volney@21:1/5 to Laurence Clark Crossen on Wed Dec 27 17:39:00 2023
    On 12/27/2023 2:41 PM, Laurence Clark Crossen wrote:
    On Wednesday, December 27, 2023 at 6:36:48 AM UTC-8, Tom Roberts wrote:
    On 12/27/23 6:12 AM, Laurence Clark Crossen wrote:
    On Tuesday, December 26, 2023 at 1:42:10 PM UTC-8, Tom Roberts
    wrote:
    On 12/26/23 3:30 PM, Laurence Clark Crossen wrote:
    On Monday, December 25, 2023 at 10:09:40 PM UTC-8, Tom Roberts
    wrote:
    On 12/25/23 1:51 PM, Laurence Clark Crossen wrote:
    Because Galileo found that all objects, regardless of mass,
    are affected the same by gravity, and Eotvos found all
    objects are affected the same regardless of the material,
    Yes. With the advent of GR we now know that the first is only
    approximately valid for tiny objects, small fields, and
    velocities << c. So it does not apply to light.

    GR predicts that a massive particle moving past the sun with
    speed
    0.999999*c would be deflected approximately the same as a
    light
    ray. A massive particle moving with speed < 0.001*c would be
    deflected approximately half as much as a light ray.

    Tom Roberts
    So you admit photons have mass and make them an exception to the
    rule purely on experimental results without any physics cause.
    You REALLY need to learn how to read. I said no such thing, you're
    just making it up and attributing it to me.

    Tom Roberts
    According to the mass-energy relationship, anything with energy has
    mass,
    This is not true. Why do you just make stuff up and pretend it is true?
    so photons must have mass.
    The current measurements of photon mass are consistent with zero, with
    an upper limit of 10^-18 eV.

    That is so incredibly small that any deviation between the actual vacuum
    speed of light and c (the invariant velocity of SR and GR) is completely
    unmeasurable, even over cosmic distances.

    Tom Roberts

    How can anything with energy not have mass?

    Because for light, E=pc, where p is the momentum and c is the speed of
    light.

    If photons have no mass, there is no way they would be affected by gravity.

    Gravity actually acts on energy, not just mass.

    --- SoupGate-Win32 v1.05
    * Origin: fsxNet Usenet Gateway (21:1/5)
  • From Volney@21:1/5 to Laurence Clark Crossen on Wed Dec 27 23:38:37 2023
    On 12/27/2023 6:23 PM, Laurence Clark Crossen wrote:
    On Wednesday, December 27, 2023 at 2:39:04 PM UTC-8, Volney wrote:
    On 12/27/2023 2:41 PM, Laurence Clark Crossen wrote:

    How can anything with energy not have mass?

    Because for light, E=pc, where p is the momentum and c is the speed of
    light.

    If photons have no mass, there is no way they would be affected by gravity.

    Gravity actually acts on energy, not just mass.

    Momentum is the product of mass and velocity.

    For massive particles, that's true. Light has no mass.

    Kindly tell me what examples you have of gravity acting on energy.

    Light. Specifically the eclipse starlight deviation, Einstein crosses,
    black holes.

    --- SoupGate-Win32 v1.05
    * Origin: fsxNet Usenet Gateway (21:1/5)
  • From Volney@21:1/5 to Laurence Clark Crossen on Thu Dec 28 11:57:19 2023
    On 12/27/2023 11:57 PM, Laurence Clark Crossen wrote:
    On Wednesday, December 27, 2023 at 8:38:42 PM UTC-8, Volney wrote:
    On 12/27/2023 6:23 PM, Laurence Clark Crossen wrote:
    On Wednesday, December 27, 2023 at 2:39:04 PM UTC-8, Volney wrote:
    On 12/27/2023 2:41 PM, Laurence Clark Crossen wrote:

    How can anything with energy not have mass?

    Because for light, E=pc, where p is the momentum and c is the speed of >>>> light.

    If photons have no mass, there is no way they would be affected by gravity.

    Gravity actually acts on energy, not just mass.

    Momentum is the product of mass and velocity.

    For massive particles, that's true. Light has no mass.

    Kindly tell me what examples you have of gravity acting on energy.

    Light. Specifically the eclipse starlight deviation, Einstein crosses,
    black holes.

    Tom just said light has mass.

    No, he did not. In fact he explicitly called you out for making stuff up
    (light has mass) and attributing it to him.

    How can light have momentum without mass when momentum is a product of mass and velocity?

    Again, that's the momentum of massive particles. Physicists deal with 4-momentum and how it translates to classical momentum these days. Do
    try to keep up.

    --- SoupGate-Win32 v1.05
    * Origin: fsxNet Usenet Gateway (21:1/5)
  • From Volney@21:1/5 to Laurence Clark Crossen on Thu Dec 28 23:13:16 2023
    On 12/28/2023 4:37 PM, Laurence Clark Crossen wrote:
    On Thursday, December 28, 2023 at 8:57:23 AM UTC-8, Volney wrote:
    On 12/27/2023 11:57 PM, Laurence Clark Crossen wrote:
    On Wednesday, December 27, 2023 at 8:38:42 PM UTC-8, Volney wrote:
    On 12/27/2023 6:23 PM, Laurence Clark Crossen wrote:
    On Wednesday, December 27, 2023 at 2:39:04 PM UTC-8, Volney wrote: >>>>>> On 12/27/2023 2:41 PM, Laurence Clark Crossen wrote:

    How can anything with energy not have mass?

    Because for light, E=pc, where p is the momentum and c is the speed of >>>>>> light.

    If photons have no mass, there is no way they would be affected by gravity.

    Gravity actually acts on energy, not just mass.

    Momentum is the product of mass and velocity.

    For massive particles, that's true. Light has no mass.

    Kindly tell me what examples you have of gravity acting on energy.

    Light. Specifically the eclipse starlight deviation, Einstein crosses, >>>> black holes.

    Tom just said light has mass.

    No, he did not. In fact he explicitly called you out for making stuff up
    (light has mass) and attributing it to him.

    How can light have momentum without mass when momentum is a product of mass and velocity?

    Again, that's the momentum of massive particles. Physicists deal with
    4-momentum and how it translates to classical momentum these days. Do
    try to keep up.

    So light is a particle without mass that dislodges electrons?

    Yes, because photons have energy and momentum. Again, do try to keep up.

    --- SoupGate-Win32 v1.05
    * Origin: fsxNet Usenet Gateway (21:1/5)
  • From Tom Roberts@21:1/5 to Laurence Clark Crossen on Fri Dec 29 12:19:28 2023
    On 12/27/23 1:41 PM, Laurence Clark Crossen wrote:
    How can anything with energy not have mass?

    Simple -- energy and mass are different quantities. Moreover, the
    mass of a given object is invariant, while its energy is frame
    dependent.

    (BTW this is no different from Newtonian mechanics, in
    which the kinetic energy of an object is CLEARLY frame
    dependent, while mass is CLEARLY intrinsic to the object.)

    You REALLY need to learn basic physics.

    In SR and GR, a given object has a given 4-momentum. Its mass is the
    norm of its 4-mometum, its energy is the time component of its
    4-momentum, and its (3-)momentum is the 3 spatial components of its
    4-momentum; here all components are projected onto the desired (locally) inertial frame.

    (Here "locally" is required in GR but not in SR; "3-" is
    required in SR and GR but not Newtonian mechanics.)

    So a photon can have zero mass if its energy is equal to its
    (3-)momentum (c=1), and still have nonzero energy and nonzero (3-)momentum.

    If photons have no mass, there is no way they would be affected by
    gravity.

    This is simply not true. But one does need to use GR to model gravity.

    Everything about relativity is pure fiction. There is no fourth
    spatial dimension except in the minds of the stupid and gullible,
    and time is not a spatial dimension.

    This is just your ignorance and stupidity talking -- time is indeed not
    a spatial dimension, it is a TEMPORAL dimension of spaceTIME.

    Other nonsense written by you in this thread:

    Tom just said light has mass.

    I NEVER said that. You need to stop making stuff up and attributing it
    to me.

    if [a photon doesn't have mass, then the consensus that it is
    affected by gravity is wrong because gravity only affects mass.

    This is just plain false. The consensus among physicists is that gravity affects all types of energy. For the simple reason that it is observed
    to do so, and GR models this appropriately and accurately.

    4-momentum is the pseudoscience of relativity.

    No, it's just that you are so comprehensively ignorant and outrageously
    stupid that you do not understand much of anything about modern physics
    (or about classical physics, either).

    Tom Roberts

    --- SoupGate-Win32 v1.05
    * Origin: fsxNet Usenet Gateway (21:1/5)
  • From Volney@21:1/5 to Laurence Clark Crossen on Fri Dec 29 17:40:54 2023
    On 12/28/2023 11:39 PM, Laurence Clark Crossen wrote:
    On Thursday, December 28, 2023 at 8:13:20 PM UTC-8, Volney wrote:
    On 12/28/2023 4:37 PM, Laurence Clark Crossen wrote:

    So light is a particle without mass that dislodges electrons?

    Yes, because photons have energy and momentum. Again, do try to keep up.

    Is heat slowed by gravity when it leaves the Sun? I do not think so.

    All photons from the sun move at c.

    There is a redshift of infrared photons which decreases their frequency
    very slightly.

    --- SoupGate-Win32 v1.05
    * Origin: fsxNet Usenet Gateway (21:1/5)
  • From Volney@21:1/5 to Laurence Clark Crossen on Fri Dec 29 18:53:32 2023
    On 12/28/2023 11:29 PM, Laurence Clark Crossen wrote:
    On Thursday, December 28, 2023 at 8:13:20 PM UTC-8, Volney wrote:
    On 12/28/2023 4:37 PM, Laurence Clark Crossen wrote:

    So light is a particle without mass that dislodges electrons?

    Yes, because photons have energy and momentum. Again, do try to keep up.

    Your confusing disagreeing with misunderstanding. Do try not to be a mindless devotee.

    To you it must be the same thing. Anything you can't understand you
    disagree with it and claim it to be false, rather than learn about it.

    --- SoupGate-Win32 v1.05
    * Origin: fsxNet Usenet Gateway (21:1/5)
  • From Paul B. Andersen@21:1/5 to Paul B. Andersen on Sat Dec 30 15:22:38 2023
    Den 30.12.2023 14:22, skrev Lou:

    Regarding GPS and Relativity... I noticed a paper, cited below from NIST, which says that the on board satelitte oscillator broadcasts at 10.23 MHz. (Not to 10229999.99543Mhz as normally assumed.) This is also confirmed
    by a former NASA GPS engineer Larry Ortega who concurs that ,contrary
    to common assumption, the on board sat clocks are NOT preset to 10229999.99543Hz
    But rather set to 10.23 Mhz and instead corrected every 24 hours
    from the ground.


    You have been told before:

    On Wednesday, 15 November 2023 at 18:20:28 UTC, Paul B. Andersen wrote:

    The correction is specified in the
    "INTERFACE SPECIFICATION DOCUMENT" for GPS:

    https://www.gps.gov/technical/icwg/IS-GPS-200N.pdf

    From 3.3.1.1 Frequency Plan:
    "The carrier frequencies for the L1 and L2 signals shall be
    coherently derived from a common frequency source within the SV.
    The nominal frequency of this source -- as it appears to an observer
    on the ground -- is 10.23 MHz. The SV carrier frequency and clock
    rates -- as they would appear to an observer located in the SV --
    are offset to compensate for relativistic effects. The clock rates
    are offset by Δf/f = -4.4647E-10, equivalent to a change in the
    P-code chipping rate of 10.23 MHz offset by a Δf = -4.5674E-3 Hz.
    This is equal to 10.2299999954326 MHz."

    SV = space vehicle, satellite.


    --
    Paul

    https://paulba.no/

    --- SoupGate-Win32 v1.05
    * Origin: fsxNet Usenet Gateway (21:1/5)
  • From Volney@21:1/5 to Lou on Sat Dec 30 10:51:09 2023
    On 12/29/2023 6:11 AM, Lou wrote:
    On Tuesday 26 December 2023 at 20:28:04 UTC, Volney wrote:
    On 12/26/2023 11:59 AM, Maciej Wozniak wrote:
    On Tuesday 26 December 2023 at 16:48:43 UTC+1, Tom Roberts wrote:
    On 12/26/23 1:37 AM, Maciej Wozniak wrote:

    And with the advent of GPS we now know that GR shit
    has nothing in common with real clocks, real observers
    or real anything.
    Not true. We just know that Maciej Wozniak knows nothing about GPS, GR, >>>> or clocks.

    Spitting and ravings won't help, poor fanatic trash. the
    "improper" clocks of GPS will keep measuring t'=t, just
    like all serious clocks always did.

    And once again, you got it back asswards. t'≠t is the whole reason why
    the master clock divisor on a GPS satellite is set to 9192631774.1 and
    not 9192631770.

    4.1? I get 4.2 using Pauls method from his website.
    1.000000000457 × 9192661770= 9192661774.2


    10.23 MHz (frequency received on ground)
    10.2299999954326 MHz (transmitted frequency, from gps.gov website)

    10.23/10.2299999954326 = 1.0000000004464711634446932934426

    1.0000000004464711634446932934426 * 9192631770 = 9,192,631,774.1042450014705502072059

    That's the divisor of a Cs clock to generate an exact 1 pps signal on
    the ground.

    --- SoupGate-Win32 v1.05
    * Origin: fsxNet Usenet Gateway (21:1/5)
  • From Volney@21:1/5 to Lou on Sat Dec 30 18:13:06 2023
    On 12/30/2023 3:01 PM, Lou wrote:
    On Saturday 30 December 2023 at 15:51:14 UTC, Volney wrote:
    On 12/29/2023 6:11 AM, Lou wrote:
    On Tuesday 26 December 2023 at 20:28:04 UTC, Volney wrote:
    On 12/26/2023 11:59 AM, Maciej Wozniak wrote:
    On Tuesday 26 December 2023 at 16:48:43 UTC+1, Tom Roberts wrote:
    On 12/26/23 1:37 AM, Maciej Wozniak wrote:

    And with the advent of GPS we now know that GR shit
    has nothing in common with real clocks, real observers
    or real anything.
    Not true. We just know that Maciej Wozniak knows nothing about GPS, GR, >>>>>> or clocks.

    Spitting and ravings won't help, poor fanatic trash. the
    "improper" clocks of GPS will keep measuring t'=t, just
    like all serious clocks always did.

    And once again, you got it back asswards. t'≠t is the whole reason why >>>> the master clock divisor on a GPS satellite is set to 9192631774.1 and >>>> not 9192631770.

    4.1? I get 4.2 using Pauls method from his website.
    1.000000000457 × 9192661770= 9192661774.2


    10.23 MHz (frequency received on ground)
    10.2299999954326 MHz (transmitted frequency, from gps.gov website)

    10.23/10.2299999954326 = 1.0000000004464711634446932934426

    1.0000000004464711634446932934426 * 9192631770 =
    9,192,631,774.1042450014705502072059

    That's the divisor of a Cs clock to generate an exact 1 pps signal on
    the ground.

    Yes, but let’s try the same calculation using the clock gain of 457 for both SR and GR from 10.22999Mhz.
    1.000000000457x 9192661770= 9192661774.2

    Where did you get those numbers from? The second is defined to be
    9192631770 Cs cycles, not 9192661770.

    --- SoupGate-Win32 v1.05
    * Origin: fsxNet Usenet Gateway (21:1/5)
  • From Volney@21:1/5 to Lou on Sat Dec 30 22:17:20 2023
    On 12/30/2023 8:07 PM, Lou wrote:
    On Saturday 30 December 2023 at 23:13:15 UTC, Volney wrote:
    On 12/30/2023 3:01 PM, Lou wrote:
    On Saturday 30 December 2023 at 15:51:14 UTC, Volney wrote:
    On 12/29/2023 6:11 AM, Lou wrote:
    On Tuesday 26 December 2023 at 20:28:04 UTC, Volney wrote:
    On 12/26/2023 11:59 AM, Maciej Wozniak wrote:
    On Tuesday 26 December 2023 at 16:48:43 UTC+1, Tom Roberts wrote: >>>>>>>> On 12/26/23 1:37 AM, Maciej Wozniak wrote:

    And with the advent of GPS we now know that GR shit
    has nothing in common with real clocks, real observers
    or real anything.
    Not true. We just know that Maciej Wozniak knows nothing about GPS, GR,
    or clocks.

    Spitting and ravings won't help, poor fanatic trash. the
    "improper" clocks of GPS will keep measuring t'=t, just
    like all serious clocks always did.

    And once again, you got it back asswards. t'≠t is the whole reason why >>>>>> the master clock divisor on a GPS satellite is set to 9192631774.1 and >>>>>> not 9192631770.

    4.1? I get 4.2 using Pauls method from his website.
    1.000000000457 × 9192661770= 9192661774.2


    10.23 MHz (frequency received on ground)
    10.2299999954326 MHz (transmitted frequency, from gps.gov website)

    10.23/10.2299999954326 = 1.0000000004464711634446932934426

    1.0000000004464711634446932934426 * 9192631770 =
    9,192,631,774.1042450014705502072059

    That's the divisor of a Cs clock to generate an exact 1 pps signal on
    the ground.

    Yes, but let’s try the same calculation using the clock gain of 457 for >>> both SR and GR from 10.22999Mhz.
    1.000000000457x 9192661770= 9192661774.2
    Where did you get those numbers from? The second is defined to be
    9192631770 Cs cycles, not 9192661770.

    Yes, sorry my typos. It is 9192631770.

    And the calculations based on your incorrect value?

    But my point is that you say you can calculate total gains of 457 for 10.22999Mhz
    from the clock gains of another frequency ( that frequency being 8.9875518e+16hz

    That number 8.9875518e+16 is not a frequency. It is c^2.

    which has a gain of +446 which you then use to calculate the gain
    of +457 for 10.22999Mhz)

    And you have that value wrong as well.

    [snip rest of GIGO]

    You are very, very confused.

    --- SoupGate-Win32 v1.05
    * Origin: fsxNet Usenet Gateway (21:1/5)
  • From Paul B. Andersen@21:1/5 to All on Sun Dec 31 11:16:50 2023
    Den 30.12.2023 20:40, skrev Lou:
    On Saturday 30 December 2023 at 14:19:33 UTC, Paul B. Andersen wrote:
    Den 30.12.2023 14:22, skrev Lou:

    Regarding GPS and Relativity... I noticed a paper, cited below from NIST, >>> which says that the on board satelitte oscillator broadcasts at 10.23 MHz. >>> (Not to 10229999.99543Mhz as normally assumed.) This is also confirmed
    by a former NASA GPS engineer Larry Ortega who concurs that ,contrary
    to common assumption, the on board sat clocks are NOT preset to
    10229999.99543Hz
    But rather set to 10.23 Mhz and instead corrected every 24 hours
    from the ground.


    You have been told before:

    On Wednesday, 15 November 2023 at 18:20:28 UTC, Paul B. Andersen wrote:

    The correction is specified in the
    "INTERFACE SPECIFICATION DOCUMENT" for GPS:

    https://www.gps.gov/technical/icwg/IS-GPS-200N.pdf

    From 3.3.1.1 Frequency Plan:
    "The carrier frequencies for the L1 and L2 signals shall be
    coherently derived from a common frequency source within the SV.
    The nominal frequency of this source -- as it appears to an observer
    on the ground -- is 10.23 MHz. The SV carrier frequency and clock
    rates -- as they would appear to an observer located in the SV --
    are offset to compensate for relativistic effects. The clock rates
    are offset by Δf/f = -4.4647E-10, equivalent to a change in the
    P-code chipping rate of 10.23 MHz offset by a Δf = -4.5674E-3 Hz.
    This is equal to 10.2299999954326 MHz."

    SV = space vehicle, satellite.

    Yes. But it’s the only time in the whole paper that 1022999 is mentioned. Every other reference in the paper says the sat clock broadcast frequency is 10.23Mhz. And that it’s NOT preset before launch.

    The "INTERFACE SPECIFICATION DOCUMENT" _SPECIFIES_ that
    the frequency standard is set to 10.2299999954326 MHz.

    Do you know what a SPECIFICATION is?

    The PRN-P code chipping rate is 10.23 Mbps measured
    relative to GPS-coordinated time.
    (or "- as it appears to an observer on the ground -")

    That is 10.2299999954326 Mbps measured by a SI-clock in the satellite.
    (or "-as they would appear to an observer located in the SV -")

    --
    Paul

    https://paulba.no/

    --- SoupGate-Win32 v1.05
    * Origin: fsxNet Usenet Gateway (21:1/5)
  • From Volney@21:1/5 to Maciej Wozniak on Sun Dec 31 12:08:44 2023
    On 12/31/2023 2:33 AM, Maciej Wozniak wrote:
    On Sunday 31 December 2023 at 00:13:15 UTC+1, Volney wrote:

    Where did you get those numbers from? The second is defined to be
    9192631770 Cs cycles

    No, stupid Mike, sorry, all the serious timekeeping system and clocks are ignoring your ideological madness. Te second is still 1/(24*60*60) of
    a day, just like it was when your idiot guru lived and mumbled.

    Sorry, janitor, but that hasn't been true for nearly 57 years now.

    SI Second: "The second, symbol s, is the SI unit of time. It is defined
    by taking the fixed numerical value of the caesium frequency, ∆νCs, the unperturbed ground-state hyperfine transition frequency of the caesium
    133 atom, to be 9 192 631 770 when expressed in the unit Hz, which is
    equal to s−1."

    Since you've been told that repeatedly, what's your excuse for *still*
    being wrong?

    --- SoupGate-Win32 v1.05
    * Origin: fsxNet Usenet Gateway (21:1/5)
  • From Volney@21:1/5 to Lou on Sun Dec 31 12:27:40 2023
    On 12/30/2023 2:40 PM, Lou wrote:
    On Saturday 30 December 2023 at 14:19:33 UTC, Paul B. Andersen wrote:
    Den 30.12.2023 14:22, skrev Lou:

    Regarding GPS and Relativity... I noticed a paper, cited below from NIST, >>> which says that the on board satelitte oscillator broadcasts at 10.23 MHz. >>> (Not to 10229999.99543Mhz as normally assumed.) This is also confirmed
    by a former NASA GPS engineer Larry Ortega who concurs that ,contrary
    to common assumption, the on board sat clocks are NOT preset to
    10229999.99543Hz
    But rather set to 10.23 Mhz and instead corrected every 24 hours
    from the ground.

    You have been told before:

    On Wednesday, 15 November 2023 at 18:20:28 UTC, Paul B. Andersen wrote:

    The correction is specified in the
    "INTERFACE SPECIFICATION DOCUMENT" for GPS:

    https://www.gps.gov/technical/icwg/IS-GPS-200N.pdf

    From 3.3.1.1 Frequency Plan:
    "The carrier frequencies for the L1 and L2 signals shall be
    coherently derived from a common frequency source within the SV.
    The nominal frequency of this source -- as it appears to an observer
    on the ground -- is 10.23 MHz. The SV carrier frequency and clock
    rates -- as they would appear to an observer located in the SV --
    are offset to compensate for relativistic effects. The clock rates
    are offset by Δf/f = -4.4647E-10, equivalent to a change in the
    P-code chipping rate of 10.23 MHz offset by a Δf = -4.5674E-3 Hz.
    This is equal to 10.2299999954326 MHz."

    SV = space vehicle, satellite.

    Yes. But it’s the only time in the whole paper that 1022999 is mentioned. Every other reference in the paper says the sat clock broadcast frequency is 10.23Mhz.

    That's because that's where the frequency to be used is DEFINED. It's
    the Interface Specification Document. All other references (to 10.23
    MHz) are either on the the ground/geoid frequencies or described as a
    nominal frequency.


    And that it’s NOT preset before launch.

    As this is where the frequency is specified, it is set by design, and
    most certainly is set before launch. By Design.

    --- SoupGate-Win32 v1.05
    * Origin: fsxNet Usenet Gateway (21:1/5)
  • From Volney@21:1/5 to Laurence Clark Crossen on Sun Dec 31 12:35:48 2023
    On 12/31/2023 10:08 AM, Laurence Clark Crossen wrote:

    It would have to change from a lower to a higher frequency. Because the speed of light moving from the vacuum of space into the atmosphere changes from 3 x 10e8 m/sec to 2.997x 10e8m/sec it forms compression waves increasing the frequency from 10.
    2299999954326 Mbps measured by a SI-clock in the satellite to 10.23 Mbps.

    Word salad gibberish.

    The speed of light 299,792,458 m/s is the VACUUM speed of light. In a non-vacuum like air, the frequency is unchanged. Only the speed and
    wavelength change.

    No relatovity required or involved whatsoever.

    Sorry, but the 10.2299999954326 MHz frequency is explicitly derived from
    10.23 MHz using GR calculations for the satellite orbit. You're in denial.

    --- SoupGate-Win32 v1.05
    * Origin: fsxNet Usenet Gateway (21:1/5)
  • From Volney@21:1/5 to Lou on Sun Dec 31 12:19:10 2023
    On 12/31/2023 5:27 AM, Lou wrote:
    On Sunday 31 December 2023 at 03:17:25 UTC, Volney wrote:
    On 12/30/2023 8:07 PM, Lou wrote:
    On Saturday 30 December 2023 at 23:13:15 UTC, Volney wrote:
    On 12/30/2023 3:01 PM, Lou wrote:
    On Saturday 30 December 2023 at 15:51:14 UTC, Volney wrote:
    On 12/29/2023 6:11 AM, Lou wrote:
    On Tuesday 26 December 2023 at 20:28:04 UTC, Volney wrote:
    On 12/26/2023 11:59 AM, Maciej Wozniak wrote:
    On Tuesday 26 December 2023 at 16:48:43 UTC+1, Tom Roberts wrote: >>>>>>>>>> On 12/26/23 1:37 AM, Maciej Wozniak wrote:

    And with the advent of GPS we now know that GR shit
    has nothing in common with real clocks, real observers
    or real anything.
    Not true. We just know that Maciej Wozniak knows nothing about GPS, GR,
    or clocks.

    Spitting and ravings won't help, poor fanatic trash. the
    "improper" clocks of GPS will keep measuring t'=t, just
    like all serious clocks always did.

    And once again, you got it back asswards. t'≠t is the whole reason why
    the master clock divisor on a GPS satellite is set to 9192631774.1 and >>>>>>>> not 9192631770.

    4.1? I get 4.2 using Pauls method from his website.
    1.000000000457 × 9192661770= 9192661774.2


    10.23 MHz (frequency received on ground)
    10.2299999954326 MHz (transmitted frequency, from gps.gov website) >>>>>>
    10.23/10.2299999954326 = 1.0000000004464711634446932934426

    1.0000000004464711634446932934426 * 9192631770 =
    9,192,631,774.1042450014705502072059

    That's the divisor of a Cs clock to generate an exact 1 pps signal on >>>>>> the ground.

    Yes, but let’s try the same calculation using the clock gain of 457 for >>>>> both SR and GR from 10.22999Mhz.
    1.000000000457x 9192661770= 9192661774.2
    Where did you get those numbers from? The second is defined to be
    9192631770 Cs cycles, not 9192661770.

    Yes, sorry my typos. It is 9192631770.
    And the calculations based on your incorrect value?
    But my point is that you say you can calculate total gains of 457 for 10.22999Mhz
    from the clock gains of another frequency ( that frequency being 8.9875518e+16hz
    That number 8.9875518e+16 is not a frequency. It is c^2.
    which has a gain of +446 which you then use to calculate the gain
    of +457 for 10.22999Mhz)
    And you have that value wrong as well.

    47379129.4927 ÷ 10229999.9954= 4.63139095932
    If this above calculation and its resulting value is wrong...

    What is this 47379129.4927 number? And 10229999.9954 is *still* wrong.
    You need to use the exact values with all the sig figs to get things EXACT.

    Then presumably you think using the same formula as above but
    dividing into a different frequency (c^2),

    c^2 is NOT a frequency!

    also gives the wrong value?
    47379430.8842 ÷ 8.9875518e+16 (GPS) = 5.2716726e-10

    GIGO. 8.9875518e+16 is NOT a frequency!

    [snip rest of GIGO]

    You are very, very confused.

    Not so confused as to realise that all GR does is divide potential (GM/r) into frequency to get a clock gain of + 5.2716726e-10

    GIGO.

    --- SoupGate-Win32 v1.05
    * Origin: fsxNet Usenet Gateway (21:1/5)
  • From Volney@21:1/5 to Laurence Clark Crossen on Mon Jan 1 12:04:23 2024
    On 12/31/2023 12:50 PM, Laurence Clark Crossen wrote:
    On Sunday, December 31, 2023 at 9:35:52 AM UTC-8, Volney wrote:
    On 12/31/2023 10:08 AM, Laurence Clark Crossen wrote:

    It would have to change from a lower to a higher frequency. Because the speed of light moving from the vacuum of space into the atmosphere changes from 3 x 10e8 m/sec to 2.997x 10e8m/sec it forms compression waves increasing the frequency from 10.
    2299999954326 Mbps measured by a SI-clock in the satellite to 10.23 Mbps.

    Word salad gibberish.

    The speed of light 299,792,458 m/s is the VACUUM speed of light. In a
    non-vacuum like air, the frequency is unchanged. Only the speed and
    wavelength change.

    No relatovity required or involved whatsoever.

    Sorry, but the 10.2299999954326 MHz frequency is explicitly derived from
    10.23 MHz using GR calculations for the satellite orbit. You're in denial.

    The frequency and the wavelength are two sides of the same coin. The only reason the frequency changes in this case is due to the forming of the compression waves.

    Is today "Make up a fact day" or something? "Compression waves" is a
    bite of word salad.

    --- SoupGate-Win32 v1.05
    * Origin: fsxNet Usenet Gateway (21:1/5)
  • From Volney@21:1/5 to Laurence Clark Crossen on Mon Jan 1 12:02:25 2024
    On 12/31/2023 4:42 PM, Laurence Clark Crossen wrote:
    On Sunday, December 31, 2023 at 9:35:52 AM UTC-8, Volney wrote:
    On 12/31/2023 10:08 AM, Laurence Clark Crossen wrote:

    It would have to change from a lower to a higher frequency. Because the speed of light moving from the vacuum of space into the atmosphere changes from 3 x 10e8 m/sec to 2.997x 10e8m/sec it forms compression waves increasing the frequency from 10.
    2299999954326 Mbps measured by a SI-clock in the satellite to 10.23 Mbps.

    Word salad gibberish.

    The speed of light 299,792,458 m/s is the VACUUM speed of light. In a
    non-vacuum like air, the frequency is unchanged. Only the speed and
    wavelength change.

    No relatovity required or involved whatsoever.

    Sorry, but the 10.2299999954326 MHz frequency is explicitly derived from
    10.23 MHz using GR calculations for the satellite orbit. You're in denial.

    Anderson said it would be 10.2299999954326 at the satellite and 10.23 at the receiver; did he not?

    And it's derived from GR using the satellite's altitude and orbital
    velocity.

    --- SoupGate-Win32 v1.05
    * Origin: fsxNet Usenet Gateway (21:1/5)
  • From Volney@21:1/5 to Lou on Mon Jan 1 11:57:20 2024
    On 12/31/2023 2:48 PM, Lou wrote:
    On Sunday 31 December 2023 at 17:19:13 UTC, Volney wrote:
    On 12/31/2023 5:27 AM, Lou wrote:
    On Sunday 31 December 2023 at 03:17:25 UTC, Volney wrote:
    On 12/30/2023 8:07 PM, Lou wrote:
    On Saturday 30 December 2023 at 23:13:15 UTC, Volney wrote:
    On 12/30/2023 3:01 PM, Lou wrote:
    On Saturday 30 December 2023 at 15:51:14 UTC, Volney wrote:
    On 12/29/2023 6:11 AM, Lou wrote:
    On Tuesday 26 December 2023 at 20:28:04 UTC, Volney wrote:
    On 12/26/2023 11:59 AM, Maciej Wozniak wrote:
    On Tuesday 26 December 2023 at 16:48:43 UTC+1, Tom Roberts wrote: >>>>>>>>>>>> On 12/26/23 1:37 AM, Maciej Wozniak wrote:

    And with the advent of GPS we now know that GR shit
    has nothing in common with real clocks, real observers >>>>>>>>>>>>> or real anything.
    Not true. We just know that Maciej Wozniak knows nothing about GPS, GR,
    or clocks.

    Spitting and ravings won't help, poor fanatic trash. the >>>>>>>>>>> "improper" clocks of GPS will keep measuring t'=t, just
    like all serious clocks always did.

    And once again, you got it back asswards. t'≠t is the whole reason why
    the master clock divisor on a GPS satellite is set to 9192631774.1 and
    not 9192631770.

    4.1? I get 4.2 using Pauls method from his website.
    1.000000000457 × 9192661770= 9192661774.2


    10.23 MHz (frequency received on ground)
    10.2299999954326 MHz (transmitted frequency, from gps.gov website) >>>>>>>>
    10.23/10.2299999954326 = 1.0000000004464711634446932934426

    1.0000000004464711634446932934426 * 9192631770 =
    9,192,631,774.1042450014705502072059

    That's the divisor of a Cs clock to generate an exact 1 pps signal on >>>>>>>> the ground.

    Yes, but let’s try the same calculation using the clock gain of 457 for
    both SR and GR from 10.22999Mhz.
    1.000000000457x 9192661770= 9192661774.2
    Where did you get those numbers from? The second is defined to be
    9192631770 Cs cycles, not 9192661770.

    Yes, sorry my typos. It is 9192631770.
    And the calculations based on your incorrect value?
    But my point is that you say you can calculate total gains of 457 for 10.22999Mhz
    from the clock gains of another frequency ( that frequency being 8.9875518e+16hz
    That number 8.9875518e+16 is not a frequency. It is c^2.
    which has a gain of +446 which you then use to calculate the gain
    of +457 for 10.22999Mhz)
    And you have that value wrong as well.

    47379129.4927 ÷ 10229999.9954= 4.63139095932
    If this above calculation and its resulting value is wrong...

    What is this 47379129.4927 number?

    Please note I did start off trying to be polite. But seeing as I’m talking to
    someone from the gutter ( who doesn’t even know what GM/r is...)
    Ever heard of GM/r ? Obviously not. Because that’s what 47379129.4927 is.

    You never said so. And when you are coming up with word salad like
    calling the numerical value of c^2 a frequency, how am I to guess what
    some random number you may have pulled out of your ass is supposed to
    mean? I cannot read your mind, even if it is only a few neurons.

    I think you are just upset that you don’t need a byzantine formulae from GR to come up with the “clock gains” of 5.27e-10 for 4.12r. A simple GM/r ÷f
    does the job just as well. Considering 5.27e-10 for 4.12r is also what GR calculates
    Oh! And by the way. You probably don’t know what 4.12 r is either.
    I’ll give you a hint. It has something to do with the average orbital radius of
    GPS satelittes.

    The average orbital radius of GPS satellites is 5.27e-10? In what units?

    And 10229999.9954 is *still* wrong.

    No more wrong then if you added a further 50 digits to the number.
    Anyways my calculation used 10229999.99543.The 3 got lost in cut and paste.

    You "lost" two more NECESSARY digits in your cut and paste. Most of the
    time when I see that many sig figs I smirk how someone doesn't know how
    to do calculations to the needed accuracy. But this is definitely an
    exception; the precision needed is extreme, about 500 parts per
    trillion, and the full value is given in the GPS specifications to the necessary number of digits. It is 10.2299999954326 MHz. You need all the
    digits to get the correct values.

    But this is crybaby stuff from you. Notice pretty well every relativist reference on
    it uses at most 10.229999.99543.

    No, in the GPS specs it is explicitly stated to be 10.2299999954326 MHz.
    Using fewer digits is simply sloppy and gives you the incorrect answers.

    also gives the wrong value?
    47379430.8842 ÷ 8.9875518e+16 (GPS) = 5.2716726e-10

    GIGO. 8.9875518e+16 is NOT a frequency!

    Since when can any number not be a frequency!!
    Wow, you don’t realise frequencies can be any number
    Including 8.9875518e+16 hz !!!

    But c^2 is NOT a frequency! It doesn't have the correct units of
    frequency (it is m^2/s^2, frequencies have units of 1/s)
    Remember, if the units are incorrect, the answer is AUTOMATICALLY wrong!


    [snip rest of GIGO]

    You are very, very confused.

    Not so confused as to realise that all GR does is divide potential (GM/r) >>> into frequency to get a clock gain of + 5.2716726e-10

    Prove 47379430.8842 ÷ 8.9875518e+16 does not equal 5.2716726e-1
    Cant? Thought not.

    It is actually 5.2716726e-10, but what are 9 orders of magnitude between friends?

    So for instance in the frequency 8.9875518e+16 hz...

    Again, explain how c^2 is a frequency?

    You are completely clueless. Word salad GIGO.

    --- SoupGate-Win32 v1.05
    * Origin: fsxNet Usenet Gateway (21:1/5)
  • From Volney@21:1/5 to Lou on Mon Jan 1 16:12:24 2024
    On 12/31/2023 2:04 PM, Lou wrote:
    On Sunday 31 December 2023 at 17:27:44 UTC, Volney wrote:
    On 12/30/2023 2:40 PM, Lou wrote:

    And that it’s NOT preset before launch.

    As this is where the frequency is specified, it is set by design, and
    most certainly is set before launch. By Design.

    Word salad to cover up the fact that the satelitte broadcasts its
    signal...to the earth receiver...at 10.23Mhz.

    Why does the design specification state explicitly the frequency is 10.2299999954326 MHz, not 10.23 MHz?

    As is confirmed by pretty well every tech spec from NIST etc.


    --- SoupGate-Win32 v1.05
    * Origin: fsxNet Usenet Gateway (21:1/5)
  • From Volney@21:1/5 to Lou on Mon Jan 1 22:47:42 2024
    On 1/1/2024 3:46 PM, Lou wrote:
    On Monday 1 January 2024 at 16:57:33 UTC, Volney wrote:
    On 12/31/2023 2:48 PM, Lou wrote:
    On Sunday 31 December 2023 at 17:19:13 UTC, Volney wrote:
    On 12/31/2023 5:27 AM, Lou wrote:
    On Sunday 31 December 2023 at 03:17:25 UTC, Volney wrote:
    On 12/30/2023 8:07 PM, Lou wrote:
    On Saturday 30 December 2023 at 23:13:15 UTC, Volney wrote:
    On 12/30/2023 3:01 PM, Lou wrote:
    On Saturday 30 December 2023 at 15:51:14 UTC, Volney wrote: >>>>>>>>>> On 12/29/2023 6:11 AM, Lou wrote:
    On Tuesday 26 December 2023 at 20:28:04 UTC, Volney wrote: >>>>>>>>>>>> On 12/26/2023 11:59 AM, Maciej Wozniak wrote:
    On Tuesday 26 December 2023 at 16:48:43 UTC+1, Tom Roberts wrote: >>>>>>>>>>>>>> On 12/26/23 1:37 AM, Maciej Wozniak wrote:

    And with the advent of GPS we now know that GR shit >>>>>>>>>>>>>>> has nothing in common with real clocks, real observers >>>>>>>>>>>>>>> or real anything.
    Not true. We just know that Maciej Wozniak knows nothing about GPS, GR,
    or clocks.

    Spitting and ravings won't help, poor fanatic trash. the >>>>>>>>>>>>> "improper" clocks of GPS will keep measuring t'=t, just >>>>>>>>>>>>> like all serious clocks always did.

    And once again, you got it back asswards. t'≠t is the whole reason why
    the master clock divisor on a GPS satellite is set to 9192631774.1 and
    not 9192631770.

    4.1? I get 4.2 using Pauls method from his website.
    1.000000000457 × 9192661770= 9192661774.2


    10.23 MHz (frequency received on ground)
    10.2299999954326 MHz (transmitted frequency, from gps.gov website) >>>>>>>>>>
    10.23/10.2299999954326 = 1.0000000004464711634446932934426 >>>>>>>>>>
    1.0000000004464711634446932934426 * 9192631770 =
    9,192,631,774.1042450014705502072059

    That's the divisor of a Cs clock to generate an exact 1 pps signal on
    the ground.

    Yes, but let’s try the same calculation using the clock gain of 457 for
    both SR and GR from 10.22999Mhz.
    1.000000000457x 9192661770= 9192661774.2
    Where did you get those numbers from? The second is defined to be >>>>>>>> 9192631770 Cs cycles, not 9192661770.

    Yes, sorry my typos. It is 9192631770.
    And the calculations based on your incorrect value?
    But my point is that you say you can calculate total gains of 457 for 10.22999Mhz
    from the clock gains of another frequency ( that frequency being 8.9875518e+16hz
    That number 8.9875518e+16 is not a frequency. It is c^2.
    which has a gain of +446 which you then use to calculate the gain >>>>>>> of +457 for 10.22999Mhz)
    And you have that value wrong as well.

    47379129.4927 ÷ 10229999.9954= 4.63139095932
    If this above calculation and its resulting value is wrong...

    What is this 47379129.4927 number?

    Please note I did start off trying to be polite. But seeing as I’m talking to
    someone from the gutter ( who doesn’t even know what GM/r is...)
    Ever heard of GM/r ? Obviously not. Because that’s what 47379129.4927 is. >> You never said so. And when you are coming up with word salad like
    calling the numerical value of c^2 a frequency, how am I to guess what
    some random number you may have pulled out of your ass is supposed to
    mean? I cannot read your mind, even if it is only a few neurons.
    I think you are just upset that you don’t need a byzantine formulae from GR
    to come up with the “clock gains” of 5.27e-10 for 4.12r. A simple GM/r ÷f
    does the job just as well. Considering 5.27e-10 for 4.12r is also what GR >>> calculates
    Oh! And by the way. You probably don’t know what 4.12 r is either.
    I’ll give you a hint. It has something to do with the average orbital radius of
    GPS satelittes.
    The average orbital radius of GPS satellites is 5.27e-10? In what units?

    You don’t know the average orbital radius of GPS satelittes? > Try Google.

    The orbital radius of GPS satellites is 26600 km. What units give an
    orbital radius of 5.27e-10? Light years? Not specifying units is very
    sloppy physics.

    (just checked, light years is incorrect but not absurdly so)

    1.00000000044567113 × 10229999.9954326=10230000

    Well yes, the total time dilation times the compensated transmit
    frequency gives the desired receive frequency, that was the whole point
    of compensation. GR is where that number came from. Tell us something new.

    also gives the wrong value?
    47379430.8842 ÷ 8.9875518e+16 (GPS) = 5.2716726e-10

    GIGO. 8.9875518e+16 is NOT a frequency!

    Since when can any number not be a frequency!!
    Wow, you don’t realise frequencies can be any number
    Including 8.9875518e+16 hz !!!

    But c^2 is NOT a frequency! It doesn't have the correct units of

    But 8.9875518e+16 is just a number.

    No, it is not. It is c^2 which is m^2/s^2, which are not units of frequency.

    If you got a frequency 8.9875518e+16 Hz from somewhere else, and it is
    merely a bizarre coincidence that it just happens to be the value of c^2
    in SI units, explain where that frequency comes from.

    And any number can be a
    frequency.

    Well, if you want to use the number which just happens to be the value
    of c^2 in SI units as a frequency, the frequency of WHAT???? Justify
    your answer as relevant to the GPS.

    [snip rest of GIGO]

    frequency (it is m^2/s^2, frequencies have units of 1/s)
    Remember, if the units are incorrect, the answer is AUTOMATICALLY wrong!

    The usual total for relativists GR calculation for clock gains is 5.27e-10 What units is that number in?

    Unitless.

    And Paul and other relativists use 1.000000000527 to calculate frequency gains. In relativity Lala land what units is 1.000000000527 in?

    Unitless. It is a ratio, converting one frequency to another for example.

    And as you may know, in the relativist formula to calculate clock
    gains c^2 is used.
    You say that’s OK to do. OK so tell me...
    In that context what units is your c^2 in? You just said it
    cant be a frequency, nor is m^2/s^2 a unit of speed . What units is it supposed to be then?

    Once again m^2/s^2 which is not a common property like speed, length, or
    area. c is m/s which is a speed. c^2 is part of larger equations, it's
    not used by itself.


    [snip rest of GIGO]

    You are very, very confused.

    Not so confused as to realise that all GR does is divide potential (GM/r) >>>>> into frequency to get a clock gain of + 5.2716726e-10

    Prove 47379430.8842 ÷ 8.9875518e+16 does not equal 5.2716726e-1
    Cant? Thought not.

    It is actually 5.2716726e-10, but what are 9 orders of magnitude between
    friends?

    Thought you couldn’t prove it wrong.

    What do you mean? 47379430.8842 / 8.9875518e+16 is 5.2716726e-10 not 5.2716726e-1, just as I said.

    --- SoupGate-Win32 v1.05
    * Origin: fsxNet Usenet Gateway (21:1/5)
  • From Volney@21:1/5 to Laurence Clark Crossen on Tue Jan 2 02:05:34 2024
    On 1/2/2024 12:25 AM, Laurence Clark Crossen wrote:

    IF RELATIVITY HAD PREDICTED THAT CLOSELY, IT WOULD HAVE KNOWN WHICH SIGN, SO IT DID NOT!

    Since the switch had only two positions, they obviously knew the sign.
    Anyone who understood GR would know the sign. If they didn't know the
    sign they would have needed 3 possible settings, Newton, GR increases
    frequency and GR decreases frequency. They didn't have 3 settings, only two.

    My guess was there was a moron anti-relativity manager on the
    engineering team (who got there via the Peter Principle) who demanded
    the Newtonian setting. The sane engineers demanded GR and finally, as a compromise to prevent the moron anti-relativist manager from ruining the
    whole project, they added the remote control divisor which allowed the
    moron manager to save face and wouldn't blow the budget.

    --- SoupGate-Win32 v1.05
    * Origin: fsxNet Usenet Gateway (21:1/5)
  • From Volney@21:1/5 to Lou on Tue Jan 2 12:40:40 2024
    On 1/2/2024 8:43 AM, Lou wrote:
    On Tuesday 2 January 2024 at 07:05:40 UTC, Volney wrote:
    On 1/2/2024 12:25 AM, Laurence Clark Crossen wrote:

    IF RELATIVITY HAD PREDICTED THAT CLOSELY, IT WOULD HAVE KNOWN WHICH SIGN, SO IT DID NOT!
    Since the switch had only two positions, they obviously knew the sign.
    Anyone who understood GR would know the sign. If they didn't know the
    sign they would have needed 3 possible settings, Newton, GR increases
    frequency and GR decreases frequency. They didn't have 3 settings, only two. >>
    My guess was there was a moron anti-relativity manager

    Sorry about the redundant description of the manager.

    on the
    engineering team (who got there via the Peter Principle) who demanded
    the Newtonian setting. The sane engineers demanded GR and finally, as a
    compromise to prevent the moron anti-relativist manager from ruining the
    whole project, they added the remote control divisor which allowed the
    moron manager to save face and wouldn't blow the budget.

    Very funny. But it wasn’t just the sign that wasn’t known. Neil Ashby himself says it was the * pmagnitude* that also wasn’t known.
    So how could they have preset a specific magnitude before launch,
    when they didn’t know what magnitude to expect?

    Umm, GR is quite explicit in its calculations, the magnitude was clearly
    known as was the sign. Again the only choice was whether to use GR or
    not, a two position switch.

    --- SoupGate-Win32 v1.05
    * Origin: fsxNet Usenet Gateway (21:1/5)
  • From Paul B. Andersen@21:1/5 to All on Tue Jan 2 19:10:43 2024
    Den 02.01.2024 06:25, skrev Laurence Clark Crossen:
    Yes, Ashby's quote clearly proves the effect was learned empirically and not predicted by relativity: “Relativity in the Global Positioning System”

    "There is an interesting story about this frequency offset.
    At the time of launch of the NTS-2 satellite (23 June 1977),
    which contained the first Cesium atomic clock to be placed
    in orbit, it was recognized that orbiting clocks would require
    a relativistic correction, but there was uncertainty as to its
    magnitude as well as its sign. Indeed, there were some who doubted
    that relativistic effects were truths that would need to be
    incorporated [5]!"


    Some may have been uncertain of the sign of the correction, but
    that was certainly not the case for those who built NTS-2, as
    should be blatantly obvious in the following quotation:

    "A frequency synthesizer was built into the satellite clock system
    so that after launch, if in fact the rate of the clock in its final
    orbit was that predicted by general relativity, then the synthesizer
    could be turned on, bringing the clock to the coordinate rate necessary
    for operation. After the Cesium clock was turned on in NTS-2, it was
    operated for about 20 days to measure its clock rate before turning on
    the synthesizer [11]. The frequency measured during that interval was
    +442.5 parts in 10¹² compared to clocks on the ground, while general relativity predicted +446.5 parts in 10¹². The difference was well
    within the accuracy capabilities of the orbiting clock. This then
    gave about a 1% verification of the combined second-order Doppler and gravitational frequency shift effects for a clock at 4.2 earth radii."


    IF RELATIVITY HAD PREDICTED THAT CLOSELY, IT WOULD HAVE KNOWN WHICH SIGN, SO IT DID NOT!

    This is the "interesting story" Ashby was referring to:

    INITIAL RESULTS OF THE NAVSTAR GPS NTS-2 SATELLITE https://paulba.no/paper/Initial_results_of_GPS_satellite_1977.pdf

    Facts:
    The satellite was first run with uncorrected clock for 6.5 days,
    see fig 20.
    The monitor stations measured the offset between the satellite
    clock and the USNO reference clock 11 times between day 198.5
    and day 205, 1977. The offset increased more than 200 μs,
    equivalent to +443.1E-12 too fast.

    "The {T-O) slope gives the frequency offset of +442.5 pp10¹²
    with respect to the PMA clock. Inclusion of the PMA frequency
    offset of +0.6 PP 10¹² produces an NTS measured value
    of +443.1 pp10¹². Comparison of this value to the predicted
    value of the relativistic offset of +445.0 pp10¹² gives
    a difference of -3.1 pp10¹²."


    "On Day 215,1977, the NTS-2 PRO-5 output signal was
    offset {Fig. 21) through the use of a frequency synthesizer".

    This frequency synthesizer lowered the frequency by -445.0E-12.
    After this the frequency offset was +7.9E-13.

    So you see, the correction was calculated and built into
    the frequency synthesizer before launch, and the correction
    proved to be correct within the precision of the clock.

    So it is quite stupid to claim that they didn't know the sign
    of the correction and that the correction was measured, not calculated.
    Don't you agree?

    Note that in 1977 relativity was believed to predict +445.0E-12.
    Ashby's more precise prediction is +446.47E-12 which is the value
    now specified in the interface control document.
    The _correction_ is -446.47E-12.
    If the correction had been -446.47E-12 in stead of -445.0E-12,
    the clock rate error after correction would be -6.8e-13
    in stead of +7.9E-13.



    --
    Paul

    https://paulba.no/

    --- SoupGate-Win32 v1.05
    * Origin: fsxNet Usenet Gateway (21:1/5)
  • From Volney@21:1/5 to Lou on Wed Jan 3 01:06:55 2024
    On 1/2/2024 8:39 AM, Lou wrote:
    On Tuesday 2 January 2024 at 04:14:26 UTC, Laurence Clark Crossen wrote:
    On Monday, January 1, 2024 at 5:39:10 PM UTC-8, Lou wrote:

    And on page 17:
    “ Instead, after such clocks are placed in orbit their frequencies are measured and the actual frequency corrections needed are incorporated in the clock correction polynomial that accompanies the navigation message.”
    Sounds like any corrections are made after launch.

    No, the correction polynomial is for corrections due to the orbit being
    not a perfect circle (sun and moon tug on satellites) or orbital height
    being incorrect.

    And accurately modelled using just GM/r ÷ f
    *If the preset before launch was at 10.22999Mhz.*
    The only problem is...is the sat clock preset to 10.22999Mhz?

    Definitely not, since the specs state it must be set to 10.2299999954326
    MHz.

    Except for that one NIST quote, all the other NIST and other specs say
    it’s not preset before launch. And that the sat clock is preset to
    and runs and broadcasts at 10.23Mhz.

    Again, that's because the discussion is what is RECEIVED, which is 10.23
    MHZ, or a nominal frequency.

    So that’s a contradiction that only a GPS programmer/ engineer can answer. Not any relativist wiki source based on hearsay.

    Any engineer type can understand the spec, even if you can't.

    All I can say is I also found a quora quote from a NASA GPS engineer Ortega who very explicitly says....the clocks are NOT preset to 10.22999Mhz
    before launch.

    Of course he'd say that! That's because the clock is set to
    10.2299999954326 MHz.

    --- SoupGate-Win32 v1.05
    * Origin: fsxNet Usenet Gateway (21:1/5)
  • From Volney@21:1/5 to Lou on Wed Jan 3 00:59:28 2024
    On 1/2/2024 5:57 AM, Lou wrote:
    On Tuesday 2 January 2024 at 03:47:48 UTC, Volney wrote:
    On 1/1/2024 3:46 PM, Lou wrote:


    1.00000000044567113 × 10229999.9954326=10230000

    Well yes, the total time dilation times the compensated transmit
    frequency gives the desired receive frequency,

    OoH!! not using proper SI units. Your answers must be automatically
    wrong.

    I was quoting YOU so that's YOUR mistake.

    Oh but I forgot. You can break your own stupid rules.

    Not my rules but several physics teachers have told their classes that
    if the units are wrong, the answer is automatically wrong even if
    numerically correct.

    Since when can a frequency not be 8.9875518e+16 ?
    Or any number for that matter?

    If you got a frequency 8.9875518e+16 Hz from somewhere else, and it is
    merely a bizarre coincidence that it just happens to be the value of c^2
    in SI units, explain where that frequency comes from.

    What? Like the way GR uses c^2 as a frequency.

    No, GR does not use c^2 as a frequency. The units are wrong, for one.
    See above about the professors.

    Even though it breaks
    the SI rules?

    I didn’t actually start with 8.9875518e+16 Hz. I started with 9192631770Hz seeing as that’s the frequency of a sats caesium clock.
    Found it worked.

    Yes, the Cs frequency is a frequency.

    Then tried 10.29 and then looked at the GR formula and realised that
    the c^2 in the GR formula was actually being covertly used as a frequency too.

    No, it isn't. Again, the units are wrong.

    Otherwise how did whoever invented that formula fantasise that 5.27e-10 was frequency related.?

    It is a ratio difference, not a frequency.

    And any number can be a
    frequency.

    Well, if you want to use the number which just happens to be the value
    of c^2 in SI units as a frequency, the frequency of WHAT???? Justify
    your answer as relevant to the GPS.

    And what does your 8.9875518e+16 m^2/s^2 have to do with frequency?
    Cant answer? Thought not.

    I see YOU can't answer (frequency of WHAT), much less justify your
    nonexistent answer.

    [snip rest of GIGO]

    frequency (it is m^2/s^2, frequencies have units of 1/s)
    Remember, if the units are incorrect, the answer is AUTOMATICALLY wrong! >>>
    The usual total for relativists GR calculation for clock gains is 5.27e-10 >>> What units is that number in?

    Unitless.

    Ha ha. Oh dear. Just broke your own rules again.

    Nope. Getting the units correct does include unitless values, such as
    pi, the fine structure constant among others.

    And Paul and other relativists use 1.000000000527 to calculate frequency >>> gains. In relativity Lala land what units is 1.000000000527 in?

    Unitless. It is a ratio, converting one frequency to another for example.

    And as you may know, in the relativist formula to calculate clock
    gains c^2 is used.
    You say that’s OK to do. OK so tell me...
    In that context what units is your c^2 in? You just said it
    cant be a frequency, nor is m^2/s^2 a unit of speed . What units is it
    supposed to be then?

    Once again m^2/s^2 which is not a common property like speed, length, or
    area. c is m/s which is a speed. c^2 is part of larger equations, it's
    not used by itself.

    Thought you couldn’t answer

    No, that IS the answer.

    Here try again:
    What SI units is 8.9875518e+16 m^2/s^2 in your formula?
    Cant answer? That’s because you just broke your own silly rules.

    No, by itself it's not a secondary unit at all because either no
    standalone property uses that or it's one that's extremely obscure and I
    don't know it. Anyway, c^2 is always part of a larger equation.


    [snip rest of GIGO]

    You are very, very confused.

    Not so confused as to realise that all GR does is divide potential (GM/r)
    into frequency to get a clock gain of + 5.2716726e-10

    Prove 47379430.8842 ÷ 8.9875518e+16 does not equal 5.2716726e-1
    Cant? Thought not.

    It is actually 5.2716726e-10, but what are 9 orders of magnitude between >>>> friends?

    I told you multiple times it was 5.2716726e-10, Starting on Dec 21.

    Sloppy sloppy sloppy. You wrote 5.2716726e-1 and I corrected you.


    Thought you couldn’t prove it wrong.

    What do you mean? 47379430.8842 / 8.9875518e+16 is 5.2716726e-10 not
    5.2716726e-1, just as I said.

    I told you multiple times it was 5.2716726e-10, Starting on Dec 21. Heres
    my quote from my post to you back on december 31, repeated twice that
    day then again on Jan 1, Etc...
    “47379430.8842 ÷ 8.9875518e+16 (GPS) = 5.2716726e-10 “

    Again, sloppy sloppy for writing 5.2716726e-1.

    So answer the questions
    What SI units are are 8.9875518e+16m^2/s^2 and 5.27e-10 in your formulas?

    Asked and answered.

    Cant answer?

    I already did.

    Then your GR formulas answers are, using your own logic,
    also completely wrong.

    Since I provided you with the answers, this doesn't apply.

    And incidentally, contrary to your mistaken belief that Hz is not a SI unit...

    When did I say that?

    ...the Hz of 8.9875518e+16hz I my formula IS an SI unit.

    Again, frequency of WHAT?

    So using it, or any other frequency in the formula GM/r ÷f is correct.

    The units of the result of that equation are incorrect (supposed to be unitless), so that's automatically wrong.

    Unlike your rule breaking m^2/ s^2 which isn’t unless you can answer the following question...
    What units is the c^2 in your GR formula?

    c^2 is part of a larger formula. For example, the equation E=mc^2 the
    right side has units of mass times distance^2 divided by time^2 (c=distance/time). That combination has units of energy (joules in SI)
    which is good because the E on the left side represents energy.

    --- SoupGate-Win32 v1.05
    * Origin: fsxNet Usenet Gateway (21:1/5)
  • From Paul B. Andersen@21:1/5 to All on Wed Jan 3 15:13:44 2024
    Den 02.01.2024 21:42, skrev Lou:
    On Tuesday 2 January 2024 at 18:07:32 UTC, Paul B. Andersen wrote:
    Den 02.01.2024 06:25, skrev Laurence Clark Crossen:
    Yes, Ashby's quote clearly proves the effect was learned empirically and not predicted by relativity: “Relativity in the Global Positioning System”

    "There is an interesting story about this frequency offset.
    At the time of launch of the NTS-2 satellite (23 June 1977),
    which contained the first Cesium atomic clock to be placed
    in orbit, it was recognized that orbiting clocks would require
    a relativistic correction, but there was uncertainty as to its
    magnitude as well as its sign. Indeed, there were some who doubted
    that relativistic effects were truths that would need to be
    incorporated [5]!"


    Some may have been uncertain of the sign of the correction, but
    that was certainly not the case for those who built NTS-2, as
    should be blatantly obvious in the following quotation:

    "A frequency synthesizer was built into the satellite clock system
    so that after launch, if in fact the rate of the clock in its final
    orbit was that predicted by general relativity, then the synthesizer
    could be turned on, bringing the clock to the coordinate rate necessary
    for operation. After the Cesium clock was turned on in NTS-2, it was
    operated for about 20 days to measure its clock rate before turning on
    the synthesizer [11]. The frequency measured during that interval was
    +442.5 parts in 10¹² compared to clocks on the ground, while general
    relativity predicted +446.5 parts in 10¹². The difference was well
    within the accuracy capabilities of the orbiting clock. This then
    gave about a 1% verification of the combined second-order Doppler and
    gravitational frequency shift effects for a clock at 4.2 earth radii."


    There is nothing in this quote that says the magnitude 446 was predicted before launch. Ask any lawyer. Look at the definitive part of the quote you rely
    on “ while general relativity predicted +446.5 parts in 10¹². “ It does not specify
    whether this prediction was pre or post.

    Read the quotation again.
    The frequency synthesizer was built to lower the frequency
    by the amount predicted by GR.
    If you don't understand that the prediction must have been made
    before the satellite was built, then you have a serious problem.

    That quote is completely consistent with the theorists looking at clock gains...
    And then figuring out how to predict something that matches it.
    Proof is..Have you a quote from *pre launch* that says Relativity predicts 446ps?


    And the second point you ignore is: Yes, you have one vague
    quote suggesting it’s preset to 10.229999.89543.

    It's not "suggested", it is _specified_
    According to the Interface _SPECIFICATION_ document: https://www.gps.gov/technical/icwg/IS-GPS-200N.pdf

    "The nominal frequency of this source -- as it appears to
    an observer on the ground -- is 10.23 MHz."
    "The clock rates are offset by Δf/f = -4.4647E-10"
    "This is equal to 10.2299999954326 MHz."

    If the satellites were not built according to specification,
    they wouldn't work,

    But why does the NIST specs
    you refer to and all others Ive seen, then clearly specify on at least a dozen other
    times in the same paper that the on board sat clock is set to run at 10.23Mhz?

    Because the nominal frequency of the frequency standard is 10.23Mhz.
    It's always the nominal frequencies that are specified.




    IF RELATIVITY HAD PREDICTED THAT CLOSELY, IT WOULD HAVE KNOWN WHICH SIGN, SO IT DID NOT!

    This is the "interesting story" Ashby was referring to:

    INITIAL RESULTS OF THE NAVSTAR GPS NTS-2 SATELLITE
    https://paulba.no/paper/Initial_results_of_GPS_satellite_1977.pdf

    Facts:
    The satellite was first run with uncorrected clock for 6.5 days,
    see fig 20.
    The monitor stations measured the offset between the satellite
    clock and the USNO reference clock 11 times between day 198.5
    and day 205, 1977. The offset increased more than 200 μs,
    equivalent to +443.1E-12 too fast.


    You will have to clarify this. Because you yourself in a earlier thread
    said very clearly that classical Doppler shifting of signals due to
    satelitte orbital speeds relative to ground far outweigh any predicted effects from relativity. Seeing as all satelittes orbit at different
    paths vectors v relative to any single ground observer it would be impossible to
    say any relativistic effects are apparent. It would be like saying
    we can measure the air flow from your cough from ten feet away
    in the middle of a hurricane.

    Of course no relativistic phenomena are apparent to the receiver.
    Why should it?
    The receiver doesn't measure any frequencies.
    The receiver receives two modulated carrier signals,
    with information about what the SV-time was when
    the signal was sent. With this information from four
    satellites, the receiver can calculate its time and position.


    This would concur with all the NIST and other specs which clearly
    and on numerous occasions say the sat clock is at set at 10.23Mhz.
    And that it is corrected daily. Don’t forget. The engineers don’t really care about relativity being confirmed. They just want to have a sat
    clock that can be adjusted daily. Even the atomic clocks themselves apparently routinely screw up their timekeeping.
    It doesn’t even make sense to worry about a preset clock at 10.22Mhz
    If daily gains/losses from Doppler were much larger than any minute
    daily gains accrued from relativity were also present.

    You are obviously very ignorant of how the GPS work
    and how the time is corrected.
    https://paulba.no/div/GPS_clock_correction.pdf

    t_SV is the time shown by the SV-clock.
    t_SV is never corrected while the SV is in service,
    and its error Δ@t_SV will typically be several μs.

    t_SV is sent from the SV to the receiver together with
    a few correctional parameters, the most important of
    which is the clock offset a_f0.
    The receiver can then calculate the correct system tine t,
    that is the time when the signal was sent.
    See reference above.

    Note that the correctional parameters are measured by
    the monitor stations, and uploaded to the SVs when needed,
    typically once a day. This is the "daily adjustment" you
    mentioned, _but the SV_clock is NOT adjusted_.

    Bbecause of the number of bits a_f0 is coded with,
    the clock offset must be less that 1 ms, this means
    that the SV clock error Δ@t_SV must be less then 1 ms.

    If the SV-clock was not corrected by the factor (1-4.4646E-10),
    then a_f0 would overflow after less than 26 days, and
    the SY wouldn't work.

    Bottom line:
    The _only_ reason for the GR-correction is to keep th SV clock
    correct within 1 ms.


    "The {T-O) slope gives the frequency offset of +442.5 pp10¹²
    with respect to the PMA clock. Inclusion of the PMA frequency
    offset of +0.6 PP 10¹² produces an NTS measured value
    of +443.1 pp10¹². Comparison of this value to the predicted
    value of the relativistic offset of +445.0 pp10¹² gives
    a difference of -3.1 pp10¹²."


    "On Day 215,1977, the NTS-2 PRO-5 output signal was
    offset {Fig. 21) through the use of a frequency synthesizer".

    This frequency synthesizer lowered the frequency by -445.0E-12.
    After this the frequency offset was +7.9E-13.

    So you see, the correction was calculated and built into
    the frequency synthesizer before launch, and the correction
    proved to be correct within the precision of the clock.

    So it is quite stupid to claim that they didn't know the sign
    of the correction and that the correction was measured, not calculated.
    Don't you agree?


    Can you cite a pre launch pre 1977 paper predicting exactly +446 gains
    for relativity?


    So you don't agree.
    But it IS stupid!


    --
    Paul

    https://paulba.no/

    --- SoupGate-Win32 v1.05
    * Origin: fsxNet Usenet Gateway (21:1/5)
  • From Volney@21:1/5 to Laurence Clark Crossen on Wed Jan 3 19:31:40 2024
    On 1/3/2024 1:59 PM, Laurence Clark Crossen wrote:

    The clock adjustment for gravity is not about relativity. Only gravity. It was obtained empirically.

    How the hell could it have been set empirically since the time offset
    was preset BEFORE the launch? NTS-2 was the first satellite EVER to be
    launched with Cs clocks so there was no other source of empirical data,
    either.

    And your statement "gravity is not about relativity" is contradictory as gravity is an effect of general relativity.

    --- SoupGate-Win32 v1.05
    * Origin: fsxNet Usenet Gateway (21:1/5)
  • From Volney@21:1/5 to Lou on Wed Jan 3 19:18:25 2024
    On 1/3/2024 9:27 AM, Lou wrote:
    Snip rest of volney nonsense

    Unlike your rule breaking m^2/ s^2 which isn’t unless you can answer the >>> following question...
    What units is the c^2 in your GR formula?

    c^2 is part of a larger formula. For example, the equation E=mc^2 the
    right side has units of mass times distance^2 divided by time^2
    (c=distance/time). That combination has units of energy (joules in SI)
    which is good because the E on the left side represents energy.

    Pure contradiction from Volney. Says I have to use SI units..otherwise
    any result I get is automatically incorrect.

    Quit making up crap and attributing it to me. I only mention SI units as
    what energy is measured in (joules).

    Having the correct units means having the correct fundamental units
    (length, mass, time) correct. All this means is you can't measure the
    size of a farm field in seconds as area is length^2. Nor can you measure
    time in kilograms* etc. Nor measure the square of a velocity, such as
    the speed of light, in hertz (1/second).

    Even though the result it is exactly
    the same as that predicted by GR down to at least e-12 digits.
    But then he can’t say what SI units c^2 ( m^2/s^2) is in the GR formulas. So answer the question.
    Which SI unit is c^2 in your preferred formula?
    Cant answer again?

    Once again, square meters/square seconds. It should be blazingly obvious
    since speed is meters/second and c is squared.

    According to Volney logic it looks like GRs calculation using the
    incorrect SI unit of c^2...is incorrect.

    Nope. For E=mc^2, E is energy, m is mass, c is length/time, so the right
    side is mass*length^2/time^2, which are the dimensions of energy. Both
    sides have dimensions of energy so it passes the "automatically wrong" test.

    --- SoupGate-Win32 v1.05
    * Origin: fsxNet Usenet Gateway (21:1/5)
  • From Volney@21:1/5 to Lou on Wed Jan 3 19:45:32 2024
    On 1/3/2024 10:42 AM, Lou wrote:
    On Wednesday 3 January 2024 at 14:10:32 UTC, Paul B. Andersen wrote:
    Den 02.01.2024 21:42, skrev Lou:

    Can you cite a pre launch pre 1977 paper predicting exactly +446 gains
    for relativity?

    So you don't agree.
    But it IS stupid!

    It’s not stupid to read multiple quotes from your spec saying presets are 10.23
    Maybe there is a good reason. And you can explain.
    So...Rather than repeatedly telling you the spec you cite contains many
    these references to10.23 and having you ignore these quotes
    I’ve replaced that post and just copied and pasted from your doc specs itself.
    So you can’t dispute it.
    Yes you have one sort of reference to 10.22,...But why then does the spec go on
    to contradict that quote in numerous other places? Here’s some below:

    It looks like you did an uneducated blind search for 10.23 and 1023,
    without interpreting their meanings.

    3.2.1.1 P-Code
    IS-GPS-200N 01-AUG-2022
    The PRN P-code for SV ID number i, for i = 1 to 37, is a ranging code, Pi(t), of 7 days in length at a chipping rate of 10.23 Mbps.

    Nominal on the ground, where it is exactly 10.23 MHz. Not a statement
    stating the transmitter is to be set to 10.23 MHz.


    3.2.1.3 C/A-Code
    The PRN C/A-code for SV ID number i is a Gold code, Gi(t), of 1 millisecond in length at a chipping rate of 1023 kbps. The Gi(t) sequence is a linear pattern generated by the modulo-2 addition of two sub-sequences, G1 and G2i, each of which is a 1023
    chip long linear pattern.

    First is the same. The 1023 is a count.

    3.2.1.4 L2 CM-Code (IIR-M, IIF, and subsequent blocks)
    The PRN L2 CM-code for SV ID number i is a ranging code, CM,i(t), which is 20 milliseconds in length at a chipping rate of 511.5 kbps. The epochs of the L2 CM-code are synchronized with the X1 epochs of the P-code. The CM,i(t) sequence is a linear
    pattern which is short cycled every count of 10230 chips by resetting with a specified initial state. Assignment of initial states by GPS PRN signal number is given in Table 3-IIa.

    That's 10230, and it is a count, not a frequency of 10.23 MHz.

    Table 3-IIa, Table 3-IIb
    * Short cycled period = 10230 **

    Again, a count. See above.

    3.2.3 L1/L2 Signal Structure
    ....The L2 CM-code with the 50 sps symbol stream of DC(t) is time-multiplexed with L2 CL- code at a 1023 kHz rate as described in paragraph 3.2.2.

    Again on the ground nominal.

    3.3.2 PRN Code Characteristics
    IS-GPS-200N 01-AUG-2022
    The characteristics of the P-, L2 CM-, L2 CL-, and the C/A-codes are defined below in terms of their structure and the basic method used for generating them. Figure 3-1 depicts a simplified block diagram of the scheme for generating the 10.23 Mbps
    Pi(t) and the 1.023 Mbps Gi(t) patterns (referred to as P- and C/A-codes respectively), and for modulo-2 summing these patterns with the LNAV bit train, D(t), which is clocked at 50 bps. The resultant composite bit trains are then used to modulate the
    signal carriers.

    Again, nominal, no statement of setting the clock to 10.23 MHz.

    3.3.2.1 Code Structure
    For PRN codes 1 through 37, the Pi(t) pattern (P-code) is generated by the modulo-2 summation of two PRN codes, X1(t) and X2(t - iT), where T is the period of one P-code chip and equals (1.023E7)-1 seconds,

    A count of seconds.

    3.3.2.4 L2 CM-/L2 CL-Code Generation
    IS-GPS-200N 01-AUG-2022
    Each CM,i(t) pattern (L2 CM-code) and CL,i(t) pattern (L2 CL-code) are generated using the same code generator polynomial each clocked at 511.5 kbps. Each pattern is initiated and reset with a specified initial state (defined in Table 3-II). CM,i(t)
    pattern is reset after 10230 chips resulting in a code period of 20 milliseconds,

    Another count.

    Figure 3-1. , Figure 3-6. Figure 3-1, Figure 3-11, Figure 3-12. ..all Clearly say 10.23

    Nominal again.

    For you to be correct, you have to find ONE statement stating "The base frequency is 10.23 MHz." Since there already is one statement stating
    "The base frequency is 10.2299999954326 MHz", it cannot happen since
    otherwise the spec would contradict itself if it did.

    So show us the statement "The base frequency is 10.23 MHz." in the spec.

    --- SoupGate-Win32 v1.05
    * Origin: fsxNet Usenet Gateway (21:1/5)
  • From Volney@21:1/5 to Laurence Clark Crossen on Wed Jan 3 23:27:17 2024
    On 1/3/2024 10:27 PM, Laurence Clark Crossen wrote:
    On Wednesday, January 3, 2024 at 4:31:47 PM UTC-8, Volney wrote:
    On 1/3/2024 1:59 PM, Laurence Clark Crossen wrote:

    The clock adjustment for gravity is not about relativity. Only gravity. It was obtained empirically.
    How the hell could it have been set empirically since the time offset
    was preset BEFORE the launch? NTS-2 was the first satellite EVER to be
    launched with Cs clocks so there was no other source of empirical data,
    either.

    And your statement "gravity is not about relativity" is contradictory as
    gravity is an effect of general relativity.

    The time offset can not have been preset before launch because they only found out how much to adjust it after launch.

    And why do you claim that silliness? And do you have any evidence of it?
    I didn't think so.

    Also first you said it was preset empirically (with no data to set it
    to) and now you claim it was not preset at all?

    The two-switch story is a fairy tale.

    Again why do you claim that? Evidence? Again, I didn't think so. And why
    is the switch right in the paper

    Gravity is not an effect of relativity because a theory does not cause anything. It should explain nature rationally.

    And general relativity does just that.

    Relativity is not about anything since it is not a theory.

    Why do you say that? It meets all the requirements of scientific theories.

    Relativity only pretends to explain the cause of gravity.

    Why "pretends"? It gives a logical, scientific description and explanation.

    Gravity is not even electromagnetism.

    Of course not. Just like GR says it has nothing to do with electromagnetism.

    I really can't help you with your confusion.

    It is you who is confused, thinking things like GR says gravity is electromagnetism, strange beliefs about NTS-2, don't understand what a
    theory is in science, etc.

    --- SoupGate-Win32 v1.05
    * Origin: fsxNet Usenet Gateway (21:1/5)
  • From Volney@21:1/5 to Lou on Thu Jan 4 13:08:40 2024
    On 1/4/2024 5:20 AM, Lou wrote:
    On Thursday 4 January 2024 at 00:45:36 UTC, Volney wrote:
    On 1/3/2024 10:42 AM, Lou wrote:
    On Wednesday 3 January 2024 at 14:10:32 UTC, Paul B. Andersen wrote:
    Den 02.01.2024 21:42, skrev Lou:

    Can you cite a pre launch pre 1977 paper predicting exactly +446 gains >>>>> for relativity?

    So you don't agree.
    But it IS stupid!

    It’s not stupid to read multiple quotes from your spec saying presets are 10.23
    Maybe there is a good reason. And you can explain.
    So...Rather than repeatedly telling you the spec you cite contains many
    these references to10.23 and having you ignore these quotes
    I’ve replaced that post and just copied and pasted from your doc specs itself.
    So you can’t dispute it.
    Yes you have one sort of reference to 10.22,...But why then does the spec go on
    to contradict that quote in numerous other places? Here’s some below:
    It looks like you did an uneducated blind search for 10.23 and 1023,
    without interpreting their meanings.

    3.2.1.1 P-Code
    IS-GPS-200N 01-AUG-2022
    The PRN P-code for SV ID number i, for i = 1 to 37, is a ranging code, Pi(t), of 7 days in length at a chipping rate of 10.23 Mbps.
    Nominal on the ground, where it is exactly 10.23 MHz. Not a statement
    stating the transmitter is to be set to 10.23 MHz.

    3.2.1.3 C/A-Code
    The PRN C/A-code for SV ID number i is a Gold code, Gi(t), of 1 millisecond in length at a chipping rate of 1023 kbps. The Gi(t) sequence is a linear pattern generated by the modulo-2 addition of two sub-sequences, G1 and G2i, each of which is a 1023
    chip long linear pattern.
    First is the same. The 1023 is a count.

    3.2.1.4 L2 CM-Code (IIR-M, IIF, and subsequent blocks)
    The PRN L2 CM-code for SV ID number i is a ranging code, CM,i(t), which is 20 milliseconds in length at a chipping rate of 511.5 kbps. The epochs of the L2 CM-code are synchronized with the X1 epochs of the P-code. The CM,i(t) sequence is a linear
    pattern which is short cycled every count of 10230 chips by resetting with a specified initial state. Assignment of initial states by GPS PRN signal number is given in Table 3-IIa.
    That's 10230, and it is a count, not a frequency of 10.23 MHz.

    Table 3-IIa, Table 3-IIb
    * Short cycled period = 10230 **
    Again, a count. See above.

    3.2.3 L1/L2 Signal Structure
    ....The L2 CM-code with the 50 sps symbol stream of DC(t) is time-multiplexed with L2 CL- code at a 1023 kHz rate as described in paragraph 3.2.2.
    Again on the ground nominal.

    3.3.2 PRN Code Characteristics
    IS-GPS-200N 01-AUG-2022
    The characteristics of the P-, L2 CM-, L2 CL-, and the C/A-codes are defined below in terms of their structure and the basic method used for generating them. Figure 3-1 depicts a simplified block diagram of the scheme for generating the 10.23 Mbps Pi(
    t) and the 1.023 Mbps Gi(t) patterns (referred to as P- and C/A-codes respectively), and for modulo-2 summing these patterns with the LNAV bit train, D(t), which is clocked at 50 bps. The resultant composite bit trains are then used to modulate the
    signal carriers.
    Again, nominal, no statement of setting the clock to 10.23 MHz.
    3.3.2.1 Code Structure
    For PRN codes 1 through 37, the Pi(t) pattern (P-code) is generated by the modulo-2 summation of two PRN codes, X1(t) and X2(t - iT), where T is the period of one P-code chip and equals (1.023E7)-1 seconds,
    A count of seconds.

    3.3.2.4 L2 CM-/L2 CL-Code Generation
    IS-GPS-200N 01-AUG-2022
    Each CM,i(t) pattern (L2 CM-code) and CL,i(t) pattern (L2 CL-code) are generated using the same code generator polynomial each clocked at 511.5 kbps. Each pattern is initiated and reset with a specified initial state (defined in Table 3-II). CM,i(t)
    pattern is reset after 10230 chips resulting in a code period of 20 milliseconds,
    Another count.

    Figure 3-1. , Figure 3-6. Figure 3-1, Figure 3-11, Figure 3-12. ..all Clearly say 10.23
    Nominal again.

    For you to be correct, you have to find ONE statement stating "The base
    frequency is 10.23 MHz." Since there already is one statement stating
    "The base frequency is 10.2299999954326 MHz", it cannot happen since
    otherwise the spec would contradict itself if it did.

    So show us the statement "The base frequency is 10.23 MHz." in the spec.


    Here you go Volney boy. Seeing as you havent learnt how to google
    I did it for you
    It says clearly at the top that “ Three signals are transmitted at the moment by GPS “ And then goes on to specify these 3 variations.
    One of the three copied below. Additionally it refers to these as SV signals. SV means GPS satelitte. Ask Paul. He just posted a couple days ago
    a comment in which he says that SV means....Sat clock. Savvy?
    I don’t see how you can assume this refers to anything but the transmission signal or clock frequency of the satelitte.

    https://gssc.esa.int/navipedia/index.php/GPS_Signal_Plan#:~:text=The%20PRN%20C%2FA%20Code,1023%20chip%20long%20linear%20pattern.



    “ GPS L2 Band
    GPS is transmitting in the L2 band (1227.60 MHz) a modernized civil signal known as L2C designed specifically to meet commercial needs as it enables the development of dual-frequency solutions; together with the P(Y) Code and the M-Code. The P(Y) Code
    and M-Code were already described shortly in the previous chapter and the properties and parameters are thus similar to those in the L1 band. In addition, for Block IIR-M, IIF, and subsequent blocks of SVs, two additional PRN ranging codes are
    transmitted. They are the L2 Civil Moderate (L2 CM) code and the L2 Civil Long (L2 CL) code. These two signals are time multiplexed so that the resulting chipping rate is double as high as that of each individual signal. We further describe them in the
    next lines more in detail:
    L2 CM Code is transmitted in the IIR-M, IIF, III and subsequent blocks. The PRN L2 CM Code for SV number i is a ranging code, which is 20 milliseconds in length at a chipping rate of 511.5 Kbps. The epochs of the L2 CM Code are synchronized with the X1
    epochs of the P-code. The CM sequence is a linear pattern which is short cycled every count period of ***10,230*** chips by resetting with a particular initial state. “


    Does that say 10.23 or 10229999.9954326?

    Neither. It says 10,230, which is a count of chips, not a frequency.
    As I stated, you obviously did a simple text search of 10.23 or 1023,
    without understanding what you found.

    --- SoupGate-Win32 v1.05
    * Origin: fsxNet Usenet Gateway (21:1/5)
  • From Volney@21:1/5 to Lou on Thu Jan 4 13:03:58 2024
    On 1/4/2024 4:57 AM, Lou wrote:
    On Thursday 4 January 2024 at 00:45:36 UTC, Volney wrote:

    So show us the statement "The base frequency is 10.23 MHz." in the spec.

    Hilarious. The specs are about the GPS satellite. And nominal frequency
    [bla bla bla]

    So you cannot show us the statement "The base frequency is 10.23 MHz."
    anywhere in the spec, and this is resolved. The transmitter is preset to 10.2299999954326 MHz, as defined in the spec, and as predicted by GR.

    --- SoupGate-Win32 v1.05
    * Origin: fsxNet Usenet Gateway (21:1/5)
  • From Paul B. Andersen@21:1/5 to All on Thu Jan 4 22:09:47 2024
    Den 03.01.2024 15:45, skrev Lou:
    On Wednesday 3 January 2024 at 14:10:32 UTC, Paul B. Andersen wrote:
    According to the Interface _SPECIFICATION_ document:
    https://www.gps.gov/technical/icwg/IS-GPS-200N.pdf

    "The nominal frequency of this source -- as it appears to
    an observer on the ground -- is 10.23 MHz."
    "The clock rates are offset by Δf/f = -4.4647E-10"
    "This is equal to 10.2299999954326 MHz."

    If the satellites were not built according to specification,
    they wouldn't work,


    Where’s your citation and quote from pre 1977 launch predicting
    Total gains of 446ps at 4.12 Earth radius?
    Cant find it?

    What predicted a total gain of 446 picoseconds at 4.12 Earth radius?
    Confused, Lou?

    Why does the Neil Ashby paper, copied on your website, saying the magnitude was not known pre launch contradict your false assumption it was?

    It isn't very smart to lie about something that
    is so easy to check:

    https://paulba.no/paper/Ashby.pdf

    page 16:
    "At the time of launch of the NTS-2 satellite (23 June 1977)
    . . . general relativity predicted +446.5 parts in 10¹²."

    The clock was predicted to run fast by +446.5 parts in 10¹²
    so the correction is -446.5 parts in 10¹².

    page 17:
    "When GPS satellites were first deployed, the specified
    factory frequency offset was slightly in error because
    the important contribution from earth’s centripetal potential
    had been inadvertently omitted at one stage of the evaluation.
    Although GPS managers were made aware of this error in the early
    1980s, eight years passed before system specifications were
    changed to reflect the correct calculation'."

    Which means that before the first GPS satellite was launched,
    the specified frequency offset was -446.5E-12 which
    is slightly different from the correct value +446.47E-12.

    Since the difference is less than the precision of the clocks,
    the error had no serious consequences, so the GPS did work
    before 1988 when the correct value was specified in the IS document.

    Of course no relativistic phenomena are apparent to the receiver.
    Why should it?
    The receiver doesn't measure any frequencies.
    The receiver receives two modulated carrier signals,
    with information about what the SV-time was when
    the signal was sent. With this information from four
    satellites, the receiver can calculate its time and position.


    Interesting obfuscation . To start with it is clear the sat clock
    is preset to 10.23Mz. You can’t ignore the dozens of times this is
    stated even in this one doc.
    And..Of course no relativistic phenomena would be apparent to the receiver. That’s because the sat signal is swamped by much larger classical Doppler shifting of the signal every day. As you yourself admit elsewhere.
    What you also fail to understand is that even if there ARE relativistic clock gains it would make no difference nor be measureable. Because daily Doppler shifting
    and any on board time glitches by the atomic clock would make it impossible to seperate out any smaller GR and SR effects . That’s why the clock needs to be corrected
    daily.
    In fact if you think about it, any satelitte engineer designing the GPS system would have to design in a daily correctable clock to correct all the above non relativistic clock gains/losses. Even if no
    Relativistic gains/losses were ever expected.

    A beautiful demonstration of your utter ignorance
    of how the GPS works!

    Here is an explanation of how the time reported from
    the SV is corrected:

    You are obviously very ignorant of how the GPS work
    and how the time is corrected.
    https://paulba.no/div/GPS_clock_correction.pdf

    t_SV is the time shown by the SV-clock.
    t_SV is never corrected while the SV is in service,
    and its error Δ@t_SV will typically be several μs.

    t_SV is sent from the SV to the receiver together with
    a few correctional parameters, the most important of
    which is the clock offset a_f0.
    The receiver can then calculate the correct system tine t,
    that is the time when the signal was sent.
    See reference above.

    Note that the correctional parameters are measured by
    the monitor stations, and uploaded to the SVs when needed,
    typically once a day. This is the "daily adjustment" you
    mentioned, _but the SV_clock is NOT adjusted_.

    Bbecause of the number of bits a_f0 is coded with,
    the clock offset must be less that 1 ms, this means
    that the SV clock error Δ@t_SV must be less then 1 ms.

    If the SV-clock was not corrected by the factor (1-4.4646E-10),
    then a_f0 would overflow after less than 26 days, and
    the SY wouldn't work.

    Bottom line:
    The _only_ reason for the GR-correction is to keep the SV clock
    correct within 1 ms.

    You didn't even read this, did you?


    Smart enough to be able to read your NIST specs and see
    that it says very clearly numerous times...the sat broadcast f is 10.23Mhz.

    But not smart enough to understand what you read in said document.

    Neither 10.23 MHz nor 10.2299999954326 MHz are broadcasted from the SV! =======================================================================

    The frequencies broadcasted from the satellites are the carrier
    frequencies L1 and L2.

    Let's quote from the Interface Specification Document again:

    https://www.gps.gov/technical/icwg/IS-GPS-200N.pdf
    3.3.1.1 Frequency Plan
    "For Block IIR, IIR-M, and IIF satellites, the requirements
    specified in this IS shall pertain to the signal contained
    within two 20.46 MHz bands; one centered about the L1 nominal
    frequency and the other centered about the L2 nominal frequency
    (see Table 3-Vb). The carrier frequencies for the L1 and L2
    signals shall be coherently derived from a common frequency
    source within the SV. The nominal frequency of this source
    -- as it appears to an observer on the ground -- is 10.23 MHz.
    The SV carrier frequency and clock rates -- as they would
    appear to an observer located in the SV -- are offset to
    compensate for relativistic effects. The clock rates are offset
    by Δf/f = -4.4647E-10, equivalent to a change in the P-code chipping
    rate of 10.23 MHz offset by a f = -4.5674E-3 Hz. This is equal to 10.2299999954326 MHz.
    The nominal carrier frequencies (f0) shall be 1575.42 MHz,
    and 1227.6 MHz for L1 and L2, respectively."

    ------

    You are obviously not able to understand what this means,
    so let us explain.

    READ THIS!
    ==========
    The frequencies broadcasted from the satellites are
    the carrier frequencies L1 and L2.
    The nominal carrier frequencies are 1575.42 MHz and 1227.6 MHz,
    but since "the SV carrier frequency and clock rates are offset
    by Δf/f = -4.4647E-10", the actual frequencies broadcasted from
    the SV are: L1 = 1575.4200007033778 Mhz and L2 = 1227.6000005480864.

    Since L1 and L2 are modulated with the P-code with
    the offset shipping rate "equal to 10.2299999954326 MHz",
    the actual bandwidth of L1 and L2 is 20.4599999908652 MHz.
    (which for all practical purposes is the same as 20.46 MHz)

    Shannon's principle, you know? Of course you don't-

    --
    Paul

    https://paulba.no/

    --- SoupGate-Win32 v1.05
    * Origin: fsxNet Usenet Gateway (21:1/5)
  • From Paul B. Andersen@21:1/5 to All on Thu Jan 4 22:25:18 2024
    Den 03.01.2024 18:02, skrev Prokaryotic Capase Homolog:
    On Wednesday, January 3, 2024 at 8:10:32 AM UTC-6, Paul B. Andersen wrote:
    If the SV-clock was not corrected by the factor (1-4.4646E-10),
    then a_f0 would overflow after less than 26 days, and
    the SY wouldn't work.

    I would somewhat disagree with your wording. I would prefer to
    state that if the relativistic correction hadn’t been applied,
    *then the entire GPS system would be broken from the start.*

    You are obviously right.
    But my statement above is still correct.

    I am responding to Lou, and even my simple statement
    is above his head.

    He is completely ignorant of the existence of the Control
    segment, and he snipped my statement, probably without reading:

    "Note that the correctional parameters are measured by
    the monitor stations, and uploaded to the SVs when needed,
    typically once a day. This is the "daily adjustment" you
    mentioned, _but the SV_clock is NOT adjusted_."


    People forget that GPS consists both of a Space Segment and a
    Control segment. The Space Segment is the constellation of 24
    in-service satellites plus spares.

    The control segment comprises the Master Control Station at
    Schriever AFB, the Alternate Master Control Station at Vandenberg
    AFB, and a worldwide network of tracking and monitoring stations
    that check on the health of the satellites, precisely monitor
    their positions to within centimeters, make sure that the
    satellite clocks are all sync’ed up, see to it that the
    satellite ephemerides are uploaded with the latest position
    corrections, provide the satellites with the latest ionospheric
    weather reports and so forth.

    To work effectively as a unified system. the Space Segment clocks
    and the Control Segment clocks had to all be running on the same
    time. Using 1970s technology, The Control Segment simply could not
    possibly be expected to manage the Space Segment effectively if
    the Space Segment clocks were running 38.6 microseconds per day
    fast. All of the clocks in the GPS system needed to be sync’ed
    within nanoseconds of each other whether they are in space or on
    the ground.

    Using modern technology, it is quite conceivable (although rather
    stupid) to have the Space Segment clocks running uncorrected, so
    that they continuously run further and further out of sync with
    ground clocks. All you would need would be to add a extra bits to
    af_0 so that you can run the satellites for several years without
    overflow. Indeed, that *may* have been how the Galileo designers
    originally intended that Galileo should work.

    Using 1970s technology, it was a matter of "don't be absurd."

    --
    Paul

    https://paulba.no/

    --- SoupGate-Win32 v1.05
    * Origin: fsxNet Usenet Gateway (21:1/5)
  • From Volney@21:1/5 to Lou on Thu Jan 4 23:22:25 2024
    On 1/4/2024 3:05 PM, Lou wrote:

    So....You insist above that only a quote that says “The base frequency is 10.23”
    will settle the question. How about this compromise:
    I’ll accept that the signal frequencies, are neither generated nor broadcast
    at 10.23Mhz, if you show me your specific quote from the specs doc that
    says “ The base frequency is 10.23 MHz."

    https://www.gps.gov/technical/icwg/IS-GPS-200N.pdf

    Since that spec explicitly states the common frequency source is 10.2299999954326 MHz, it won't ever state it is 10.23 MHz. Besides, it's
    up to you to prove your point that it's 10.23 MHz, why should I try to
    prove your point for you?

    --- SoupGate-Win32 v1.05
    * Origin: fsxNet Usenet Gateway (21:1/5)
  • From Volney@21:1/5 to Lou on Thu Jan 4 23:52:25 2024
    On 1/3/2024 3:28 PM, Lou wrote:

    Because although the GR correction formula does predict 5.27e-10.
    So does the much simpler classical formula GM/r ÷ c^2 , for
    any radius above and including r at earths surface.

    Aha! You managed to stumble across the Schwarzschild metric, an
    important part of general relativity discovered by Karl Schwarzschild in
    1915. It is an exact solution to the Einstein field equations that
    describes the gravitational field outside a spherical mass. Its formula
    is GM/rc^2, which is unitless. It is like a measure of the curvature of spacetime, and when very small the "weak gravity" approximation can be
    used. In the case of the earth its value is 5.27E-10, assuming your
    numbers are correct, and it is small. The Newtonian world works as if
    the Schwarzschild Metric is 0, meaning flat space.

    I guess this goes to show that even a blind squirrel finds an acorn once
    in a while.

    Oh as a part of GR, GM/rc^2 isn't "classical" unless you consider 100+
    year old general relativity as "classical" and only newer stuff like QFT
    as modern.

    Although quite why the arbitrary number 8.9875518e+16 used in both
    formula (calculated from c^2 and in an unspecified unit of m^2/s^2) gives the correct
    offset is a mystery to me.

    Look at Karl Schwarzschild's 1915 GR paper.

    Because 5.27e-10, regardless of which formula, has nothing
    to do with frequency or time.

    Correct, it is unitless.

    It’s just the force of gravity, potential if you insist,

    The curvature of spacetime in GR. GM/r is the gravitational potential.

    So dividing GM/ r

    Which is the gravitational potential (not the gravitational force, nor
    the acceleration)

    into any amount will give a different “offset”
    GM/r ÷ 1/2(8.9875518e+16 )= 1.05522118e-9 offset.

    And you found the general relativity link between the gravitational
    potential and the Schwarzschild Metric. Good job, blind squirrel, you
    found two acorns!

    --- SoupGate-Win32 v1.05
    * Origin: fsxNet Usenet Gateway (21:1/5)
  • From Volney@21:1/5 to Laurence Clark Crossen on Fri Jan 5 02:08:55 2024
    On 1/4/2024 10:55 PM, Laurence Clark Crossen wrote:

    In fact it was necessary to correct from the relativistic prediction to the empirically found frequency: "On Day 215,1977, the NTS-2 PRO-5 output signal
    was offset {Fig. 21) through the use of a frequency synthesizer {4) ." - "INITIAL RESULTS OF THE NAVSTAR GPS NTS-2 SATELLITE"

    Idjit.

    That was when the satellite was switched from the Newtonian setting (0
    clock offset) TO the relativistic prediction! Again, it was impossible
    to switch to an empirically found frequency because there was no such
    empirical data, this was the FIRST TIME a satellite was launched with Cs clocks!

    --- SoupGate-Win32 v1.05
    * Origin: fsxNet Usenet Gateway (21:1/5)
  • From Paul B. Andersen@21:1/5 to All on Fri Jan 5 13:53:17 2024
    Den 05.01.2024 04:36, skrev Laurence Clark Crossen:
    On Thursday, January 4, 2024 at 1:06:32 PM UTC-8, Paul B. Andersen wrote:
    Den 03.01.2024 15:45, skrev Lou:

    https://paulba.no/paper/Ashby.pdf

    page 16:
    "At the time of launch of the NTS-2 satellite (23 June 1977)
    . . . general relativity predicted +446.5 parts in 10¹²."

    The clock was predicted to run fast by +446.5 parts in 10¹²
    so the correction is -446.5 parts in 10¹².

    page 17:
    "When GPS satellites were first deployed, the specified
    factory frequency offset was slightly in error because
    the important contribution from earth’s centripetal potential
    had been inadvertently omitted at one stage of the evaluation.
    Although GPS managers were made aware of this error in the early
    1980s, eight years passed before system specifications were
    changed to reflect the correct calculation'."

    Which means that before the first GPS satellite was launched,
    the specified frequency offset was -446.5E-12 which
    is slightly different from the correct value +446.47E-12.

    Since the difference is less than the precision of the clocks,
    the error had no serious consequences, so the GPS did work
    before 1988 when the correct value was specified in the IS document.


    I hear you saying, as I take your meaning, that the value predicted by relativity was accurate enough not to require correction.
    However, two questions;
    1. Below, Lou asks wouldn't that be accumulative proving you wrong?
    2. I wonder what the synthesizer would have done if it didn't change from either the relativistic prediction to the empirically determined frequency OR from an alleged Newtonian switch.

    Read this first:

    On Wednesday 3 January 2024 at 14:10:32 UTC, Paul B. Andersen wrote:
    Here is an explanation of how the time reported from
    the SV is corrected:
    https://paulba.no/div/GPS_clock_correction.pdf

    t_SV is the time shown by the SV-clock.
    t_SV is never corrected while the SV is in service,
    and its error Δt_SV will typically be several μs.

    t_SV is sent from the SV to the receiver together with
    a few correctional parameters, the most important of
    which is the clock offset a_f0.
    The receiver can then calculate the correct system tine t,
    that is the time when the signal was sent.
    See reference above.

    Note that the correctional parameters are measured by
    the monitor stations, and uploaded to the SVs when needed,
    typically once a day. This is the "daily adjustment" you
    mentioned, _but the SV_clock is NOT adjusted_.

    Bbecause of the number of bits a_f0 is coded with,
    the clock offset must be less that 1 ms, this means
    that the SV clock error Δt_SV must be less then 1 ms.

    If the SV-clock was not corrected by the factor (1-4.4646E-10),
    then a_f0 would overflow after less than 26 days, and
    the SY wouldn't work.

    Bottom line:
    The _only_ reason for the GR-correction is to keep the SV clock
    correct within 1 ms.

    If you have read this, you will know that the SV-clock
    isn't corrected while the SV is in service.
    And you will know that the clock offset Δt_SV must be less then 1 ms,
    or the correction parameter a_f0 would overflow.

    If the rate of the SV-clock is exactly (1-4.4647E-10) compared
    to an SI_clock, then the clock offset Δt_SV will not change.
    (It will stay 0 if the clock initially is perfectly synced)

    If the rate of the SI-clock is not corrected at all, then
    the rate error is 4.4647E-10 and a_f0 will overflow after 26 days.

    If the clock is corrected by (1 - 4.465E-10) then the rate error
    is 3E-14 and a_f0 will overflow after 1056 years.

    So to answer your questions:

    1. Below, Lou asks wouldn't that be accumulative proving you wrong?

    Yes, with the correction (1 - 4.465E-10) the rate error would
    accumulate, so a_f0 will overflow after 1056 years.

    Which confirms my statement:
    "Since the difference is less than the precision of the clocks,
    the error had no serious consequences, so the GPS did work
    before 1988 when the correct value was specified in the IS document."

    2. I wonder what the synthesizer would have done if it didn't change from either the relativistic prediction to the empirically determined frequency OR from an alleged Newtonian switch.

    A very peculiar question.

    The synthesizer changed the frequency of the SI-clock
    by the factor (1 - 4.465E-10).

    And you ask what the synthesizer would have done if it
    hadn't done what it did! :-D

    --
    Paul

    https://paulba.no/

    --- SoupGate-Win32 v1.05
    * Origin: fsxNet Usenet Gateway (21:1/5)
  • From Paul B. Andersen@21:1/5 to All on Fri Jan 5 14:34:00 2024
    Den 05.01.2024 04:55, skrev Laurence Clark Crossen:
    On Thursday, January 4, 2024 at 1:06:32 PM UTC-8, Paul B. Andersen wrote:

    https://paulba.no/paper/Ashby.pdf

    page 16:
    "At the time of launch of the NTS-2 satellite (23 June 1977)
    . . . general relativity predicted +446.5 parts in 10¹²."

    The clock was predicted to run fast by +446.5 parts in 10¹²
    so the correction is -446.5 parts in 10¹².

    page 17:
    "When GPS satellites were first deployed, the specified
    factory frequency offset was slightly in error because
    the important contribution from earth’s centripetal potential
    had been inadvertently omitted at one stage of the evaluation.
    Although GPS managers were made aware of this error in the early
    1980s, eight years passed before system specifications were
    changed to reflect the correct calculation'."

    Which means that before the first GPS satellite was launched,
    the specified frequency offset was -446.5E-12 which
    is slightly different from the correct value -446.47E-12.

    Since the difference is less than the precision of the clocks,
    the error had no serious consequences, so the GPS did work
    before 1988 when the correct value was specified in the IS document.


    In fact it was necessary to correct from the relativistic prediction to the empirically found frequency: "On Day 215,1977, the NTS-2 PRO-5 output signal
    was offset {Fig. 21) through the use of a frequency synthesizer {4) ." - "INITIAL RESULTS OF THE NAVSTAR GPS NTS-2 SATELLITE"

    So the frequency synthesizer which prior to launch was built
    to lower the frequency of the SI-clock by the factor (1-446.5E-12)
    which was specified in the System Specification Document,
    was changed in flight to lower the frequency by the factor
    (1-442.5E-12).

    Do I have to explain why I find this hilarious?

    --
    Paul

    https://paulba.no/

    --- SoupGate-Win32 v1.05
    * Origin: fsxNet Usenet Gateway (21:1/5)
  • From Volney@21:1/5 to Lou on Fri Jan 5 11:08:28 2024
    On 1/4/2024 5:04 PM, Lou wrote:

    So assuming you are correct as you are a retired GPS engineer and know every detail of Every schematic and every table in the spec then maybe you could tell me
    why all the bother is made to code, chip, and generate a 10.23 Mhz signal
    on board the SV...to only then convert it down to a preset 10.22Mhz. Before its
    broadcast to the ground receiver?
    Why not just have the SV oscillator clock signal or whatever you call it generated at
    10.22Mhz instead and save the bother of having to add in a conversion unit from
    10.23 to 1022Mhz onboard the GPS sat?

    That is what's done. All the signals generated from the base clock are generated from the 10.2299999954326 MHz signal, and there is no
    "conversion unit"!

    --- SoupGate-Win32 v1.05
    * Origin: fsxNet Usenet Gateway (21:1/5)
  • From Volney@21:1/5 to Lou on Fri Jan 5 11:04:30 2024
    On 1/5/2024 10:16 AM, Lou wrote:

    From my understanding so far, the C-133 master clock generates 9192631770 beats a second.At which point these are then binned into larger 20 millisecond or 1.5 second parcels by a secondary 10.23Mhz oscillator. Which can only mean that a total of
    9192631770 beats per second are split into 10230000 pieces. Each piece containing:

    9192631770 ÷ 10230000= 898.595480938 beats of the total beats per second of the C-133 clock. At which point these are then binned into larger L1 L2 - 20 millisecond or 1.5 second parcels by a secondary process. In other words the ground reciever gets
    these parcels of data after being processed further as follows. Please note these finite 1.5 or 20 ms parcels STILL CONTAIN 10.23 chip rates!

    At this point the frequency plan quote says these clocks rate parcels are then offset /compressed to 10.22Mhz to account for relativity before being sent to earth.

    Boy do you have a screwed up idea how things work. There is no "offset/compression" stage. The Cs clock "ticks" using a divisor where
    there is 9192631774.1 ÷ 10230000 = 898.595481339 Cs cycles, and this corresponds to a frequency of 10.2299999954326 MHz. Then the secondary processes take place except there is no "offset /compressed" stage to 10.2299999954326 MHz since the signal is ALREADY at the correct
    frequency! Remember everything is based off the 10.2299999954326 MHz
    signal because the downward blueshift affects ALL frequencies equally,
    so on the ground the first signal is 10.23 MHz, the 20 mS and 1.5 second
    "bins" are 20 mS & 1.5 seconds long respectively etc.

    But if the chipped 10.23 SV clock is “offset” to 10.22Mhz in the SV before broadcast to ground then the offset that is modulated onto the carrier wave STILL HAS THE 10.23Mhz chip rate data !!

    No, you made an incorrect assumption. The master frequency was decreased
    so are all frequencies based off it. The reverse happens during the
    blueshift so on earth all frequencies are correct. There is no
    "modulation" since the frequencies are correct at the satellite already!

    So all the ground receiver really gets is a carrier wave with the 10.23 Mhz chip rate data!

    Because the 10.2299999954326 MHz frequency aboard the SV was blueshifted
    to 10.23 MHz chip rate.

    And
    No time dilation occurs because the SV clock is at 10.23Mhz.

    No the SV base signal is 10.2299999954326 MHz. The entire point is that
    it gets blueshifted to 10.23 MHz on the ground.

    And the ground recieves a 10.23 chip rate modulated onto the carrier wave.

    Exactly.

    Regardless of whether the data is compressed onto a 10.22 signal on the SV.

    No "compression", the SV signal is 10.2299999954326 MHz.

    --- SoupGate-Win32 v1.05
    * Origin: fsxNet Usenet Gateway (21:1/5)
  • From Volney@21:1/5 to Laurence Clark Crossen on Fri Jan 5 10:32:00 2024
    On 1/4/2024 11:51 PM, Laurence Clark Crossen wrote:
    On Thursday, January 4, 2024 at 2:20:28 AM UTC-8, Lou wrote:
    On Thursday 4 January 2024 at 00:45:36 UTC, Volney wrote:
    On 1/3/2024 10:42 AM, Lou wrote:

    Figure 3-1. , Figure 3-6. Figure 3-1, Figure 3-11, Figure 3-12. ..all Clearly say 10.23

    Nominal again.

    For you to be correct, you have to find ONE statement stating "The base
    frequency is 10.23 MHz." Since there already is one statement stating
    "The base frequency is 10.2299999954326 MHz", it cannot happen since
    otherwise the spec would contradict itself if it did.

    So show us the statement "The base frequency is 10.23 MHz." in the spec.

    Here you go Volney boy. Seeing as you havent learnt how to google
    I did it for you
    It says clearly at the top that “ Three signals are transmitted at the
    moment by GPS “ And then goes on to specify these 3 variations.
    One of the three copied below. Additionally it refers to these as SV signals.
    SV means GPS satelitte. Ask Paul. He just posted a couple days ago
    a comment in which he says that SV means....Sat clock. Savvy?
    I don’t see how you can assume this refers to anything but the transmission
    signal or clock frequency of the satelitte.

    https://gssc.esa.int/navipedia/index.php/GPS_Signal_Plan#:~:text=The%20PRN%20C%2FA%20Code,1023%20chip%20long%20linear%20pattern.



    “ GPS L2 Band
    GPS is transmitting in the L2 band (1227.60 MHz) a modernized civil signal known as L2C designed specifically to meet commercial needs as it enables the development of dual-frequency solutions; together with the P(Y) Code and the M-Code. The P(Y) Code
    and M-Code were already described shortly in the previous chapter and the properties and parameters are thus similar to those in the L1 band. In addition, for Block IIR-M, IIF, and subsequent blocks of SVs, two additional PRN ranging codes are
    transmitted. They are the L2 Civil Moderate (L2 CM) code and the L2 Civil Long (L2 CL) code. These two signals are time multiplexed so that the resulting chipping rate is double as high as that of each individual signal. We further describe them in the
    next lines more in detail:

    L2 CM Code is transmitted in the IIR-M, IIF, III and subsequent blocks. The PRN L2 CM Code for SV number i is a ranging code, which is 20 milliseconds in length at a chipping rate of 511.5 Kbps. The epochs of the L2 CM Code are synchronized with the
    X1 epochs of the P-code. The CM sequence is a linear pattern which is short cycled every count period of ***10,230*** chips by resetting with a particular initial state. “


    Does that say 10.23 or 10229999.9954326?

    Again, neither. Try again.

    I think that it just means the frequency would be 10.2299 on Earth but is 10.23 in space. Wozniak says, "Can be preset. Afterf getting on the orbit it is 10.23, as measured.
    Good bye, The Shit. "

    Are you actually quoting that insane fool who has half of his
    definitions backwards from what specifications or theory actually state?
    Such as his claim of switching the NTS-2 clock from 10.23 MHz to 10229999.9954326 MHz is switching TO Newtonian time? Hahaha!

    --- SoupGate-Win32 v1.05
    * Origin: fsxNet Usenet Gateway (21:1/5)
  • From Volney@21:1/5 to Lou on Fri Jan 5 18:13:24 2024
    On 1/5/2024 3:38 PM, Lou wrote:
    On Friday 5 January 2024 at 16:04:33 UTC, Volney wrote:
    On 1/5/2024 10:16 AM, Lou wrote:

    From my understanding so far, the C-133 master clock generates 9192631770 beats a second.At which point these are then binned into larger 20 millisecond or 1.5 second parcels by a secondary 10.23Mhz oscillator. Which can only mean that a total of
    9192631770 beats per second are split into 10230000 pieces. Each piece containing:

    9192631770 ÷ 10230000= 898.595480938 beats of the total beats per second of the C-133 clock. At which point these are then binned into larger L1 L2 - 20 millisecond or 1.5 second parcels by a secondary process. In other words the ground reciever
    gets these parcels of data after being processed further as follows. Please note these finite 1.5 or 20 ms parcels STILL CONTAIN 10.23 chip rates!

    At this point the frequency plan quote says these clocks rate parcels are then offset /compressed to 10.22Mhz to account for relativity before being sent to earth.
    Boy do you have a screwed up idea how things work.

    I need to find out how the SV clocks and signal work. Best way to learn
    is try to summarise what I know and ask the experts to clarify or correct. And it worked. I wouldnt have got your info without asking. And thanks for that info incidentally.

    There is no
    "offset/compression" stage. The Cs clock "ticks" using a divisor where
    there is 9192631774.1 ÷ 10230000 = 898.595481339 Cs cycles,

    I don’t quite understand this bit.
    You seem to suggest here that the caesium atoms and a caesium clocks frequency is at 9192631774.1 That’s not what I read.
    *Every source* I read puts the caesium atoms frequency and a caesium clocks frequency at 9102631770.

    Yes it's true that the second, by definition, is 9192631770 times the
    cycle of one Cs transition wave. The 9192631774.1 divisor produces a
    time pulse which is a tiny bit longer than a second.

    More specifically, the master frequency generator uses a divisor equal
    to 10,230,000/9192631774.1. Do the math and this actually generates a
    frequency of 10.2299999954326 MHz. All the other signals intended for
    earth are based off this slightly low frequency.

    Heres wiki:
    “By definition, radiation produced by the transition between the two hyperfine ground states of caesium (in the absence of external influences such as the Earth's magnetic field) has a frequency, ΔνCs, of exactly 9192631770 Hz.”

    Yes, and by definition, a 1 pulse per 9192631774.1 Cs transitions is NOT
    one pulse per second. Very close but slightly too long,

    So how do they get the atoms frequency to change from 9192631770 to 9102631774.1

    They don't. The Cs, by its nature, still generates its natural frequency
    of 9.192631770 GHz. Locally to the satellite, of course.

    Anyways, if I understand you correctly the chip rate is never at
    10.23. Always at 10.22.

    You shouldn't write it that way because the actual frequency is not
    close to 10.22 MHz.

    But why then would the spec always refer to it as being 10.23 if it never was?

    It's 10.23 MHz on the ground because of the blueshift that the offset compensates for. Or, they mean nominal because nobody wants to write 10.2299999954326 MHz all the time so they just write 10.23 nominal
    instead. Even you don't want to type all those digits, although writing
    it as 10.22 MHz isn't right.

    If the 10.22 chip rate is binned at 20ms or 1.5 s in the SV before transmission
    That doesn’t make sense either. Because if there is a speeding up of the frequency
    during transmission those bins would have to change length too.

    No, nobody speeds anything up. Those 10ms and 1.5s signals are what's
    received on the ground, they get blueshifted just like the master
    signal. On the satellite they are derived from the master signal and
    local to the satellite, they are slightly too long.

    Remember this: All the signals described are intended to be used on the
    ground and the frequencies involved included, after the downlink blueshift.

    Seeing
    as the SV only bins a specific amount of 10.22 chips per bin.
    How does that work. I would have thought you would end up with bins
    being shorter in time length on arrival to accomadate a higher
    frequency offset and thus over even just minutes
    one would get a problem where the sat sends x amount of bins in
    a set time length And the ground gets x amount of bins in
    a shorter amount of time. If this were the case the receiver
    would run out of bins to receive because each bins time length
    was shorter at receiver then it was at SV before it was transmitted.

    Everything gets sped up so everything gets generated slower.
    Consider playing a 33 rpm record at 45 rpm. Not just the notes are at a
    higher frequency, but the tempo of the music, the beats per minute,
    length of a musical measure etc. are all sped up as well.

    --- SoupGate-Win32 v1.05
    * Origin: fsxNet Usenet Gateway (21:1/5)
  • From Volney@21:1/5 to Laurence Clark Crossen on Fri Jan 5 22:19:21 2024
    On 1/4/2024 4:31 PM, Laurence Clark Crossen wrote:
    On Wednesday, January 3, 2024 at 8:27:22 PM UTC-8, Volney wrote:
    On 1/3/2024 10:27 PM, Laurence Clark Crossen wrote:
    On Wednesday, January 3, 2024 at 4:31:47 PM UTC-8, Volney wrote:
    On 1/3/2024 1:59 PM, Laurence Clark Crossen wrote:

    The clock adjustment for gravity is not about relativity. Only gravity. It was obtained empirically.
    How the hell could it have been set empirically since the time offset
    was preset BEFORE the launch? NTS-2 was the first satellite EVER to be >>>> launched with Cs clocks so there was no other source of empirical data, >>>> either.

    And your statement "gravity is not about relativity" is contradictory as >>>> gravity is an effect of general relativity.

    The time offset can not have been preset before launch because they only found out how much to adjust it after launch.
    And why do you claim that silliness? And do you have any evidence of it?
    I didn't think so.

    Also first you said it was preset empirically (with no data to set it
    to) and now you claim it was not preset at all?
    The two-switch story is a fairy tale.
    Again why do you claim that? Evidence? Again, I didn't think so. And why
    is the switch right in the paper

    Gravity is not an effect of relativity because a theory does not cause anything. It should explain nature rationally.
    And general relativity does just that.

    Relativity is not about anything since it is not a theory.
    Why do you say that? It meets all the requirements of scientific theories. >>> Relativity only pretends to explain the cause of gravity.
    Why "pretends"? It gives a logical, scientific description and explanation. >>> Gravity is not even electromagnetism.
    Of course not. Just like GR says it has nothing to do with electromagnetism. >>>
    I really can't help you with your confusion.

    It is you who is confused, thinking things like GR says gravity is
    electromagnetism, strange beliefs about NTS-2, don't understand what a
    theory is in science, etc.

    1. Anderson above (and this: "INITIAL RESULTS OF THE NAVSTAR GPS
    NTS-2 SATELLITE) said the prediction was not more than 1% off.
    2. Therefore, it was not exact.

    I do not know the experimental errors that may have been involved, such
    as determining the actual orbital elevation eccentricity and so forth.

    3. the clock rate (frequency) must be exact for the GPS to function.

    Specifically, small errors will show errors in location. Errors in
    frequency will show a stationary GPS user as apparently moving off in a
    certain direction. Specifically, not using the GR predicted offset will
    show a stationary GPS user moving about 10 km/day, each and every day.

    4. Relativity did not predict The exact amount (this is what you haven't gotten).

    There was a small error, which would cause a small wandering off. It was
    usable since it would remain in range of the GPS's global error offset
    for something like 1000 years, while not using the GR correction would
    exceed the range of the global error offset in something like 12 days.

    5. Therefore, the synthesizer must have adjusted it from the relativity prediction to the exact amount.

    Yes they do this because of the sun and moon tugging on satellites'
    orbits, solar storms, the ionosphere etc. This is the offset that would
    run out in about 12 days if GR was ignored.

    6. Above are the quotes from Essen saying it is not a scientific theory. You are welcome to read his papers criticizing relativity.

    Those arguments don't disqualify anything as a theory. Actually GR is
    the theory, GPS is an application of the theory.

    7. According to Essen and thousands of other scientists you have not studied, it doesn't give a logical explanation but a self-contradictory one.

    I saw nothing self-contradictory in any GPS specs. Point one out, if you
    can.

    --- SoupGate-Win32 v1.05
    * Origin: fsxNet Usenet Gateway (21:1/5)
  • From Paul B. Andersen@21:1/5 to All on Sat Jan 6 15:27:41 2024
    Den 04.01.2024 23:04, skrev Lou:

    Yes Paul. I’ve read your rants already. And read the spec which says there is a
    carrier etc offset to 10.22999 . A supposed offset which incidentally can also
    be explained just as well by a classical non relativistic model using GM/r ÷ f

    There is no "classical non relativistic model" which can "explain"
    the rate of clocks in a gravitational field.

    An approximation of the Schwarzschild metric:

    The rate of Schwarzschild coordinate time t is the same
    as the rate of a clock at infinity.

    The rate of a clock at distance r and speed v in the ECI-frame
    relative to Schwarzschild coordinate time is:

    dτ/dt = (1 - GM/r⋅c² - v²/2c²) (1)

    GM/r is the Newtonian gravitational potential, but that doesn't
    make the Schwarzschild metric "non relativistic".

    If the clock is in circular orbit then v² = GM/r and (1) can be written:

    dτ/dt = (1 - 1.5⋅GM/r⋅c²) (2)

    This is the rate of a clock in circular orbit relative to
    Schwarzschild coordinate time, but we are more interested in
    the rate of the clock relative to Universal Coordinated time (UTC).

    We consult Ashby:
    https://paulba.no/paper/Ashby.pdf
    see equation (18) page 11.

    dt_utc/dt = (1 - δutc) where δutc = 6.96927E-10

    dτ/dt_utc = (1 - 1.5⋅GM/r⋅c²)/(1 - δutc) ≃ (1 - 1.5⋅GM/r⋅c² + δutc) (3)

    Δf/f₀ = (dτ/dt_utc - 1) = - (1.5⋅GM/r⋅c² - δutc) (4)

    For the GPS the orbital period p is specified to be half a sidereal day:
    p = 43082.04525 s
    GM = 3.986004418E14 m³/s²
    c = 299792458 m/s
    r = GM⋅p²/4π² = 26561763 m

    Equation (4) yields; Δf/f₀ = 4.46471409E-10

    This means that the clock will run fast relative
    to UTC, so to stay in sync with UTC it must be
    corrected by Δf/f₀ = - 4.46471409E-10

    In the GPS specification the correction is set to
    Δf/f₀ = - 4.4647E-10


    Nonetheless the rest of your spec clearly says the nominal SV signal, frequency, clock
    chip rates etc on the sat are set at 10.23. You can’t deny that.
    So assuming you are correct as you are a retired GPS engineer and know every detail of Every schematic and every table in the spec then maybe you could tell me
    why all the bother is made to code, chip, and generate a 10.23 Mhz signal
    on board the SV...to only then convert it down to a preset 10.22Mhz. Before its
    broadcast to the ground receiver?

    Why not just have the SV oscillator clock signal or whatever you call it generated at
    10.22Mhz instead and save the bother of having to add in a conversion unit from
    10.23 to 1022Mhz onboard the GPS sat?

    Of course that's what is done.

    The frequency of the common frequency source is simply
    f₀ = 10.2299999954326 MHz, as measured by a local SI-clock.
    It is not 'converted' from anything else.

    The frequencies of the carriers are derived from f₀:
    L1 = 154⋅f₀ = 1575.4200007033778 Mhz
    L2 = 120⋅f₀ = 1227.6000005480864 Mhz
    as measured by a local SI-clock.

    Measured by local UTC-clocks the frequencies are:
    f₀ = 10.23 MHz,
    L1 = 154⋅f₀ = 1575.42 MHz and L2 = 120⋅f₀ = 1227.6 MHz.

    Chew on that! :-D

    SI-clock = a clock with time unit seconds as defined by SI.
    UTC-clock = a clock showing UTC

    --
    Paul

    https://paulba.no/

    --- SoupGate-Win32 v1.05
    * Origin: fsxNet Usenet Gateway (21:1/5)
  • From Volney@21:1/5 to Volney on Sat Jan 6 10:55:37 2024
    On 1/5/2024 10:32 AM, Volney wrote:
    On 1/4/2024 11:51 PM, Laurence Clark Crossen wrote:
    On Thursday, January 4, 2024 at 2:20:28 AM UTC-8, Lou wrote:

    Does that say 10.23 or 10229999.9954326?

    Again, neither. Try again.

    I think that it just means the frequency would be 10.2299 on Earth but
    is 10.23 in space. Wozniak says, "Can be preset. Afterf getting on the
    orbit it is 10.23, as measured.
    Good bye, The Shit. "

    Are you actually quoting that insane fool who has half of his
    definitions backwards from what specifications or theory actually state?
    Such as his claim of switching the NTS-2 clock from 10.23 MHz to 10229999.9954326 MHz is switching TO Newtonian time? Hahaha!

    That (clock measuring 10229999.9954326 MHz on earth but 10.23 MHz in
    space) is actually a fine example of Wozniak getting things backwards.
    The satellite in space (as measured by a comoving Cs clock) generates a
    signal at 10229999.9954326 MHz in space as set up before launch, but is measured at 10.23 MHz on the ground. Not the other way around.

    --- SoupGate-Win32 v1.05
    * Origin: fsxNet Usenet Gateway (21:1/5)
  • From Paul B. Andersen@21:1/5 to All on Sun Jan 7 14:01:35 2024
    Den 07.01.2024 05:27, skrev Laurence Clark Crossen:
    On Saturday, January 6, 2024 at 6:24:23 AM UTC-8, Paul B. Andersen wrote:
    Den 04.01.2024 23:04, skrev Lou:

    Yes Paul. I’ve read your rants already. And read the spec which says there is a
    carrier etc offset to 10.22999 . A supposed offset which incidentally can also
    be explained just as well by a classical non relativistic model using GM/r ÷ f

    There is no "classical non relativistic model" which can "explain"
    the rate of clocks in a gravitational field.

    Then, there was no Newtonian prediction for the GPS clock. How does gravity affect the rate of an atomic clock? If gravity affects it, then Newtonian would have an exlanation. How does the relativistic explanation compare?

    Read "the relativistic explanation" below:


    An approximation of the Schwarzschild metric:

    The rate of Schwarzschild coordinate time t is the same
    as the rate of a clock at infinity.

    The rate of a clock at distance r and speed v in the ECI-frame
    relative to Schwarzschild coordinate time is:

    dτ/dt = (1 - GM/r⋅c² - v²/2c²) (1)

    GM/r is the Newtonian gravitational potential, but that doesn't
    make the Schwarzschild metric "non relativistic".

    If the clock is in circular orbit then v² = GM/r and (1) can be written:

    dτ/dt = (1 - 1.5⋅GM/r⋅c²) (2)

    This is the rate of a clock in circular orbit relative to
    Schwarzschild coordinate time, but we are more interested in
    the rate of the clock relative to Universal Coordinated time (UTC).

    We consult Ashby:
    https://paulba.no/paper/Ashby.pdf
    see equation (18) page 11.

    dt_utc/dt = (1 - δutc) where δutc = 6.96927E-10

    dτ/dt_utc = (1 - 1.5⋅GM/r⋅c²)/(1 - δutc) ≃ (1 - 1.5⋅GM/r⋅c² + δutc) (3)

    Δf/f₀ = (dτ/dt_utc - 1) = - (1.5⋅GM/r⋅c² - δutc) (4)

    For the GPS the orbital period p is specified to be half a sidereal day:
    p = 43082.04525 s
    GM = 3.986004418E14 m³/s²
    c = 299792458 m/s
    r = GM⋅p²/4π² = 26561763 m

    Equation (4) yields; Δf/f₀ = 4.46471409E-10

    This means that the clock will run fast relative
    to UTC, so to stay in sync with UTC it must be
    corrected by Δf/f₀ = - 4.46471409E-10

    In the GPS specification the correction is set to
    Δf/f₀ = - 4.4647E-10

    Den 04.01.2024 23:04, skrev Lou:
    Why not just have the SV oscillator clock signal or whatever you call it generated at
    10.22Mhz instead and save the bother of having to add in a conversion unit from
    10.23 to 1022Mhz onboard the GPS sat?

    Of course that's what is done.

    The frequency of the common frequency source is simply
    f₀ = 10.2299999954326 MHz, as measured by a local SI-clock.
    It is not 'converted' from anything else.

    The frequencies of the carriers are derived from f₀:
    L1 = 154⋅f₀ = 1575.4200007033778 Mhz
    L2 = 120⋅f₀ = 1227.6000005480864 Mhz
    as measured by a local SI-clock.

    Measured by local UTC-clocks the frequencies are:
    f₀ = 10.23 MHz,
    L1 = 154⋅f₀ = 1575.42 MHz and L2 = 120⋅f₀ = 1227.6 MHz.

    Chew on that! :-D

    SI-clock = a clock with time unit seconds as defined by SI.
    UTC-clock = a clock showing UTC


    --
    Paul

    https://paulba.no/

    --- SoupGate-Win32 v1.05
    * Origin: fsxNet Usenet Gateway (21:1/5)
  • From Paul B. Andersen@21:1/5 to All on Sun Jan 7 15:44:29 2024
    Den 06.01.2024 04:55, skrev Laurence Clark Crossen:
    On Friday, January 5, 2024 at 4:50:01 AM UTC-8, Paul B. Andersen wrote:
    Den 05.01.2024 04:36, skrev Laurence Clark Crossen >>> I wonder what the synthesizer would have done if it didn't change
    from either the relativistic prediction to the empirically determined
    frequency OR from an alleged Newtonian switch.

    A very peculiar question.

    The synthesizer changed the frequency of the SI-clock
    by the factor (1 - 4.465E-10).

    And you ask what the synthesizer would have done if it
    hadn't done what it did! :-D


    It seems you are saying it changed from the Newtonian mode to the relativistic one? Then, what was the meaning of the Newtonian mode? Was it half relativistic?

    There is no "Newtonian mode".

    There is only a clock which is counting seconds
    according to the SI definition. ( an "SI-clock")

    And a clock which is modified by a frequency synthesizer
    to run slow by the factor (1 - 4.465E-10) compared to an "SI-clock".

    > Newtonian would not be that used on Earth because, according to
    Newton gravity would affect light and the cesium clock uses six laser
    beams in the cesium gas.

    Are you claiming that according to Newton atomic clocks on Earth
    don't work?

    I don't think Newton has much to say about atomic clocks. :-D

    --
    Paul

    https://paulba.no/

    --- SoupGate-Win32 v1.05
    * Origin: fsxNet Usenet Gateway (21:1/5)
  • From Volney@21:1/5 to Laurence Clark Crossen on Sun Jan 7 16:42:42 2024
    On 1/5/2024 10:55 PM, Laurence Clark Crossen wrote:
    On Friday, January 5, 2024 at 4:50:01 AM UTC-8, Paul B. Andersen wrote:

    If you have read this, you will know that the SV-clock
    isn't corrected while the SV is in service.
    And you will know that the clock offset Δt_SV must be less then 1 ms,
    or the correction parameter a_f0 would overflow.

    If the rate of the SV-clock is exactly (1-4.4647E-10) compared
    to an SI_clock, then the clock offset Δt_SV will not change.
    (It will stay 0 if the clock initially is perfectly synced)

    If the rate of the SI-clock is not corrected at all, then
    the rate error is 4.4647E-10 and a_f0 will overflow after 26 days.

    If the clock is corrected by (1 - 4.465E-10) then the rate error
    is 3E-14 and a_f0 will overflow after 1056 years.

    So to answer your questions:
    1. Below, Lou asks wouldn't that be accumulative proving you wrong?
    Yes, with the correction (1 - 4.465E-10) the rate error would
    accumulate, so a_f0 will overflow after 1056 years.

    Which confirms my statement:
    "Since the difference is less than the precision of the clocks,
    the error had no serious consequences, so the GPS did work
    before 1988 when the correct value was specified in the IS document."
    2. I wonder what the synthesizer would have done if it didn't change from either the relativistic prediction to the empirically determined frequency OR from an alleged Newtonian switch.
    A very peculiar question.

    The synthesizer changed the frequency of the SI-clock
    by the factor (1 - 4.465E-10).

    And you ask what the synthesizer would have done if it
    hadn't done what it did! :-D

    --
    Paul

    https://paulba.no/

    Thank you for your patient explanation. It seems you are saying it changed from the Newtonian mode to the relativistic one? Then, what was the meaning of the Newtonian mode? Was it half relativistic? Newtonian would not be that used on Earth because,
    according to Newton gravity would affect light and the cesium clock uses six laser beams in the cesium gas.

    You are very confused (again). "Newtonian mode" simply means Newton's
    concept of time, which is the same everywhere in the universe. There is
    no time dilation in Newton's view of the universe nor could there be,
    clocks everywhere tick in synch. That includes a GPS satellite, it would transmit at 10.23 MHz to be received at 10.23 MHz, since the transmitter frequency is just another clock. So "Newtonian mode" for the first NTS-2 satellite simply means not to enable any time offset.

    Also "half relativistic" applies to the gravitational deflection of
    light ONLY. The "Newtonian mode" for clock offset obviously cannot be
    "half relativistic" because it is zero and "relativistic" is nonzero.
    And time dilation is not light bending by gravity, either.

    --- SoupGate-Win32 v1.05
    * Origin: fsxNet Usenet Gateway (21:1/5)
  • From Volney@21:1/5 to Laurence Clark Crossen on Sun Jan 7 21:58:39 2024
    On 1/7/2024 4:56 PM, Laurence Clark Crossen wrote:

    Thank you for explaining what Newtonian mode is. Not having heard that, I thought the reasonable supposition was that an honest comparison would be made between the Newtonian gravitational effect on atomic clocks and the relativistic. Most of the
    effect on the atomic clocks is gravitational.

    The orbiting atomic clocks are in freefall, so there is NO gravitational
    force on them.

    In fact, all of it is since there is no time dilation.

    Assertion without evidence. In contrast, many experiments, most famously
    the cosmic muon effect, show you to be completely wrong.

    That is just read into the empirical data.

    And again, how could there be any empirical data if NTS-2 was the first satellite to fly with Cs clocks, and the time dilation offset was
    programmed into it before launch? Are you really stooopid or something?

    --- SoupGate-Win32 v1.05
    * Origin: fsxNet Usenet Gateway (21:1/5)
  • From Paul B. Andersen@21:1/5 to All on Mon Jan 8 13:50:20 2024
    Den 07.01.2024 21:08, skrev Laurence Clark Crossen:
    The Schwartzschild metric is about the fiction of curved space, which is the reification fallacy; therefore, it explains nothing. The Schwarzschild Metric tells us the amount of time dilation. Time dilation is a fiction. It is not physics.

    All these experiments confirm the predictions of
    the Schwartzschild metric:

    https://paulba.no/paper/Pound&Rebka.pdf
    https://paulba.no/paper/Hafele.pdf
    https://paulba.no/paper/Alley.pdf
    (see experiment on pages 708-716 https://paulba.no/paper/Initial_results_of_GPS_satellite_1977.pdf https://paulba.no/paper/Vessot.pdf

    And the satellite navigation systems
    GPS, GLONASS, Galileo and BeiDou are continuously confirming
    of the predictions of the Schwartzschild metric.

    The hallmark of a crank is that he ignores experimental evidence.

    --
    Paul

    https://paulba.no/

    --- SoupGate-Win32 v1.05
    * Origin: fsxNet Usenet Gateway (21:1/5)
  • From Paul B. Andersen@21:1/5 to All on Mon Jan 8 14:42:21 2024
    Den 07.01.2024 21:09, skrev Laurence Clark Crossen:

    Any effect of gravity on atomic clocks would be Newtonian. Relativity explains nothing about it.

    It is a proven fact that a clock in circular orbit with radius
    26561763 metres will advance 43082.045269235 seconds per orbit
    while a clock on the geoid will advance 43082.045250000 seconds.
    The difference is 19.235 microseconds

    Please show us how Newton explains this gravitational effect.

    (If you wonder, the clock is in GPS orbit, and the effect is
    confirmed by the fact that the GPS works.)

    --
    Paul

    https://paulba.no/

    --- SoupGate-Win32 v1.05
    * Origin: fsxNet Usenet Gateway (21:1/5)
  • From Volney@21:1/5 to Laurence Clark Crossen on Mon Jan 8 17:51:48 2024
    On 1/8/2024 4:57 PM, Laurence Clark Crossen wrote:
    On Sunday, January 7, 2024 at 6:58:42 PM UTC-8, Volney wrote:
    On 1/7/2024 4:56 PM, Laurence Clark Crossen wrote:

    That is just read into the empirical data.

    And again, how could there be any empirical data if NTS-2 was the first
    satellite to fly with Cs clocks, and the time dilation offset was
    programmed into it before launch? Are you really stooopid or something?

    [bla bla not addressing the unavailability of empirical data]

    You could answer my question rather than spewing nonsense.

    You could listen better instead of insulting people.

    If you don't want to be called stooopid, don't act stooopid. There is no possible empirical data available at the time of the NTS-2 launch. Don't
    you agree that it's rather stooopid to claim empirical data was used if
    it was nonexistent at the time?

    It refers to the interpretation of part of the rate change as having been caused by SR that is not caused by SR.

    Huh? Are you talking about the part that was caused by SR (the motion of
    the satellites causing time dilation)?

    All the difference is caused by gravity. What do you think causes the faster clock rate in orbit if it isn't lesser gravity?

    It's not caused by gravitational force that could affect a clock. It is
    caused by the Schwarzschild metric, which is proportional to the
    gravitational potential in the weak field. The motion of the satellites
    reduce the difference from 45µS/day to 38µS/day.

    --- SoupGate-Win32 v1.05
    * Origin: fsxNet Usenet Gateway (21:1/5)
  • From Volney@21:1/5 to Laurence Clark Crossen on Mon Jan 8 21:32:52 2024
    On 1/8/2024 6:30 PM, Laurence Clark Crossen wrote:
    On Monday, January 8, 2024 at 2:51:52 PM UTC-8, Volney wrote:
    On 1/8/2024 4:57 PM, Laurence Clark Crossen wrote:

    [nothing]

    I note how you continue not to explain where the empirical value for the
    clock offset of the NTS-2 satellite came from.

    All the difference is caused by gravity. What do you think causes the faster clock rate in orbit if it isn't lesser gravity?

    It's not caused by gravitational force that could affect a clock. It is
    caused by the Schwarzschild metric, which is proportional to the
    gravitational potential in the weak field. The motion of the satellites
    reduce the difference from 45µS/day to 38µS/day.

    Volney, you never listened even when I said I was taking about something else! I'm talking about the fact that the total difference in the clock rate in orbit is partly attributed to SR and part to GR. The point was that there is no SR time dilation so
    the whole difference is due to gravity.

    Assertions are not facts.

    SR time dilation is well known and well understood. Are you near sea
    level? If so why are all those cosmic muons striking you reaching you at
    all? Why not tell Tom R. that the kilometer long pion beam he often
    mentions here can't exist, despite the fact that it does or did.

    38 is gravity and there is no 45 except in the imagination of relativists.

    Nope. GR and SR predicted the value of the NTS-2 satellite offset which
    was shown to be correct. Gravity Probe A showed the GR value itself.

    If it is not caused by gravity it has nothing to do with GR or the S-metric. The metric is an attempt to measure the gravitational effect on the clock.

    Except it has two components, one is the Schwarzschild metric from the
    orbital height and the other the speed of the satellite in its orbit.
    And again, the Schwarzschild metric is based on the gravitational
    potential, not on the gravitational force.

    --- SoupGate-Win32 v1.05
    * Origin: fsxNet Usenet Gateway (21:1/5)
  • From Volney@21:1/5 to Laurence Clark Crossen on Mon Jan 8 23:19:04 2024
    On 1/8/2024 10:56 PM, Laurence Clark Crossen wrote:
    On Monday, January 8, 2024 at 6:32:56 PM UTC-8, Volney wrote:
    On 1/8/2024 6:30 PM, Laurence Clark Crossen wrote:

    If it is not caused by gravity it has nothing to do with GR or the S-metric. The metric is an attempt to measure the gravitational effect on the clock.

    Except it has two components, one is the Schwarzschild metric from the
    orbital height and the other the speed of the satellite in its orbit.
    And again, the Schwarzschild metric is based on the gravitational
    potential, not on the gravitational force.

    Then, you must be able to explain what the physical cause is of the atomic clock moving fast.
    No clock runs fast. Each clock ticks at its normal rate of 1 second per
    second.

    What you see is the blueshift of the signal from the satellite to the
    ground. That makes the ground observer think the satellite clock seems
    to run fast.

    What effect does the gravitational potential have on the gas or laser beams in the clock?

    None. The only effect is on signals between different potentials, either redshift or blueshift.

    --- SoupGate-Win32 v1.05
    * Origin: fsxNet Usenet Gateway (21:1/5)
  • From Paul B. Andersen@21:1/5 to All on Tue Jan 9 13:42:32 2024
    Den 08.01.2024 22:58, skrev Laurence Clark Crossen:
    On Monday, January 8, 2024 at 4:46:58 AM UTC-8, Paul B. Andersen wrote:

    All these experiments confirm the predictions of
    the Schwartzschild metric:

    https://paulba.no/paper/Pound&Rebka.pdf
    https://paulba.no/paper/Hafele.pdf
    https://paulba.no/paper/Alley.pdf
    (see experiment on pages 708-716
    https://paulba.no/paper/Initial_results_of_GPS_satellite_1977.pdf
    https://paulba.no/paper/Vessot.pdf

    And the satellite navigation systems
    GPS, GLONASS, Galileo and BeiDou are continuously confirming
    of the predictions of the Schwartzschild metric.

    The hallmark of a crank is that he ignores experimental evidence.


    An illogical theory cannot be proven by experiment because it does not predict. When the necromancer predicts the winner of a horse race we do not think it was thanks to necromancy.

    Your opinion of the consistency of GR is simply wrong.
    It is a fact that GR is mathematically consistent.

    No theory can be proven, but a theory must be falsifiable.
    A theory is tested by calculating what the theory predicts will
    be measured in an experiment, and then comparing the predicted
    values with the measured values when the experiment is performed.

    If the predictions are in accordance with the measurements
    within the precision of the measurements, then the theory
    is confirmed (NOT proven).
    If the predictions are not in accordance with the measurements,
    then the theory is falsified.

    It is a fact that predictions of GR for
    the experiments above are in accordance with
    the measurements.

    So it is PROVEN that GR gives the correct predictions
    for those particular experiments, and for the satellite
    navigation systems GPS, GLONASS, Galileo and BeiDou.

    Denying facts is irrational behaviour!

    --
    Paul

    https://paulba.no/

    --- SoupGate-Win32 v1.05
    * Origin: fsxNet Usenet Gateway (21:1/5)
  • From Paul B. Andersen@21:1/5 to All on Tue Jan 9 14:22:03 2024
    Den 08.01.2024 22:59, skrev Laurence Clark Crossen:
    On Monday, January 8, 2024 at 5:38:59 AM UTC-8, Paul B. Andersen wrote:
    Den 07.01.2024 21:09, skrev Laurence Clark Crossen:

    Any effect of gravity on atomic clocks would be Newtonian. Relativity explains nothing about it.

    It is a proven fact that a clock in circular orbit with radius
    26561763 metres will advance 43082.045269235 seconds per orbit
    while a clock on the geoid will advance 43082.045250000 seconds.
    The difference is 19.235 microseconds

    Please show us how Newton explains this gravitational effect.

    (If you wonder, the clock is in GPS orbit, and the effect is
    confirmed by the fact that the GPS works.)

    https://paulba.no/

    How can it fail to explain it? Relativity has not even explained the difference in clock rates at all.

    No theory "Explains" anything.
    Neither Newton nor GR "explain" gravitation, they postulate it.

    All a theory of physics can do is to predict what will be
    measured in experiments.


    This is Newtonian physics:
    ---------------------------
    1. Newtons laws of motion. 2. law: F = dp/dt
    2. Newtons law of gravitation: F = GMm/r²
    3. Galilean relativity.
    t' = t
    x' = x - vt
    y' = y
    z' = z

    Please show what this theory will predict for the following
    experiment:

    A clock is in circular orbit with radius 26561763 metres,
    and the orbital period measured by clocks on the ground
    is 43082.045250000 seconds. What is the orbital period
    measured by the orbiting clock?

    We know that GR correctly predicts that the orbiting clock
    will measure the orbital period to be 43082.045269235 seconds.

    Will Newton predict the same?

    --
    Paul

    https://paulba.no/

    --- SoupGate-Win32 v1.05
    * Origin: fsxNet Usenet Gateway (21:1/5)
  • From Volney@21:1/5 to Lou on Tue Jan 9 09:42:53 2024
    On 1/9/2024 9:28 AM, Lou wrote:
    On Tuesday 9 January 2024 at 13:18:42 UTC, Paul B. Andersen wrote:
    Den 08.01.2024 22:59, skrev Laurence Clark Crossen:
    On Monday, January 8, 2024 at 5:38:59 AM UTC-8, Paul B. Andersen wrote: >>>> Den 07.01.2024 21:09, skrev Laurence Clark Crossen:

    Any effect of gravity on atomic clocks would be Newtonian. Relativity explains nothing about it.

    It is a proven fact that a clock in circular orbit with radius
    26561763 metres will advance 43082.045269235 seconds per orbit
    while a clock on the geoid will advance 43082.045250000 seconds.
    The difference is 19.235 microseconds

    Please show us how Newton explains this gravitational effect.

    (If you wonder, the clock is in GPS orbit, and the effect is
    confirmed by the fact that the GPS works.)

    https://paulba.no/

    How can it fail to explain it? Relativity has not even explained the difference in clock rates at all.
    No theory "Explains" anything.
    Neither Newton nor GR "explain" gravitation, they postulate it.

    All a theory of physics can do is to predict what will be
    measured in experiments.


    This is Newtonian physics:
    ---------------------------
    1. Newtons laws of motion. 2. law: F = dp/dt
    2. Newtons law of gravitation: F = GMm/r²
    3. Galilean relativity.
    t' = t
    x' = x - vt
    y' = y
    z' = z

    Please show what this theory will predict for the following
    experiment:

    A clock is in circular orbit with radius 26561763 metres,
    and the orbital period measured by clocks on the ground
    is 43082.045250000 seconds. What is the orbital period
    measured by the orbiting clock?

    We know that GR correctly predicts that the orbiting clock
    will measure the orbital period to be 43082.045269235 seconds.

    Will Newton predict the same?

    Depends on which Newton formula you use.

    That makes no sense, since there will be only one applicable Newtonian
    formula for any situation.

    If you use Newton’s potential, as Relativity does, then yes Newton
    can predict the clock gains with an accuracy almost as good
    as Relativity.

    Umm, no. Newtonian time is the same everywhere. Remember the Galilean
    transform where t' = t?

    Newtonian physics doesn't use the gravitational potential at all except
    to calculate potential energy due to gravity.

    And from that I suppose one can predict
    the orbital period.

    With Newtonian physics, if the ground time is 43082.045250000 seconds,
    then the satellite time is also 43082.045250000 seconds. Remember, t'=t.
    So Newtonian physics gives the wrong answer.

    --- SoupGate-Win32 v1.05
    * Origin: fsxNet Usenet Gateway (21:1/5)
  • From Volney@21:1/5 to Lou on Tue Jan 9 09:28:26 2024
    On 1/9/2024 8:09 AM, Lou wrote:
    On Tuesday 9 January 2024 at 12:39:08 UTC, Paul B. Andersen wrote:
    Den 08.01.2024 22:58, skrev Laurence Clark Crossen:
    On Monday, January 8, 2024 at 4:46:58 AM UTC-8, Paul B. Andersen wrote: >>
    All these experiments confirm the predictions of
    the Schwartzschild metric:

    https://paulba.no/paper/Pound&Rebka.pdf
    https://paulba.no/paper/Hafele.pdf
    https://paulba.no/paper/Alley.pdf
    (see experiment on pages 708-716
    https://paulba.no/paper/Initial_results_of_GPS_satellite_1977.pdf
    https://paulba.no/paper/Vessot.pdf

    And the satellite navigation systems
    GPS, GLONASS, Galileo and BeiDou are continuously confirming
    of the predictions of the Schwartzschild metric.

    The hallmark of a crank is that he ignores experimental evidence.

    An illogical theory cannot be proven by experiment because it does not predict. When the necromancer predicts the winner of a horse race we do not think it was thanks to necromancy.
    Your opinion of the consistency of GR is simply wrong.
    It is a fact that GR is mathematically consistent.

    No theory can be proven, but a theory must be falsifiable.
    A theory is tested by calculating what the theory predicts will
    be measured in an experiment, and then comparing the predicted
    values with the measured values when the experiment is performed.

    If the predictions are in accordance with the measurements
    within the precision of the measurements, then the theory
    is confirmed (NOT proven).
    If the predictions are not in accordance with the measurements,
    then the theory is falsified.

    It is a fact that predictions of GR for
    the experiments above are in accordance with
    the measurements.

    So it is PROVEN that GR gives the correct predictions
    for those particular experiments, and for the satellite
    navigation systems GPS, GLONASS, Galileo and BeiDou.

    Denying facts is irrational behaviour!

    Yes. But asking for the facts isn’t.

    And ignoring the facts, once given to you, certainly is.

    It’s still not clear where the original prediction for a total offset of 4.1
    for the frequency C-133 (9192631770) comes from. The earliest I can
    find is GM/r ÷ c^2.

    As I mentioned, you stumbled across GR's Schwarzschild's metric, GM/rc².
    Good job at helping validate GR.

    And 1/2 v^2. But neither specifically state exact
    predicted amounts for clock gains/losses. Just ratios of gains vs
    radius using Newtonian potential.

    You merely have to apply GR's formula once you have the values.

    Because its not clear if GR did predict a 9192631774.1 total offset for r4.12 As I can’t find any actual prediction of this 4.1 frequency offset until after 1977.

    Look at the design specs for the NTS-2 satellite.

    At which point the clock gain had already been measured by the first
    GPS satelitte.

    How could they have preset the value into the satellite before launch?
    You keep ignoring this question!

    And as Ashby says in his paper the theorists and GPS engineers were
    not sure what the magnitude would be before launch.

    The GR formula is quite specific. What did you misinterpret here?

    Why would
    Ashby the expert, if anyone is, say this if it wasn’t true?

    We need to find what you misinterpreted.

    So my question is ..Was an exact offset of 4.1 for C-133 preset in the
    first GPS sat?

    Go check the design specs for NTS-2. The answer is yes.

    Or was the offset “switch” variable?

    No, it wasn't. It was yes/no, or Newtonian/GR.

    Because the various literature
    on the first test indicates the ground engineers were able to monitor
    the daily offset over weeks *before* switching on.

    That was part of the purpose of the switch. It was an experiment of
    whether the predictions of GR would be confirmed or falsified.

    So it looks they
    measured the offset before they knew it’s exact magnitude.

    Wrong. They knew the exact magnitude from the GR formula.

    So my question is: Had the option to *vary* the switchs daily offset magnitude been built into the first satelitte once they knew the exact magnitude of the clock gains from a few weeks observation?

    No.

    Because
    if Ashby was correct and no exact magnitude was ever even predicted
    ....would any sane NASA engineer have sent up a test satelitte with
    an offset switch of an exact amount attached if they didn’t
    know how much offset to set the switch to was known?

    They knew. It was just an argument of whether GR applied, and a simple
    remote on/off switch was all they would need.

    Is there any cited reference on this?

    Yes, the design spec for the NTS-2. It's online; I found it before.

    --- SoupGate-Win32 v1.05
    * Origin: fsxNet Usenet Gateway (21:1/5)
  • From Paul B. Andersen@21:1/5 to All on Tue Jan 9 20:06:41 2024
    Den 09.01.2024 14:09, skrev Lou:
    On Tuesday 9 January 2024 at 12:39:08 UTC, Paul B. Andersen wrote:
    Den 08.01.2024 22:58, skrev Laurence Clark Crossen:
    On Monday, January 8, 2024 at 4:46:58 AM UTC-8, Paul B. Andersen wrote: >>
    All these experiments confirm the predictions of
    the Schwartzschild metric:

    https://paulba.no/paper/Pound&Rebka.pdf
    https://paulba.no/paper/Hafele.pdf
    https://paulba.no/paper/Alley.pdf
    (see experiment on pages 708-716
    https://paulba.no/paper/Initial_results_of_GPS_satellite_1977.pdf
    https://paulba.no/paper/Vessot.pdf

    And the satellite navigation systems
    GPS, GLONASS, Galileo and BeiDou are continuously confirming
    of the predictions of the Schwartzschild metric.

    The hallmark of a crank is that he ignores experimental evidence.

    An illogical theory cannot be proven by experiment because it does not predict. When the necromancer predicts the winner of a horse race we do not think it was thanks to necromancy.
    Your opinion of the consistency of GR is simply wrong.
    It is a fact that GR is mathematically consistent.

    No theory can be proven, but a theory must be falsifiable.
    A theory is tested by calculating what the theory predicts will
    be measured in an experiment, and then comparing the predicted
    values with the measured values when the experiment is performed.

    If the predictions are in accordance with the measurements
    within the precision of the measurements, then the theory
    is confirmed (NOT proven).
    If the predictions are not in accordance with the measurements,
    then the theory is falsified.

    It is a fact that predictions of GR for
    the experiments above are in accordance with
    the measurements.

    So it is PROVEN that GR gives the correct predictions
    for those particular experiments, and for the satellite
    navigation systems GPS, GLONASS, Galileo and BeiDou.

    Denying facts is irrational behaviour!


    Yes. But asking for the facts isn’t.

    You are obviously ignoring the facts given above.
    Why is that?
    Are you denying them?

    --
    Paul

    https://paulba.no/

    --- SoupGate-Win32 v1.05
    * Origin: fsxNet Usenet Gateway (21:1/5)
  • From Paul B. Andersen@21:1/5 to All on Tue Jan 9 19:53:31 2024
    Den 08.01.2024 22:06, skrev Lou:
    Do you know if there was any particular reason why the GPS engineers
    chose 10230000hz for the clock chip rate frequency?

    No, but it was probably because this chipping rate was appropriate.
    One of the reasons for the modulation of the carrier with
    the PRN code was to make the bandwidth wider, with 10.23 MHz
    chipping rate the bandwidth will be 20.46 MHz. It's a kind of
    spread spectrum technology to make the signal less sensitive for
    noise and for other signals within the bandwidth (jamming).
    Since the receiver knows the PRN codes, the receiver can recognise
    the signal buried in the noise (matched filter), and identify
    which satellite sent the signal.

    Remember that all the satellites send the same carrier frequencies,
    and at the receiver all these carriers are Doppler shifted differently,
    and the signal from many satellites will be received by the same
    radio receiver, so the signal is very noisy. So it is only kind
    of matched filters that can separate the signals from the different
    satellites.


    I was wondering if it was
    because there was a relationship between the caesium clock frequency
    of 9192631770, c^2 and 10230000.

    Don't think so.
    It is not only Cs with frequency 9192631770 Hz that is used,
    it is also Rubidium with frequency 6834682610.904 Hz.
    So the frequency synthesiser has to be different if it is
    Rb oscillator than a Cs oscillator.

    https://www.electricity-magnetism.org/frequency-synthesizers/

    Note that in a frequency synthesiser the output frequency
    is N/M x reference_frequency where N and M are integers.

    I tried variations of the 3 to see if there was any patterns
    where r’ is GPS orbit of 4.12x 6371000m:

    Why should the radius of the orbit have anything to do with
    the choice of chipping frequency?

    That is a rather nonsensical idea!


    GM/r-r’ = 47379430.8842

    r-r' ? :-D

    r' = 4.12 Earth radii is an approximation of the GPS radius
    with a precision of only three significant digits
    (or even less due to the ambiguity of "Earth radius"),
    so why would you subtract this from the precise radius which
    is 26561763 m with a precision of 8 significant digits?

    What a weird idea!

    c^2 = 8.9875518e+16

    47379430.8842 ÷ 9192631770= 0.005154066
    47379430.8842 ÷ 10229999.99543 = 4.63142042086
    4.63142042086 ÷ 0.005154066= 898.59548187
    1 ÷ 0.005154066= 194.021574423
    898.495561647 ÷ 194.021574423= 4.63090542543
    10230000 × 898.59548134 = 9192631774.1
    898.59548134 × 10229999.9954 =9192631770
    9192631770 ÷ 10229999.9954326= 898.59548134



    --
    Paul

    https://paulba.no/

    --- SoupGate-Win32 v1.05
    * Origin: fsxNet Usenet Gateway (21:1/5)
  • From Paul B. Andersen@21:1/5 to All on Wed Jan 10 10:22:29 2024
    Den 09.01.2024 22:29, skrev Laurence Clark Crossen:
    On Tuesday, January 9, 2024 at 4:39:08 AM UTC-8, Paul B. Andersen wrote:
    Den 08.01.2024 22:58, skrev Laurence Clark Crossen:

    An illogical theory cannot be proven by experiment because it does not predict. When the necromancer predicts the winner of a horse race we do not think it was thanks to necromancy.

    Your opinion of the consistency of GR is simply wrong.
    It is a fact that GR is mathematically consistent.

    No theory can be proven, but a theory must be falsifiable.
    A theory is tested by calculating what the theory predicts will
    be measured in an experiment, and then comparing the predicted
    values with the measured values when the experiment is performed.

    If the predictions are in accordance with the measurements
    within the precision of the measurements, then the theory
    is confirmed (NOT proven).
    If the predictions are not in accordance with the measurements,
    then the theory is falsified.


    Two typical defense tactics of relativists are to talk down to their opponents and to demand unnecessarily precise language.

    A typical tactics for cranks is to ignore experimental
    evidence like this:

    https://paulba.no/paper/Pound&Rebka.pdf
    https://paulba.no/paper/Hafele.pdf
    https://paulba.no/paper/Alley.pdf
    (see experiment on pages 708-716 https://paulba.no/paper/Initial_results_of_GPS_satellite_1977.pdf https://paulba.no/paper/Vessot.pdf

    And the satellite navigation systems
    GPS, GLONASS, Galileo and BeiDou are continuously confirming
    of the predictions of GR.

    The fact that these experiments prove that GR gives
    the correct predictions for those particular experiments,
    and for the satellite navigation systems GPS, GLONASS,
    Galileo and BeiDou won't go away because Laurence Clark Crossen
    doesn't like them.

    Denying facts is irrational behavior!


    Everyone knows your points, and I read Karl Popper long ago.

    Everyone knows that relativity asserts that it is perfectly mathematically consistent. Many excellent scientists have shown this is not true.

    In so far as the math at times is consistent, the physics is not.

    It is irrational behavior to fail to answer my most crucial point.
    You have not conceded that an illogical theory does not make verifiable predictions.
    An illogical theory cannot be verified/confirmed because it does not make unambiguous predictions.

    Relativity is an illogical theory, so it does not make predictions. As Essen, Rutherford, and Soddy said, it is not even a scientific theory. Relativity is thoroughly irrational.

    The idea that a postulate does not explain is ridiculous.

    What is the relativity mechanism or cause of the difference in clock rates? If it is gravity, then Newton has a prediction.

    Claiming the Newton mode would be no adjustment at all is so flagrantly false as to be a deceitful obfuscatory tactic.

    I acknowledge the accurate prediction was made and deny it could have had anything to do with relativity contrary to James A. Buisson, Roger L. Easton, Thomas B. McCaskill
    U. S. Naval Research Laboratory (NRL). How did they really predict it?

    It's frustrating that a theory which Laurence Clark Crossen
    claims is illogical and can't make unambiguous predictions,
    can make precise correct predictions for how clocks behave in
    the gravitational field in Earth's vicinity, isn't it? :-D

    Must be necromancy, don't you think?

    BTW, here are more experiments which confirm SR/GR : https://paulba.no/paper/Kennedy_Thorndike.pdf https://paulba.no/paper/Ives_Stilwell.pdf https://paulba.no/paper/Ives_Stilwell_II.pdf https://paulba.no/paper/Clemence.pdf https://paulba.no/paper/Babcock_Bergman.pdf https://paulba.no/paper/Frisch_Smith.pdf https://paulba.no/paper/Alvager_et_al.pdf https://paulba.no/paper/Beckmann_Mandics.pdf https://paulba.no/paper/Filippas_Fox.pdf https://paulba.no/paper/Shapiro_1964.pdf https://paulba.no/paper/Shapiro_1968.pdf
    https://paulba.no/paper/Brecher.pdf
    https://paulba.no/paper/Brillet_Hall.pdf https://paulba.no/paper/PPN_gamma_Hipparcos.pdf https://paulba.no/paper/PPN_gamma_Cassini.pdf https://paulba.no/paper/Shapiro_2004.pdf
    https://paulba.no/paper/Liu.pdf
    https://paulba.no/paper/GravDeflection.pdf https://paulba.no/paper/Remmen_McCreary.pdf https://paulba.no/paper/Botermann.pdf https://paulba.no/paper/LIGO_GravitationalWaves.pdf https://paulba.no/paper/LIGO_TestOfGR.pdf https://paulba.no/paper/LIGO_GravitationalWaves_2.pdf

    They will all go away if you ignore them! :-D


    --
    Paul

    https://paulba.no/

    --- SoupGate-Win32 v1.05
    * Origin: fsxNet Usenet Gateway (21:1/5)
  • From Volney@21:1/5 to Lou on Wed Jan 10 12:35:09 2024
    On 1/9/2024 10:08 AM, Lou wrote:
    On Tuesday 9 January 2024 at 14:42:58 UTC, Volney wrote:
    On 1/9/2024 9:28 AM, Lou wrote:
    On Tuesday 9 January 2024 at 13:18:42 UTC, Paul B. Andersen wrote:
    Den 08.01.2024 22:59, skrev Laurence Clark Crossen:
    On Monday, January 8, 2024 at 5:38:59 AM UTC-8, Paul B. Andersen wrote: >>>>>> Den 07.01.2024 21:09, skrev Laurence Clark Crossen:

    Any effect of gravity on atomic clocks would be Newtonian. Relativity explains nothing about it.

    It is a proven fact that a clock in circular orbit with radius
    26561763 metres will advance 43082.045269235 seconds per orbit
    while a clock on the geoid will advance 43082.045250000 seconds.
    The difference is 19.235 microseconds

    Please show us how Newton explains this gravitational effect.

    (If you wonder, the clock is in GPS orbit, and the effect is
    confirmed by the fact that the GPS works.)

    https://paulba.no/

    How can it fail to explain it? Relativity has not even explained the difference in clock rates at all.
    No theory "Explains" anything.
    Neither Newton nor GR "explain" gravitation, they postulate it.

    All a theory of physics can do is to predict what will be
    measured in experiments.


    This is Newtonian physics:
    ---------------------------
    1. Newtons laws of motion. 2. law: F = dp/dt
    2. Newtons law of gravitation: F = GMm/r²
    3. Galilean relativity.
    t' = t
    x' = x - vt
    y' = y
    z' = z

    Please show what this theory will predict for the following
    experiment:

    A clock is in circular orbit with radius 26561763 metres,
    and the orbital period measured by clocks on the ground
    is 43082.045250000 seconds. What is the orbital period
    measured by the orbiting clock?

    We know that GR correctly predicts that the orbiting clock
    will measure the orbital period to be 43082.045269235 seconds.

    Will Newton predict the same?

    Depends on which Newton formula you use.
    That makes no sense, since there will be only one applicable Newtonian
    formula for any situation.

    Yes. It’s called potential and it’s GM/r. And it works. Otherwise relativity
    wouldn’t have borrowed it.

    As the weak field approximation, GR derived it, as an approximation.

    If you use Newton’s potential, as Relativity does, then yes Newton
    can predict the clock gains with an accuracy almost as good
    as Relativity.

    Umm, no. Newtonian time is the same everywhere. Remember the Galilean
    transform where t' = t?

    Yes. Classical physics has time same everywhere. But it explains
    clock gains with resonance using GM/r.

    Your second sentence contradicts the first. If time was the same
    everywhere, then there will be no clock gain, because, well, the time is
    the same everywhere! t'=t.

    Newtonian physics doesn't use the gravitational potential at all except
    to calculate potential energy due to gravity.

    Newton didn’t. But classical physics does. And it works.
    Albeit with error margins of 0.00021 vs Relativity’s 0.0001.
    Although fact is if it’s more accurate,I don’t see why GM/r ÷ c^2
    cant be used by a classical model.

    Since GR is a classical model (not quantum), the classical GR model uses
    it just fine as an approximation.

    After all c^2 is just a number

    No, it is not. It has dimensionality of length²/time².

    and GM/r is “Newtonian”

    and a component of the Schwarzschild metric. It also has dimensions of length²/time². Divide it by c² and get the Schwarzschild metric, GM/rc²,
    a dimensionless number that's essentially the curvature of space


    And from that I suppose one can predict
    the orbital period.
    With Newtonian physics, if the ground time is 43082.045250000 seconds,
    then the satellite time is also 43082.045250000 seconds. Remember, t'=t.

    So Newtonian physics gives the wrong answer.

    You forgot. The time doesn’t change. The atoms resonant frequency beats faster at higher altitudes.

    No, the satellite clocks are designed to tick at the correct rate and
    not to be affected by external events. That's why they are so accurate,
    they can't be affected by external things like a ground level Cs clock
    being affected by the gravity of a fat scientist walking by it. Besides,
    any "resonant frequency" effects would be proportional to force, which
    varies as 1/r², but the altitude effect varies at 1/r. And the clocks
    are in freefall, so there is no force or 1/r² effect on them. Newtonian physics nothing is affected proportional to 1/r other than the potential
    energy of a mass.

    --- SoupGate-Win32 v1.05
    * Origin: fsxNet Usenet Gateway (21:1/5)
  • From Volney@21:1/5 to Laurence Clark Crossen on Wed Jan 10 12:43:19 2024
    On 1/9/2024 4:44 PM, Laurence Clark Crossen wrote:
    On Tuesday, January 9, 2024 at 6:42:58 AM UTC-8, Volney wrote:
    On 1/9/2024 9:28 AM, Lou wrote:

    And from that I suppose one can predict
    the orbital period.

    With Newtonian physics, if the ground time is 43082.045250000 seconds,
    then the satellite time is also 43082.045250000 seconds. Remember, t'=t.
    So Newtonian physics gives the wrong answer.

    To equate the functioning of the clock with time is not correct.
    Claiming that Newton can't predict the clock's rate because Newton does not have variable time is obvious nonsense.

    "Can't predict the clock's rate" is wrong. Newtonian physics predicts no
    effect on the clock rate because all clocks in the universe tick the
    same and nothing affects them. In Newtonian physics the ground time and satellite time are both 43082.045250000 seconds (no change, t'=t), but
    this is not what is measured.

    --- SoupGate-Win32 v1.05
    * Origin: fsxNet Usenet Gateway (21:1/5)
  • From Paul B. Andersen@21:1/5 to All on Wed Jan 10 22:21:50 2024
    Den 10.01.2024 11:21, skrev Lou:
    Den 08.01.2024 22:06, skrev Lou:

    GM/r-r’ = 47379430.8842

    How does one get the potential for GPS orbital radius.?
    You calculate potentials for earth surface r and GPS orbit r’
    Then to get total for GPS one subtracts r’ from r.

    A clock stationary in the ECI frame at a distance r' from
    the center of the Earth with radius r would run at a rate
    relative to a clock on the geoid:

    dτ/dt ≈ 1 + (GM/r - GM/r')/c²

    So according to you: (GM/r - GM/r') = GM/(r-r') :-D

    'nuff said.


    Which using average 4.12 and assuming radius r is 6371000m
    gives 47379430.8842
    At that point divide by whatever you feel like. GR divides
    into fantasy number 8.9875518e+16 for some unknown bizarre reason
    Which is why I question all the assumptions of 5.27e-10
    being used as a frequency ratio 1.000000000527 to calculate
    clock gains for GR.
    How on earth do you guys figure that 1/89875518000000000
    of 47379430.8842 has anything to do with frequency ratios?
    All 5.27e-10 is ...is 1/89875518000000000 of the potential at
    earth orbit 4.12. Your formula 1.000000000527 is a fantasy number.
    It should have WAY more zeros if 5.27e-10 is just
    1/89875518000000000 of GM/r4.12
    That’s why I question the whole pretext of observed clock gains
    matching predictions by GR.
    You faked the formula 1.000000000527 to match the observations.

    Confused, Lou? :-D

    I have shown you this before, SO READ IT!

    An approximation of the Schwarzschild metric:

    The rate of Schwarzschild coordinate time t is the same
    as the rate of a clock at infinity.

    The rate of a clock at distance r and speed v in the ECI-frame
    relative to Schwarzschild coordinate time is:

    dτ/dt = (1 - GM/r⋅c² - v²/2c²) (1)

    GM/r is the Newtonian gravitational potential, but that doesn't
    make the Schwarzschild metric "non relativistic".

    If the clock is in circular orbit then v² = GM/r and (1) can be written:

    dτ/dt = (1 - 1.5⋅GM/r⋅c²) (2)

    This is the rate of a clock in circular orbit relative to
    Schwarzschild coordinate time, but we are more interested in
    the rate of the clock relative to Universal Coordinated time (UTC).

    We consult Ashby:
    https://paulba.no/paper/Ashby.pdf
    see equation (18) page 11.

    dt_utc/dt = (1 - δutc) where δutc = 6.96927E-10

    dτ/dt_utc = (1 - 1.5⋅GM/r⋅c²)/(1 - δutc) ≃ (1 - 1.5⋅GM/r⋅c² + δutc) (3)

    Δf/f₀ = (dτ/dt_utc - 1) = - (1.5⋅GM/r⋅c² - δutc) (4)

    For the GPS the orbital period p is specified to be half a sidereal day:
    p = 43082.04525 s
    GM = 3.986004418E14 m³/s²
    c = 299792458 m/s
    r = cbrt(GM⋅p²/4π²) = 26561763 m

    Equation (4) yields; Δf/f₀ = 4.46471409E-10

    This means that the clock will run fast relative
    to UTC, so to stay in sync with UTC it must be
    corrected by Δf/f₀ = - 4.46471409E-10

    In the GPS specification the correction is set to
    Δf/f₀ = - 4.4647E-10





    --
    Paul

    https://paulba.no/

    --- SoupGate-Win32 v1.05
    * Origin: fsxNet Usenet Gateway (21:1/5)
  • From Volney@21:1/5 to Laurence Clark Crossen on Wed Jan 10 19:28:01 2024
    On 1/10/2024 4:35 PM, Laurence Clark Crossen wrote:
    On Wednesday, January 10, 2024 at 9:43:23 AM UTC-8, Volney wrote:
    On 1/9/2024 4:44 PM, Laurence Clark Crossen wrote:
    On Tuesday, January 9, 2024 at 6:42:58 AM UTC-8, Volney wrote:
    On 1/9/2024 9:28 AM, Lou wrote:

    And from that I suppose one can predict
    the orbital period.

    With Newtonian physics, if the ground time is 43082.045250000 seconds, >>>> then the satellite time is also 43082.045250000 seconds. Remember, t'=t. >>>> So Newtonian physics gives the wrong answer.

    To equate the functioning of the clock with time is not correct.
    Claiming that Newton can't predict the clock's rate because Newton does not have variable time is obvious nonsense.
    "Can't predict the clock's rate" is wrong. Newtonian physics predicts no
    effect on the clock rate because all clocks in the universe tick the
    same and nothing affects them. In Newtonian physics the ground time and
    satellite time are both 43082.045250000 seconds (no change, t'=t), but
    this is not what is measured.

    Newton does not say all clocks work the same under any conditions.

    "If time was the same
    everywhere, then there will be no clock gain, because, well, the time is
    the same everywhere! t'=t." Yes, well try moving a pendulum to a different latitude without changing its length. It will run at a different rate yet time is the same. It is one one of the dumbest parts of relativity to equate time and the readings of
    clocks.

    You are (deliberately) forgetting that part of a pendulum clock is the
    mass of the earth. Take the clock to the moon and you replace earth's
    mass with the moon's mass and it no longer keeps correct time, not even
    close. Without any mass (your cuckoo clock in deep space) it doesn't
    work at all.

    But yes, a pendulum clock (excluding the earth's mass) is an example of
    a clock that really is affected by force (gravity on the pendulum).

    As Lou said, "> You forgot. The time doesn’t change. The atoms resonant frequency beats faster at higher altitudes." - That's why it is set to a lower frequency.

    And, obviously, Lou is wrong. As are you.

    --- SoupGate-Win32 v1.05
    * Origin: fsxNet Usenet Gateway (21:1/5)
  • From Volney@21:1/5 to Lou on Wed Jan 10 19:43:36 2024
    On 1/10/2024 4:42 PM, Lou wrote:
    On Wednesday 10 January 2024 at 21:18:24 UTC, Paul B. Andersen wrote:
    Den 10.01.2024 11:21, skrev Lou:
    Den 08.01.2024 22:06, skrev Lou:
    GM/r-r’ = 47379430.8842
    How does one get the potential for GPS orbital radius.?
    You calculate potentials for earth surface r and GPS orbit r’
    Then to get total for GPS one subtracts r’ from r.
    A clock stationary in the ECI frame at a distance r' from
    the center of the Earth with radius r would run at a rate
    relative to a clock on the geoid:

    dτ/dt ≈ 1 + (GM/r - GM/r')/c²

    So according to you: (GM/r - GM/r') = GM/(r-r') :-D
    'nuff said.

    Hahahaha!!!!!

    Not sure how you figure I’m wrong. To start with relativity uses
    something very similar. But let me do my formulas calculation for you.
    (GM/r -earth) - (GM/r sat) ÷ f
    Using r= 6371000 and f= 8.9875518e+16:

    GM/r (earth) = 62565145.91115994
    GM/r (sat) = 15185715.02698057
    (GM/r)-(GM/r’) = 47379430.8842
    47379430.8842/8.9875518e+16 =5.2716726e-10

    Does that sound incorrect to you?

    Not since you fixed your mistake! Remember you wrote:
    How does one get the potential for GPS orbital radius.?
    You calculate potentials for earth surface r and GPS orbit r’
    Then to get total for GPS one subtracts r’ from r.

    Your GM/(r-r') is wrong.

    --- SoupGate-Win32 v1.05
    * Origin: fsxNet Usenet Gateway (21:1/5)
  • From Volney@21:1/5 to Lou on Wed Jan 10 19:15:58 2024
    On 1/10/2024 4:01 PM, Lou wrote:
    On Wednesday 10 January 2024 at 17:35:14 UTC, Volney wrote:
    On 1/9/2024 10:08 AM, Lou wrote:
    On Tuesday 9 January 2024 at 14:42:58 UTC, Volney wrote:

    Umm, no. Newtonian time is the same everywhere. Remember the Galilean
    transform where t' = t?

    Yes. Classical physics has time same everywhere. But it explains
    clock gains with resonance using GM/r.
    Your second sentence contradicts the first. If time was the same
    everywhere, then there will be no clock gain, because, well, the time is
    the same everywhere! t'=t.

    You forgot. A Classical model doesn’t have time change.

    I assume you mean Newtonian physics doesn't because classical GR
    certainly does.

    It
    has harmonic oscillators resonating at higher frequencies at
    lower gravitational fields.

    Except that would be proportional to the gravitational force or
    proportional to 1/r² while the weak field time dilation is proportional
    to 1/r. As you have been told REPEATEDLY. Did you forget or are you
    really stooopid?

    As observed.

    Observed to be proportional to 1/r, esp. as other sat navigation systems
    in different orbits (GALILEO, GLONASS, BEIDOU etc.) show.

    Newtonian physics doesn't use the gravitational potential at all except >>>> to calculate potential energy due to gravity.

    Newton didn’t. But classical physics does. And it works.

    Newtonian physics only uses it for potential energy per unit mass.

    Albeit with error margins of 0.00021 vs Relativity’s 0.0001.

    Obviously misapplied Newtonian physics is wrong.

    After all c^2 is just a number
    No, it is not. It has dimensionality of length²/time².

    What SI unit is it?

    Can't you read? Length²/time² or in SI, meters²/seconds².

    And from that I suppose one can predict
    the orbital period.
    With Newtonian physics, if the ground time is 43082.045250000 seconds, >>>> then the satellite time is also 43082.045250000 seconds. Remember, t'=t. >>
    So Newtonian physics gives the wrong answer.

    You forgot. The time doesn’t change. The atoms resonant frequency beats >>> faster at higher altitudes.

    No, the satellite clocks are designed to tick at the correct rate and
    not to be affected by external events. That's why they are so accurate,
    they can't be affected by external things like a ground level Cs clock
    being affected by the gravity of a fat scientist walking by it. Besides,
    any "resonant frequency" effects would be proportional to force, which
    varies as 1/r², but the altitude effect varies at 1/r. And the clocks
    are in freefall, so there is no force or 1/r² effect on them. Newtonian
    physics nothing is affected proportional to 1/r other than the potential
    energy of a mass.

    A clock at the top of Everest also has clock gains under classical theory using
    GM/r. Yet it’s not in free fall.

    So it has a force on it, yet it still works correctly, showing how a
    properly operating clock works correctly in freefall or not in freefall.
    The Schwarzschild metric still applies, of course.

    --- SoupGate-Win32 v1.05
    * Origin: fsxNet Usenet Gateway (21:1/5)
  • From Volney@21:1/5 to Lou on Thu Jan 11 13:23:48 2024
    On 1/11/2024 7:11 AM, Lou wrote:
    On Thursday 11 January 2024 at 00:43:42 UTC, Volney wrote:
    On 1/10/2024 4:42 PM, Lou wrote:
    On Wednesday 10 January 2024 at 21:18:24 UTC, Paul B. Andersen wrote:
    Den 10.01.2024 11:21, skrev Lou:
    Den 08.01.2024 22:06, skrev Lou:
    GM/r-r’ = 47379430.8842
    How does one get the potential for GPS orbital radius.?
    You calculate potentials for earth surface r and GPS orbit r’
    Then to get total for GPS one subtracts r’ from r.
    A clock stationary in the ECI frame at a distance r' from
    the center of the Earth with radius r would run at a rate
    relative to a clock on the geoid:

    dτ/dt ≈ 1 + (GM/r - GM/r')/c²

    So according to you: (GM/r - GM/r') = GM/(r-r') :-D
    'nuff said.

    Hahahaha!!!!!

    Not sure how you figure I’m wrong. To start with relativity uses
    something very similar. But let me do my formulas calculation for you.
    (GM/r -earth) - (GM/r sat) ÷ f
    Using r= 6371000 and f= 8.9875518e+16:

    GM/r (earth) = 62565145.91115994
    GM/r (sat) = 15185715.02698057
    (GM/r)-(GM/r’) = 47379430.8842
    47379430.8842/8.9875518e+16 =5.2716726e-10

    Does that sound incorrect to you?

    Not since you fixed your mistake! Remember you wrote:
    How does one get the potential for GPS orbital radius.?
    You calculate potentials for earth surface r and GPS orbit r’
    Then to get total for GPS one subtracts r’ from r.
    Your GM/(r-r') is wrong.

    Really?
    5.2716726e-10 is not the correct answer? It looks exactly the same as the prediction from GR. Are you finally admitting GR is wrong?
    GM/r -earth) - (GM/r sat) ÷ f
    Using r= 6371000 and f= 8.9875518e+16:

    GM/r (earth) = 62565145.91115994
    GM/r (sat) = 15185715.02698057
    (GM/r)-(GM/r’) = 47379430.8842
    47379430.8842/8.9875518e+16 =5.2716726e-10

    Again, your ORIGINAL claim, GM/(r-r') is wrong! Paul caught you, and in
    your reply to him, you corrected your mistake by writing (GM/r)-(GM/r’).
    I am laughing at your ORIGINAL mistake! Sloppy, sloppy, sloppy!! (GM/r)-(GM/r’) is NOT equal to GM/(r-r')! (and you persist in your
    sloppiness by calling the number which has the value of c² a frequency.
    In GR it really is c². Units don't match, automatically wrong!)

    --- SoupGate-Win32 v1.05
    * Origin: fsxNet Usenet Gateway (21:1/5)
  • From Paul B. Andersen@21:1/5 to All on Thu Jan 11 21:26:52 2024
    Den 10.01.2024 22:42, skrev Lou:
    On Wednesday 10 January 2024 at 21:18:24 UTC, Paul B. Andersen wrote:
    Den 10.01.2024 11:21, skrev Lou:
    Den 08.01.2024 22:06, skrev Lou:

    GM/r-r’ = 47379430.8842
    How does one get the potential for GPS orbital radius.?
    You calculate potentials for earth surface r and GPS orbit r’
    Then to get total for GPS one subtracts r’ from r.

    So you claim that the potential difference is GM/(r-r’)

    A clock stationary in the ECI frame at a distance r' from
    the center of the Earth with radius r would run at a rate
    relative to a clock on the geoid:

    Should be: relative to a clock stationary in the ECI-frame
    at a distance r from the center of the Earth.


    dτ/dt ≈ 1 + (GM/r - GM/r')/c²

    So according to you: (GM/r - GM/r') = GM/(r-r') :-D

    This is a gigantic blunder, and you have still not got it!


    Not sure how you figure I’m wrong. To start with relativity uses
    something very similar. But let me do my formulas calculation for you.
    (GM/r -earth) - (GM/r sat) ÷ f

    That's not your formula, it's my formula above,
    but why do you call c² for f?

    Using r= 6371000 and f= 8.9875518e+16:

    GM/r (earth) = 62565145.91115994
    GM/r (sat) = 15185715.02698057
    (GM/r)-(GM/r’) = 47379430.8842
    47379430.8842/8.9875518e+16 =5.2716726e-10

    Does that sound incorrect to you?

    Close enough.
    It is my formula with your numbers:
    Δf/f₀ = dτ/dt - 1 = (GM/r - GM/r')/c² = 5.2915706e-10

    Note that this means that a stationary clock in the ECI frame
    at a distance r' from the centre of the Earth would gain
    5.2915706e-10⋅(24⋅60⋅60) s/day = 4.57 μs/day
    on a stationary clock in the ECI frame at a distance r from
    the centre of the Earth.

    Note that the latter clock isn't at the Earth,
    and a GPS SV isn't stationary in the ECI frame,
    so this hasn't much to do with GPS.

    YOUR formula is:
    Δf/f₀ = (GM/(r - r'))/c² = 2.231178e-10

    which is WRONG.

    Learn this:
    (1/r - 1/r') =/= 1/(r - r')

    You have made a gigantic mathematical blunder!

    Which using average 4.12 and assuming radius r is 6371000m
    gives 47379430.8842

    You throw out numbers without defining what they are, and
    always without dimensions.

    Here you probably mean:
    (GM/r - GM/r') = 47379430.8842 m²/s²
    So why didn't you say so?

    But let us see what this mean.
    m²/s² = m/s² x m = N/kg x m = J/kg

    According to Newton:

    If you lift a mass m = 1kg from r to r' from the center
    of the Earth, the mass will gain a potential energy U.

    The gravitational acceleration is g = -GM/r²
    and the force is gm. So the gained energy is:

    U = ∫ᵣʳ'gm dr = -GMm∫ᵣʳ'(1/r²)dr = mGM(1/r - 1/r') = 47379430.8842 J

    So far this has nothing to do with relativity or clock rates.

    At that point divide by whatever you feel like. GR divides
    into fantasy number 8.9875518e+16 for some unknown bizarre reason

    The square of the speed of light a fantasy number? :-D

    THIS equation is a prediction of GR:
    Δf/f₀ = (GM/rc² - GM/r'c²) = 5.2915706e-10
    and the result is a ratio of two clock rates.

    That you don't understand why the mysterious c²
    is there is because you don't know GR.

    But look at the dimension.
    (GM/r - GM/r') has the dimension m²/s²
    and c² has the dimension m²/s²

    so Δf/f₀ = (GM/rc² - GM/r'c²) = 5.2915706e-10
    is a dimensionless ratio.


    I have shown you this before, SO READ IT!

    Yes you showed me more than a month ago. And it’s still wrong.
    Because you still have yet to explain your fantasy as to how dividing GM/r into 89875518000000000 pieces has anything to do
    with -10, frequencies or ratios of frequencies.
    You just arbitrarily pretended you could use 1/ 89875518000000000
    of GM/r in a formula with just nine zeros and multiply 10229999.9954326
    with it. You forgot a bunch numbers. Your 5.27 should be 1.0000000000000527 Not 1.000000000527
    But because the proper number of zeros doesn’t work...you faked it.

    The equation is:
    Δf/f₀ = - (1.5⋅GM/r⋅c² - δutc)

    And the relevant numbers are:
    GM = 3.986004418E14 m³/s²
    c = 299792458 m/s
    r = 26561763 m
    δutc = 6.96927E-10

    Maybe you can calculate Δf/f₀ and get all the zeros on their
    right place so you can get the correct answer in stead of
    the answer I faked?





    An approximation of the Schwarzschild metric:

    The rate of Schwarzschild coordinate time t is the same
    as the rate of a clock at infinity.

    The rate of a clock at distance r and speed v in the ECI-frame
    relative to Schwarzschild coordinate time is:

    dτ/dt = (1 - GM/r⋅c² - v²/2c²) (1)

    GM/r is the Newtonian gravitational potential, but that doesn't
    make the Schwarzschild metric "non relativistic".

    If the clock is in circular orbit then v² = GM/r and (1) can be written:

    dτ/dt = (1 - 1.5⋅GM/r⋅c²) (2)

    This is the rate of a clock in circular orbit relative to
    Schwarzschild coordinate time, but we are more interested in
    the rate of the clock relative to Universal Coordinated time (UTC).

    We consult Ashby:
    https://paulba.no/paper/Ashby.pdf
    see equation (18) page 11.

    dt_utc/dt = (1 - δutc) where δutc = 6.96927E-10

    dτ/dt_utc = (1 - 1.5⋅GM/r⋅c²)/(1 - δutc) ≃ (1 - 1.5⋅GM/r⋅c² + δutc) (3)

    Δf/f₀ = (dτ/dt_utc - 1) = - (1.5⋅GM/r⋅c² - δutc) (4)

    For the GPS the orbital period p is specified to be half a sidereal day:
    p = 43082.04525 s
    GM = 3.986004418E14 m³/s²
    c = 299792458 m/s
    r = cbrt(GM⋅p²/4π²) = 26561763 m

    Equation (4) yields; Δf/f₀ = 4.46471409E-10

    This means that the clock will run fast relative
    to UTC, so to stay in sync with UTC it must be
    corrected by Δf/f₀ = - 4.46471409E-10

    In the GPS specification the correction is set to
    Δf/f₀ = - 4.4647E-10


    --
    Paul

    https://paulba.no/

    --- SoupGate-Win32 v1.05
    * Origin: fsxNet Usenet Gateway (21:1/5)
  • From Paul B. Andersen@21:1/5 to All on Thu Jan 11 22:18:53 2024
    Den 10.01.2024 22:38, skrev Laurence Clark Crossen:
    On Wednesday, January 10, 2024 at 1:19:04 AM UTC-8, Paul B. Andersen wrote:
    Den 09.01.2024 22:29, skrev Laurence Clark Crossen:

    Relativity is an illogical theory, so it does not make predictions. As Essen, Rutherford, and Soddy said, it is not even a scientific theory. Relativity is thoroughly irrational.

    The idea that a postulate does not explain is ridiculous.

    What is the relativity mechanism or cause of the difference in clock rates? If it is gravity, then Newton has a prediction.

    Claiming the Newton mode would be no adjustment at all is so flagrantly false as to be a deceitful obfuscatory tactic.

    I acknowledge the accurate prediction was made and deny it could have had anything to do with relativity contrary to James A. Buisson, Roger L. Easton, Thomas B. McCaskill
    U. S. Naval Research Laboratory (NRL). How did they really predict it?


    It's frustrating that a theory which Laurence Clark Crossen
    claims is illogical and can't make unambiguous predictions,
    can make precise correct predictions for how clocks behave in
    the gravitational field in Earth's vicinity, isn't it? :-D

    Must be necromancy, don't you think?

    BTW, here are more experiments which confirm SR/GR :
    https://paulba.no/paper/Kennedy_Thorndike.pdf
    https://paulba.no/paper/Ives_Stilwell.pdf
    https://paulba.no/paper/Ives_Stilwell_II.pdf
    https://paulba.no/paper/Clemence.pdf
    https://paulba.no/paper/Babcock_Bergman.pdf
    https://paulba.no/paper/Frisch_Smith.pdf
    https://paulba.no/paper/Alvager_et_al.pdf
    https://paulba.no/paper/Beckmann_Mandics.pdf
    https://paulba.no/paper/Filippas_Fox.pdf
    https://paulba.no/paper/Shapiro_1964.pdf
    https://paulba.no/paper/Shapiro_1968.pdf
    https://paulba.no/paper/Brecher.pdf
    https://paulba.no/paper/Brillet_Hall.pdf
    https://paulba.no/paper/PPN_gamma_Hipparcos.pdf
    https://paulba.no/paper/PPN_gamma_Cassini.pdf
    https://paulba.no/paper/Shapiro_2004.pdf
    https://paulba.no/paper/Liu.pdf
    https://paulba.no/paper/GravDeflection.pdf
    https://paulba.no/paper/Remmen_McCreary.pdf
    https://paulba.no/paper/Botermann.pdf
    https://paulba.no/paper/LIGO_GravitationalWaves.pdf
    https://paulba.no/paper/LIGO_TestOfGR.pdf
    https://paulba.no/paper/LIGO_GravitationalWaves_2.pdf

    They will all go away if you ignore them! :-D



    You cannot defeat my argument by insistently asserting that experiments have proved an illogical theory that never made any unambiguous predictions. You can only do so by demonstrating the logical character of your theory, which you only assert.
    Countless excellent scientists have shown it is totally illogical.

    Experimental evidence trumps your opinion and belief.
    GR is thoroughly confirmed and never falsified.


    HOW CAN RELATIVITY PREDICT DOUBLE NEWTONIAN FOR THE ECLIPSE AND NEWTONIAN FOR POUND & REBKA? Then, the predictions are as ambiguous as those of a necromancer.

    You are rather funny in your naivety!
    Do you really believe that your failure to understand
    how GR can predict the gravitational deflection of light
    means that GR can't predict it? :-D

    You can see the predictions here: https://paulba.no/pdf/GravitationalDeflection.pdf
    Can you find an ambiguous prediction?

    A simulation:
    https://paulba.no/Deflection.html


    Interpretations of experiments are not facts. Failure to question factoids is imbecilic.

    In that case you can point out a wrong interpretations of
    the experiments above. Or can't you?

    --
    Paul

    https://paulba.no/

    --- SoupGate-Win32 v1.05
    * Origin: fsxNet Usenet Gateway (21:1/5)
  • From Volney@21:1/5 to Laurence Clark Crossen on Thu Jan 11 18:23:02 2024
    On 1/11/2024 4:42 PM, Laurence Clark Crossen wrote:
    On Wednesday, January 10, 2024 at 4:28:05 PM UTC-8, Volney wrote:
    On 1/10/2024 4:35 PM, Laurence Clark Crossen wrote:

    As Lou said, "> You forgot. The time doesn’t change. The atoms resonant frequency beats faster at higher altitudes." - That's why it is set to a lower frequency.

    And, obviously, Lou is wrong. As are you.

    So, you stand by your position that clocks in orbit are not affected by gravity? And, Newton mode is when you pretend Newtonian can't explain the different rate of the clocks in space because that is time dilation and not instrumental error as when a
    pendulum clock is used in space?

    Clocks in orbit are affected by relativity's Schwarzschild metric,
    GM/rc². Gravity is also an effect of general relativity. So the answer
    really is that clocks in orbit aren't affected by gravity since both the
    clock rate and gravity itself are effects of GR.

    --- SoupGate-Win32 v1.05
    * Origin: fsxNet Usenet Gateway (21:1/5)
  • From Paul B. Andersen@21:1/5 to All on Fri Jan 12 13:58:52 2024
    Den 11.01.2024 22:23, skrev Laurence Clark Crossen:
    It is amusing that you exhibit no comprehension of the point plainly stated: How can relativity predict twice Newtonian in the eclipse and once Newtonian in Pound & Rebka if it makes an unambiguous prediction? The experimental results are contradictory.
    Then, do they verify Newton or Einstein? Why don't you just relinquish P&R?

    Amusing indeed. This is even hilarious! :-D

    --
    Paul

    https://paulba.no/

    --- SoupGate-Win32 v1.05
    * Origin: fsxNet Usenet Gateway (21:1/5)
  • From Paul B. Andersen@21:1/5 to All on Fri Jan 12 13:44:39 2024
    Den 11.01.2024 23:48, skrev Lou:

    Ignore Paul. Like his peers, Paul is a Pathological liar.
    I’m sure I’ve even pointed out to him in the past year that the original much cited
    papers on the Cassini Shapiro delay...admitted that not only did they NOT check
    if classical refraction was or was not possible. The papers
    authors admitted they couldn’t have because the the data to prove the
    myth was at best incomplete and therefore could not rule out refraction even if They had tried to in their paper. Which they didn’t.

    I suppose the "myth" the authors allegedly admitted not
    to be able to defend was GR's prediction of gravitational
    deflection of light.

    https://paulba.no/paper/PPN_gamma_Cassini.pdf
    "In terms of the standard post-Newtonian parameter γ,
    we find that γ − 1 = (−1.3 ± 5.2) × 10⁻⁵, in agreement
    with the theory of General Relativity."

    https://paulba.no/paper/PPN_gamma_Cassini_2.pdf
    "In terms of the standard post-Newtonian parameter γ,
    the result is γ−1 = (−4.8±5.7) × 10⁻⁵, including both
    random and systematic error. Einstein’s theory has
    survived yet another test."

    --
    Paul

    https://paulba.no/

    --- SoupGate-Win32 v1.05
    * Origin: fsxNet Usenet Gateway (21:1/5)
  • From Volney@21:1/5 to Clas Rigatos Demetrious on Fri Jan 12 10:38:22 2024
    On 1/12/2024 8:03 AM, Clas Rigatos Demetrious wrote:
    Volney wrote:

    On 1/11/2024 7:11 AM, Lou wrote: Really?

    5.2716726e-10 is not the correct answer? It looks exactly the same as
    the prediction from GR. Are 𝘆𝗼𝘂 𝗳𝗶𝗻𝗮𝗹𝗹𝘆 𝗮𝗱𝗺𝗶𝘁𝘁𝗶𝗻𝗴 𝗚𝗥 𝗶𝘀 𝘄𝗿𝗼𝗻𝗴?
    GM/r -earth) - (GM/r sat) ÷ f Using r= 6371000 and f= 8.9875518e+16:

    Again,

    eW91ciBPUklHSU5BTCBjbGFpbSwgR00vKHItcicpIGlzIHdyb25nISBQYXVsIGNhdWdodCB5b3UsIGFuZCBpbg==
    your reply to him,
    IHlvdSBjb3JyZWN0ZWQgeW91ciBtaXN0YWtlIGJ5IHdyaXRpbmcgKEdNL3IpLShHTS9y4oCZKS4=
    I am laughing at your ORIGINAL mistake!
    U2xvcHB5LCBzbG9wcHksIHNsb3BweQ==!!

    Nymshifter, your garbage posting software is br0ken again.

    --- SoupGate-Win32 v1.05
    * Origin: fsxNet Usenet Gateway (21:1/5)
  • From Volney@21:1/5 to Prokaryotic Capase Homolog on Fri Jan 12 19:42:59 2024
    On 1/12/2024 6:56 AM, Prokaryotic Capase Homolog wrote:
    On Wednesday, January 10, 2024 at 4:21:28 AM UTC-6, Lou wrote:
    On Tuesday 9 January 2024 at 18:50:08 UTC, Paul B. Andersen wrote:

    It is not only Cs with frequency 9192631770 Hz that is used,
    it is also Rubidium with frequency 6834682610.904 Hz.
    So the frequency synthesiser has to be different if it is
    Rb oscillator than a Cs oscillator.

    https://www.electricity-magnetism.org/frequency-synthesizers/

    Note that in a frequency synthesiser the output frequency
    is N/M x reference_frequency where N and M are integers.
    I didn’t see much at that link of any help. But it’s an interesting
    formula you mention above. But I don’t understand the terms in it.
    So can I get a clarification . What is ‘reference’ frequency in your
    N/M x ref frequency. Is that 9192631770?
    And what integers are N and M ?

    Paul presents a frequency synthesizer diagram that would have
    been somewhat beyond state-of-the-art in the 1970s, when the
    first GPS satellites were flown. In particular, the N and M
    counters were assembled using 7400 series TTL logic hard-wired on
    printed circuit boards and were absolutely *NOT* programmable.

    Likely 5400 series, but these are just military spec 7400 series with
    the same numbers except the 5 vs 7.

    TTL logic is not capable of handling gigahertz frequencies. The
    output frequencies of the cesium or rubidium atomic frequency
    standards first needed to be brought down using high-speed ECL
    prescalers to something below, say, 100 MHz. So let us assume a
    128x prescaler. The input frequency from a CAFS would therefore
    be 71.817435703125 MHz, while the input frequency from a RAFS
    would be 53.3959578976875 MHz.

    I'm not sure what the high speed prescaler logic would be but ECL sounds correct, being of that time period.

    If we assume 24 bit counters, then to get from 71.817435703125 MHz
    to 10.23 MHz you need N=2104729 and M=14775781

    To get from 71.817435703125 MHz to 10.2299999954326 MHz you need
    N=1660979 and M=11660533

    To get from 53.3959578976875 MHz to 10.23 MHz you need N=705947
    and M=3684723

    To get from 53.3959578976875 MHz to 10.2299999954326 MHz you need
    N=2784179 and M=14532151

    Simple?

    I was recently looking at how such synthesizers work. One of the two
    dividers divides down the fixed Cs or Rb clock output while the other
    divides down the output of a voltage controlled oscillator. The two
    outputs are fed into a phase locked loop circuit which controls the
    voltage controlled oscillator.

    --- SoupGate-Win32 v1.05
    * Origin: fsxNet Usenet Gateway (21:1/5)
  • From Volney@21:1/5 to Laurence Clark Crossen on Sat Jan 13 02:15:53 2024
    On 1/12/2024 4:37 PM, Laurence Clark Crossen wrote:
    On Thursday, January 11, 2024 at 3:23:07 PM UTC-8, Volney wrote:

    Clocks in orbit are affected by relativity's Schwarzschild metric,
    GM/rc². Gravity is also an effect of general relativity. So the answer
    really is that clocks in orbit aren't affected by gravity since both the
    clock rate and gravity itself are effects of GR.

    I thought GR was measuring the gravitational effects.

    GR is mass warping spacetime. In GR gravity is not a force, it is the
    effect of objects following geodesics in warped spacetime which look
    curved so look like a force to us.

    You really should learn GR before declaring it wrong.

    --- SoupGate-Win32 v1.05
    * Origin: fsxNet Usenet Gateway (21:1/5)
  • From Paul B. Andersen@21:1/5 to All on Sat Jan 13 13:15:16 2024
    Den 13.01.2024 05:19, skrev Laurence Clark Crossen:
    On Friday, January 12, 2024 at 7:20:54 PM UTC-8, Prokaryotic Capase Homolog wrote:
    On Friday 12 January 2024 at 22:56:08 UTC, Laurence Clark Crossen wrote: >>>> On Friday, January 12, 2024 at 4:55:24 AM UTC-8, Paul B. Andersen wrote: >>>> R. V. Pound and J. L. Snider, "Effect of Gravity on Gamma Radiation": "It is not our purpose here to enter into the many-sided discussion of the relationship between the effect under study and general relativity or energy conservation. It is to be
    noted that no strictly relativistic concepts are involved and the description of the effect as an "apparent weight" of photons is suggestive. The velocity difference predicted is identical to that which a material object would acquire in free fall for a
    time equal to the time of flight. [...]"

    Both you and Lau show profound misunderstandings
    and ignorance of what you've read. In addition to Paul's
    writings, I recommend that you read the Wikipedia article
    on the Pound-Rebka experiment
    https://en.wikipedia.org/wiki/Pound%E2%80%93Rebka_experiment
    of which my authorship stands at 86%:
    https://xtools.wmcloud.org/articleinfo/en.wikipedia.org/Pound%E2%80%93Rebka_experiment#tool-authorship

    The fact is the quote cannot be misunderstood and plainly states the effects of gravity on photons are the same as every other object, so since you cannot concede this you have profoundly misunderstood, no doubt because you foolishly accept relativity.

    Right.
    The effects of gravity on photons are the same
    as every other object.

    The gravitational (coordinate) acceleration does not depend
    on the mass of the object, it only depends on its speed.
    If the speed is low, like for satellites orbiting the Earth,
    relativity and Newton will predict the same deflection
    by Earth's gravitation and thus predict the same orbit.
    But if the speed of a test particle with mass is very close
    to c, the deflection will be the same as for a photon,
    which is twice of what Newton predicts.

    Note that in deflection of a photon the acceleration
    is transverse to the velocity, so the speed does not change.
    The same is the case for a test particle with speed
    very close to c, its speed does not change.

    If the object's initial velocity is radially, then GR
    predicts that the speed will increase and approach
    asymptotically to c, while Newton predicts that the speed
    will keep increasing indefinitely (to it hits Earth).
    If the initial speed is c (photon), GR predicts that
    that the speed will be constant c, and the increased
    kinetic energy will be stored in the increased frequency
    of the photon.
    Newton predicts that the speed will increase beyond c,
    and the kinetic energy will be mv²/2.

    The speed of gamma particles can be measured.
    It is always c as predicted by GR, never
    beyond c as predicted by Newton.

    -------------

    I have two simulations illustrating this point.
    Used in both simulations is the post-Newtonian approximation
    of the gravitational acceleration predicted by GR.

    Se equation (2) in:
    https://paulba.no/pdf/GravitationalDeflection.pdf

    The simulation:
    https://paulba.no/Satellites.html
    Simulates the orbits of several satellites,
    among them are Geostationary, GPS, GLONASS and Galileo.
    The point in this context is that GR predicts the same
    orbits as Newton. (I have tried.)
    So with the low speed of the planets, GR an Newton predicts
    the same "deflection" of the velocity of the planets.

    The simulation:
    https://paulba.no/Deflection.html
    Simulates the deflection of light from a star by the Sun
    for any angle star-Sun as observed from the Earth.
    The photon is treated as a test particle with initial speed c.
    In this case, when the speed is c, GR gives exactly twice
    the deflection of Newton.

    --
    Paul

    https://paulba.no/

    --- SoupGate-Win32 v1.05
    * Origin: fsxNet Usenet Gateway (21:1/5)
  • From Volney@21:1/5 to Lou on Sat Jan 13 16:04:28 2024
    On 1/10/2024 8:03 PM, Lou wrote:
    On Thursday 11 January 2024 at 00:16:04 UTC, Volney wrote:
    On 1/10/2024 4:01 PM, Lou wrote:
    On Wednesday 10 January 2024 at 17:35:14 UTC, Volney wrote:
    On 1/9/2024 10:08 AM, Lou wrote:
    On Tuesday 9 January 2024 at 14:42:58 UTC, Volney wrote:

    Umm, no. Newtonian time is the same everywhere. Remember the Galilean >>>>>> transform where t' = t?

    Yes. Classical physics has time same everywhere. But it explains
    clock gains with resonance using GM/r.
    Your second sentence contradicts the first. If time was the same
    everywhere, then there will be no clock gain, because, well, the time is >>>> the same everywhere! t'=t.

    You forgot. A Classical model doesn’t have time change.
    I assume you mean Newtonian physics doesn't because classical GR
    certainly does.

    Classical GR? Since when does classical physics use imaginary spacetime
    and time travelling twins!! :D

    No, classical GR uses real spacetime. Time traveling twins is a figment
    of your imagination.

    It
    has harmonic oscillators resonating at higher frequencies at
    lower gravitational fields.
    Except that would be proportional to the gravitational force or
    proportional to 1/r² while the weak field time dilation is proportional
    to 1/r. As you have been told REPEATEDLY. Did you forget or are you
    really stooopid?

    Show me the classical (non relativistic ) formula for modelling
    resonant frequencies at different altitudes that doesn’t use r.

    Sorry, but anything that involves gravitational force such as your
    defective clock components that depends on variable "resonances" involve effects proportional to the force, or proportional to 1/r²

    As observed.

    Observed to be proportional to 1/r, esp. as other sat navigation systems
    in different orbits (GALILEO, GLONASS, BEIDOU etc.) show.

    Exactly. And modelled proportional to r to a great accuracy by
    using the simple non relativistic formula (GM/r)-(GM/r’) ÷ f

    That formula, if you replace the incorrect "f" (some frequency) with c²
    is the general relativity Schwarzschild metric.

    Newton didn’t. But classical physics does. And it works.

    Newtonian physics only uses it for potential energy per unit mass.

    Albeit with error margins of 0.00021 vs Relativity’s 0.0001.

    Obviously misapplied Newtonian physics is wrong.

    After all c^2 is just a number
    No, it is not. It has dimensionality of length²/time².

    What SI unit is it?

    Can't you read? Length²/time² or in SI, meters²/seconds².

    So meters²/seconds² is what in the formula..acceleration?

    No, acceleration is meters/seconds².

    You forgot. The time doesn’t change. The atoms resonant frequency beats >>>>> faster at higher altitudes.

    No, the satellite clocks are designed to tick at the correct rate and
    not to be affected by external events. That's why they are so accurate, >>>> they can't be affected by external things like a ground level Cs clock >>>> being affected by the gravity of a fat scientist walking by it. Besides, >>>> any "resonant frequency" effects would be proportional to force, which >>>> varies as 1/r², but the altitude effect varies at 1/r. And the clocks >>>> are in freefall, so there is no force or 1/r² effect on them. Newtonian >>>> physics nothing is affected proportional to 1/r other than the potential >>>> energy of a mass.

    A clock at the top of Everest also has clock gains under classical theory using
    GM/r. Yet it’s not in free fall.
    So it has a force on it, yet it still works correctly, showing how a
    properly operating clock works correctly in freefall or not in freefall.
    The Schwarzschild metric still applies, of course.

    Yes and it also resonates at a higher frequency. And predicted as well
    by the classical resonance formula (GM/r)-(GM/r’) ÷ f.

    That is a miswritten version of relativity's Schwarzschild metric,
    GM/rc². Again, congratulations for stumbling across the metric, but Schwarzschild beat you to it by more than 100 years.

    Although you’ll have to be careful when you ask them. They get
    really touchy when you point out that their “spacey metric” term or whatever
    they call it is actually just force of gravity by another name.

    And again, I must conclude you are really, really stooopid for repeating
    that despite repeated correction. I'll mention YET AGAIN that the
    Schwarzschild metric varies as 1/r while force/acceleration of gravity
    varies as 1/r².

    Anyways the fact is harmonic oscillators

    ...which would vary as 1/r²...

    have been observed to resonate at
    higher frequencies when less force is exerted on them

    Some higher and some lower. A properly designed clock will not have
    anything that varies with gravitational force (excluding pendulum clocks
    which use gravity as part of their operation).

    and this effect
    has been well understood for centuries I believe.

    Which is why scientists designed such effects out of clocks as best they
    can. As well as effects of thermal expansion, friction etc. that have to
    be eliminated in accurate clocks.

    --- SoupGate-Win32 v1.05
    * Origin: fsxNet Usenet Gateway (21:1/5)
  • From Volney@21:1/5 to Lou on Sat Jan 13 18:18:47 2024
    On 1/13/2024 4:46 PM, Lou wrote:
    On Saturday 13 January 2024 at 21:04:33 UTC, Volney wrote:
    On 1/10/2024 8:03 PM, Lou wrote:

    Show me the classical (non relativistic ) formula for modelling
    resonant frequencies at different altitudes that doesn’t use r.

    Sorry, but anything that involves gravitational force such as your
    defective clock components that depends on variable "resonances" involve
    effects proportional to the force, or proportional to 1/r²


    But GR involves gravitational force

    No, GR doesn't even have a gravitational force, except as a pseudoforce
    (like centrifugal force). GR in its weak limit approximation will
    calculate the strength of the pseudoforce of gravity.

    and only uses r.

    It uses 1/r² in the weak field limit for gravitational 'force' and acceleration. As explained to you REPEATEDLY.

    Oh I forgot...we’re not supposed to point out that Einstein pretended gravity didn’t exist and called gravity by a different name...
    spacetime or metric or magic goblins or whatever

    Good. So you learned that in GR gravity isn't a real force.

    Smart move. That way you can use r to model gravity effects. And pretend
    you weren’t using r to model gravity effects.

    Again, to calculate the pseudoforce, it goes as 1/r², not 1/r.

    As observed.

    Observed to be proportional to 1/r, esp. as other sat navigation systems >>>> in different orbits (GALILEO, GLONASS, BEIDOU etc.) show.

    Exactly. And modelled proportional to r to a great accuracy by
    using the simple non relativistic formula (GM/r)-(GM/r’) ÷ f

    That formula, if you replace the incorrect "f" (some frequency) with c²
    is the general relativity Schwarzschild metric.

    Amazing isn’t it. A simple formula anyone can work out.

    Just because it's GR doesn't mean everything is complicated. But
    remember that's the weak field limit.

    And you don’t need to know a jot of relativity to “stumble” across it. I found it by simply using GM / r and calculating it for every radius including 4.12. Put them into a table of data and noticed that the
    larger r was... the smaller GM/r was. 1/2,1,3,1/4 etc.

    Doesn't take a math genius to 'discover' that.

    All I did then was experiment and first divide into 9192631770.
    Then 10.23Mhz and finally c^2.

    Newton didn’t. But classical physics does.

    No, it doesn't. Newtonian physics has no use for the Schwarzschild
    metric. It only uses gravitational potential to calculate the potential
    energy per unit mass.

    That is a miswritten version of relativity's Schwarzschild metric,
    GM/rc². Again, congratulations for stumbling across the metric, but
    Schwarzschild beat you to it by more than 100 years.

    No he didn’t. His was GM/rc^2. And it doesn’t give 5.27e-10
    Nothing like it.

    Did you do ALL the calculations? The SR time dilation of the satellite?
    The potential of both the geoid surface and the satellite altitude?
    (and remember not to repeat your mistake: it is NOT GM/(r-rs)!

    You relativist made up your latest “correct “ formula to try to
    retrodict relativity into the 442 ms/day observed by the first GpS test.
    You had no idea before 1977 what the clock gain would be for r4.12
    ...until the first GPS told you how much.

    And yet again, how do you explain that the satellite was launched with
    the correct value built into its switchable frequency synthesizer, and
    managed to get it right? And anyone can use the GR formulas for the GPS
    (and BEIDOU, and GLONASS, and GALILEO) satellites and come up with the
    same answer?

    Remember, the only choice available for NTS-2 was zero time offset or
    38µS/day offset. Do you really think they guessed at this number?

    Although you’ll have to be careful when you ask them. They get
    really touchy when you point out that their “spacey metric” term or whatever
    they call it is actually just force of gravity by another name.

    And again, I must conclude you are really, really stooopid for repeating
    that despite repeated correction. I'll mention YET AGAIN that the
    Schwarzschild metric varies as 1/r while force/acceleration of gravity
    varies as 1/r².

    Anyways the fact is harmonic oscillators

    ...which would vary as 1/r²...

    Incorrect. GPS has proved that harmonic oscillators will resonate
    at different frequencies relative to r,

    Nope. They'd vary with force. 1/r².

    Besides, Rb and Cs clocks like used in the satellites don't even have
    any sort of resonant frequency (other than 9192631770 Hz for Cs clocks, different number for Rb clocks). And they don't depend on force, they
    are a fixed constant.

    --- SoupGate-Win32 v1.05
    * Origin: fsxNet Usenet Gateway (21:1/5)
  • From Volney@21:1/5 to RichD on Sat Jan 13 22:41:47 2024
    On 1/13/2024 5:23 PM, RichD wrote:
    On January 13, Prokaryotic Capase Homolog wrote:
    Let us suppose that, in a given time period, 1000000000000 waves
    are emitted by an EM source on a high tower, and 1000000000001
    waves are received on the ground.
    How does Doppler shifting explain where the extra wave come from?

    The Pound Rebka experiment counted individual wave crests,
    at 10 ^ 19 Hz, to 12 digits of precision?

    --
    Rich

    The Fe-57 atoms used in Pound-Rebka have a very, Very, VERY narrow
    frequency absorption band, and yes, it has to be correct to about 1 part
    in 10^12 or so. This is the same frequency it emits. The experiment uses
    this to detect how GR blueshift/redshift and Doppler shift affects the absorption.

    --- SoupGate-Win32 v1.05
    * Origin: fsxNet Usenet Gateway (21:1/5)
  • From Volney@21:1/5 to Lou on Sun Jan 14 18:46:02 2024
    On 1/14/2024 9:21 AM, Lou wrote:
    On Sunday 14 January 2024 at 03:41:54 UTC, Volney wrote:
    On 1/13/2024 5:23 PM, RichD wrote:
    On January 13, Prokaryotic Capase Homolog wrote:
    Let us suppose that, in a given time period, 1000000000000 waves
    are emitted by an EM source on a high tower, and 1000000000001
    waves are received on the ground.
    How does Doppler shifting explain where the extra wave come from?

    The Pound Rebka experiment counted individual wave crests,
    at 10 ^ 19 Hz, to 12 digits of precision?

    --
    Rich

    The Fe-57 atoms used in Pound-Rebka have a very, Very, VERY narrow
    frequency absorption band, and yes, it has to be correct to about 1 part
    in 10^12 or so. This is the same frequency it emits. The experiment uses
    this to detect how GR blueshift/redshift and Doppler shift affects the
    absorption.

    Ignore Volney. He’s trying to pretend that a narrow frequency absorption band
    means that somehow a very small frequency offset from classical Doppler shifting cannot explain the data. Talk about ridiculous logic based on fantasy
    assumptions backed by zero science.
    Although this 1+1=3 logic from relativists is predictable.

    I see you can't follow along with the experiment. If the Doppler shift
    is exactly equal and opposite to the gravitational blueshift, the
    frequency will be exactly correct to be absorbed. That way they measure
    the blueshift, by the counteraction of the Doppler redshift.

    --- SoupGate-Win32 v1.05
    * Origin: fsxNet Usenet Gateway (21:1/5)
  • From Paul B. Andersen@21:1/5 to All on Mon Jan 15 22:08:39 2024
    Den 15.01.2024 15:27, skrev Lou:

    Proke’s nonsensical pseudoscientific editing of wiki physics pages is a good example of how facism can take many guises.

    You arguments are getting better and better, Lou! :-D

    Keep it up, the sky is the limit!

    --
    Paul

    https://paulba.no/

    --- SoupGate-Win32 v1.05
    * Origin: fsxNet Usenet Gateway (21:1/5)
  • From Paul B. Andersen@21:1/5 to Homolog on Mon Jan 15 22:00:29 2024
    Den 15.01.2024 12:18, skrev Lou:
    On Monday 15 January 2024 at 10:46:32 UTC, Prokaryotic Capase Homolog wrote:
    On Saturday, January 13, 2024 at 7:05:57 AM UTC-8, Prokaryotic Capase Homolog wrote:
    Let us suppose that, in a given time period, 1000000000000 waves
    are emitted by an EM source on a high tower, and 1000000000001
    waves are received on the ground.


    The thought experiment was defined a bit more precisely here:

    On Saturday 13 January 2024 at 23:36:42 UTC, Prokaryotic Capase
    Homolog wrote:

    I was describing a situation where source and observer
    are a fixed distance apart.
    After one second, source emits 1000000000000 waves and
    the observer receives 1000000000001 waves.

    After 10 seconds, source emits 10000000000000 waves and
    the observer receives 10000000000010 waves.

    After 100 seconds, source emits 100000000000000 waves and
    the observer receives 100000000000100 waves.

    ...and so on and so forth.

    The height of the tower is h ≈ 95.73 m


    It only occurred to me after I posted to you last time that in your gedanken the reason why you had an extra imaginary wave seemingly to appear magically out of nowhere.

    There is no "imaginary wave seemingly to appear
    magically out of nowhere".
    The point is that both the source and the observer
    measure time with their own, local clocks.
    Loosely expressed:
    The observer's second is longer than the source's second,
    so the 'extra wave' received by the observer is simply the first
    wave in the source's next second.

    Wasn’t you trying to prove Doppler shifting in a classical
    model created extra imaginary waves. I realised that your extra imaginary magic wave comes from...Theoretical gravitational blueshift!
    But you forgot what I have explained numerous times on this thread.
    One can explain this apparent blueshifting without relativity and without adding magic relativistic waves. Simply by using atomic resonance.

    Even if your explanation for the different rate of the clocks
    was right (which it isn't), the result would be the same as
    described above. So why do you call it "extra theoretical waves"
    and "theoretical blue shift"? All the waves are real, and
    the gravitational blue shift is real.

    The received frequency is really higher than the emitted
    frequency even if all the emitted cycles are received.

    And note: If the sender in the source is switched on and off
    so that a limited number of cycles are emitted, then
    the number of received cycles is always equal to the number
    of emitted cycles.

    To illustrate this let's make a thought experiment
    taken from the real world:

    At equator at longitude 0⁰ is a clock A.
    In a geostationary satellite at longitude 0⁰ is a clock B.
    This is equivalent to a 35796724 m high tower.

    The period of one rotation of the Earth is one sidereal day.

    Clock A will measure each rotation of the Earth to last:
    τ₀ = 86164.09050000000 s (this is one sidereal day measured with UTC) Clock B will measure each rotation of the Earth to last a bit longer:
    τ₁ = τ₀⋅(1+5.3915E-10) = 86164.09054645538 s

    The geostationary satellite transmits a radio carrier with frequency:
    f₁ = 10.0000000000000 GHz
    Clock A receives the blue shifted signal:
    f₀ = f₁⋅(1+5.3915E-10) = 10.0000000053915 GHz

    Please answer:
    How many cycles are emitted from B during one rotation of the Earth?
    How many cycles are received by A during one rotation of the Earth?


    --
    Paul

    https://paulba.no/

    --- SoupGate-Win32 v1.05
    * Origin: fsxNet Usenet Gateway (21:1/5)
  • From Volney@21:1/5 to Laurence Clark Crossen on Tue Jan 16 00:15:27 2024
    On 1/14/2024 10:36 PM, Laurence Clark Crossen wrote:
    On Friday, January 12, 2024 at 11:15:57 PM UTC-8, Volney wrote:
    On 1/12/2024 4:37 PM, Laurence Clark Crossen wrote:
    On Thursday, January 11, 2024 at 3:23:07 PM UTC-8, Volney wrote:

    Clocks in orbit are affected by relativity's Schwarzschild metric,
    GM/rc². Gravity is also an effect of general relativity. So the answer >>>> really is that clocks in orbit aren't affected by gravity since both the >>>> clock rate and gravity itself are effects of GR.

    I thought GR was measuring the gravitational effects.

    GR is mass warping spacetime. In GR gravity is not a force, it is the
    effect of objects following geodesics in warped spacetime which look
    curved so look like a force to us.

    You really should learn GR before declaring it wrong.

    Volney, have you heard of disagreeing? I obviously know that yet I still disagree. You are just one of the faithful misled by the pseudoscience of relativity. Relativity doesn't cause anything and everyone here has been trying to help you understand
    that.

    Again, you keep arguing arguments which you have been told are wrong
    repeatedly yet you keep trying to use them. Again, learn GR before
    declaring it wrong, a 'pseudoscience' (crackpot sign) or a cult/religion (another obvious crackpot sign).

    --- SoupGate-Win32 v1.05
    * Origin: fsxNet Usenet Gateway (21:1/5)
  • From Paul B. Andersen@21:1/5 to All on Tue Jan 16 13:34:26 2024
    Den 15.01.2024 22:48, skrev Lou:
    On Monday 15 January 2024 at 20:56:54 UTC, Paul B. Andersen wrote:
    Let's make a thought experiment
    taken from the real world:

    Please read the following again.
    There are a lot of geostationary satellites orbiting the Earth,
    and everything I write below does happen in the real world.

    We KNOW how clocks behave in the vicinity of the Earth.
    It is in accordance with the predictions of GR, but this
    may be by coincidence, so maybe you have the right theory. :-J

    But in either case, THIS IS HOW THEY BEHAVE!


    At equator at longitude 0⁰ is a clock A.
    In a geostationary satellite at longitude 0⁰ is a clock B.
    This is equivalent to a 35796724 m high tower.

    The period of one rotation of the Earth is one sidereal day.

    Clock A will measure each rotation of the Earth to last:
    τ₀ = 86164.09050000000 s (this is one sidereal day measured with UTC)
    Clock B will measure each rotation of the Earth to last a bit longer:
    τ₁ = τ₀⋅(1+5.3915E-10) = 86164.09054645538 s

    The geostationary satellite transmits a radio carrier with frequency:
    f₁ = 10.0000000000000 GHz
    Clock A receives the blue shifted signal:
    f₀ = f₁⋅(1+5.3915E-10) = 10.0000000053915 GHz

    Please answer:


    !! Yes my divine leader, I was right. You guys are all pushy fascists.

    Right.
    Thou shall not have any other leader than Us!
    Be humble and show respect!

    Anyways...Obviously you have the answer. So tell us what exactly does your math
    question prove?

    There is no GR math above.

    We know that clocks in geostationary satellites run
    fast relative to UTC by the factor (1 + 5.3915E-10).
    Isn't that what your theory predicts?
    You know, the gravitational force at altitude 35796724 m
    is less than it is on the Earth, so the Cs atom vibrates
    faster because the pressure is less.

    It's very simple. The Earth rotates once a sidereal day.
    So clock A advances during one rotation:
    τ₀ = 86164.09050000000 s (this is one sidereal day measured with UTC)
    Clock B will measure each rotation of the Earth to last a bit longer:
    τ₁ = τ₀⋅(1+5.3915E-10) = 86164.09054645538 s

    Of course you understand this, you are not stupid! Or are you?

    The geostationary satellite transmits a carrier with frequency
    f₁ = 10 GHz.
    During one rotation of the Earth the number of cycles emitted is:
    N₁ = τ₁⋅f₁ = 86164.09054645538s⋅10E9 Hz = 861640905464553.8 cycles

    The observer on the ground will obviously receive all these cycles
    during one rotation of the Earth, which last a time τ₀.
    So the frequency received at the ground MUST be:
    f₀ = N₁/τ₀ = 861640905464553.8/86164.09050000000 = 10.0000000053915 GHz f₀ = N₁/τ₀ = τ₁⋅f₁/τ₀ = (τ₁/τ₀)⋅f₁ = (1+5.3915E-10)⋅f₁

    Since you not are stupid, you will now understand that
    a logically inevitable consequence of the fact that
    clock B runs fast by (1 + δ) relative to UTC (and clock A) is
    that the gravitational blue shift is the same factor (1 + δ).

    Or don't you understand it?

    That the satelitte is being vibrated back and forth like
    the source in pound Rebka? We were talking about Pound Rebka werent we?

    You are not an idiot, so why are you pretending to be?
    Or are you not pretending?

    How many cycles are emitted from B during one rotation of the Earth?
    How many cycles are received by A during one rotation of the Earth?

    Let me guess..umm ....
    ...sixteen SR goblins times 141/2 GR dimensions = 105684e+696 cycles?
    Or do you need the answer in dozens of spaced out metrics?
    How close was I ?
    And more importantly what’s your point?

    But now you know the answer, don't you?

    This is my point:
    Since you correctly claim that clocks at high altitude run
    fast by some factor (1 + δ) relative to clocks on the ground,
    you know that the gravitational blue shift must be the same
    factor (1 + δ).

    Or don't you know that?

    --
    Paul

    https://paulba.no/

    --- SoupGate-Win32 v1.05
    * Origin: fsxNet Usenet Gateway (21:1/5)
  • From Paul B. Andersen@21:1/5 to Prokaryotic Capase Homolog on Wed Jan 17 22:19:26 2024
    Den 16.01.2024 21:12, skrev Lou:
    On Tuesday 16 January 2024 at 12:30:49 UTC, Paul B. Andersen wrote:
    Den 15.01.2024 22:48, skrev Lou:
    On Monday 15 January 2024 at 20:56:54 UTC, Paul B. Andersen wrote:
    Let's make a thought experiment
    taken from the real world:

    At equator at longitude 0⁰ is a clock A.
    In a geostationary satellite at longitude 0⁰ is a clock B.
    This is equivalent to a 35796724 m high tower.

    The period of one rotation of the Earth is one sidereal day.

    Clock A will measure each rotation of the Earth to last:
    τ₀ = 86164.09050000000 s (this is one sidereal day measured with UTC) >>>> Clock B will measure each rotation of the Earth to last a bit longer:
    τ₁ = τ₀⋅(1+5.3915E-10) = 86164.09054645538 s

    The geostationary satellite transmits a radio carrier with frequency:
    f₁ = 10.0000000000000 GHz
    Clock A receives the blue shifted signal:
    f₀ = f₁⋅(1+5.3915E-10) = 10.0000000053915 GHz


    We know that clocks in geostationary satellites run
    fast relative to UTC by the factor (1 + 5.3915E-10).


    I’m not sure where you get 1 + 5.3915E-10. I get 5.27e-10

    I have shown you before how to calculate the rate
    of a satellite in circular orbit relative to UTC.

    dτ/dt_utc = (1 - 1.5⋅GM/r⋅c² + δutc)

    For a geostationary satellite we have:
    GM = 3.986004418E14 m³/s²
    c = 299792458 m/s
    p = 86164.0905 s (orbital period one sidereal day)
    r = cbrt(GM⋅p²/4π²) = 42164169.6241 m

    dτ/dt_utc = (1 + 5.391498E-10)

    It's very simple. The Earth rotates once a sidereal day.
    So clock A advances during one rotation:
    τ₀ = 86164.09050000000 s (this is one sidereal day measured with UTC)
    Clock B will measure each rotation of the Earth to last a bit longer:
    τ₁ = τ₀⋅(1+5.3915E-10) = 86164.09054645538 s
    Of course you understand this, you are not stupid! Or are you?

    I’m not doing the maths here so I’ll take your word on the numbers

    (I’m assuming you are ignoring the velocity component. )

    Of course I don't ignore the velocity component.
    A satellite in circular orbit is MOVING in the ECI-frame.

    After all if B ticks faster than A then it will get more ticks
    per day. But you ignore an important fact. In a *classical model*
    Both days are the same length in time . But clock B divides
    24 hours into more ticks per day.

    Good grief! :-D

    Please answer these questions:
    You are in a spaceship with no windows. You don't
    know if you are in orbit, or is somewhere in outer space.
    All you know is that you are weightless.
    You have an atomic clock. You don't know if is a Cs, Rb or H-maser
    clock, all you know is that it ticks out seconds as defined by SI.

    Q #1. How many seconds would it be in 24 hours, as shown on that
    atomic clock?
    Q #2: How many seconds would it be in 24 hours, as shown on your
    wristwatch ?
    Q #3. You have a radio sender which according to your clock is
    sending at the frequency 10 GHz. How many cycles does it
    send during 24 hours?
    Q #4: How can you know how many seconds on your clock it is
    in 24 _real_ hours?

    But both still agree on the length of the day. Presumably that’s
    what a divisor does in an atomic clock.

    Of course they don't.

    Clock A measures the length of a sidereal day to be
    86164.09050000000 s

    Clock B measures the length of sidereal day to be
    86164.09054645538 s

    How can you say that they agree on the length of day?


    The geostationary satellite transmits a carrier with frequency
    f₁ = 10 GHz.
    During one rotation of the Earth the number of cycles emitted is:
    N₁ = τ₁⋅f₁ = 86164.09054645538s⋅10E9 Hz = 861640905464553.8 cycles

    The observer on the ground will obviously receive all these cycles
    during one rotation of the Earth, which last a time τ₀.


    Well technically it can’t because B isn’t in its line of site for 1/2 the day.
    Also there is Doppler shifting but I assume we ignore all that.

    You ARE confused, aren't you? :-D

    The radio signal doesn't come from the Sun, it comes from
    clock B which always is at the zenith as observed by A.


    So the frequency received at the ground MUST be:
    f₀ = N₁/τ₀ = 861640905464553.8/86164.09050000000 = 10.0000000053915 GHz
    f₀ = N₁/τ₀ = τ₁⋅f₁/τ₀ = (τ₁/τ₀)⋅f₁ = (1+5.3915E-10)⋅f₁


    527 actually, but it doesn’t matter. What your saying is A receives
    a higher frequency of ticks from B than A’s own clock under
    both models ignoring other effects.

    What are you trying to say?

    They are not ignoring anything.

    B is sending the frequency 10 GHz according to his own clock.
    A is receiving the frequency 10.0000000053915 GHz according to
    his own clock.

    That's all.


    Since you not are stupid, you will now understand that
    a logically inevitable consequence of the fact that
    clock B runs fast by (1 + δ) relative to UTC (and clock A) is
    that the gravitational blue shift is the same factor (1 + δ).


    I haven’t crunched your numbers but that sounds reasonable.
    After all both models predict the same magnitude but for
    different theoretical reasons.

    You don't have to crunch any numbers to UNDERSTAND
    that blue shift is an inevitable consequence of
    that clock B runs faster than clock A relative to UTC.


    This is my point:
    Since you correctly claim that clocks at high altitude run
    fast by some factor (1 + δ) relative to clocks on the ground,
    you know that the gravitational blue shift must be the same
    factor (1 + δ).

    If your calculations say they are the same and we ignore any other Effects like
    v or Doppler or oblateness then I’ll take your word for it.

    You wouldn't have do take my word for it if you had understood it!
    You obviously haven't.

    What’s your point?

    Good grief! Still not got it!!!

    Prokaryotic Capase Homolog wrote:
    | Given a situation where source and observer are separated
    | by a fixed distance:
    |
    | After one second, the source emits 1000000000000 waves and
    | the observer receives 1000000000001 waves. .

    The source emit the frequency 1000000000000 Hz
    the observer receives the frequency 1000000000001 Hz

    This is gravitational blue shift.

    To this, you commented:
    | It only occurred to me after I posted to you last time that in your
    gedanken
    | the reason why you had an extra imaginary wave seemingly to appear
    magically
    | out of nowhere.

    In other words, you think gravitational blue shift is impossible.

    So my point is still:
    A logically inevitable consequence of the fact that
    clock B runs fast by (1 + δ) relative to UTC (and clock A) is
    that the gravitational blue shift is the same factor (1 + δ).

    So it's no mystery and no ghost are involved.

    And since you correctly claim that clocks at high altitude run
    fast by some factor (1 + δ) relative to clocks on the ground,
    you know that the gravitational blue shift must be the same
    factor (1 + δ).

    But you still don't understand so you still don't know.
    Or do you?


    --
    Paul

    https://paulba.no/

    --- SoupGate-Win32 v1.05
    * Origin: fsxNet Usenet Gateway (21:1/5)
  • From Volney@21:1/5 to Lou on Wed Jan 17 23:55:39 2024
    On 1/17/2024 6:14 PM, Lou wrote:
    On Wednesday 17 January 2024 at 21:15:47 UTC, Paul B. Andersen wrote:
    Den 16.01.2024 21:12, skrev Lou:

    I haven’t crunched your numbers but that sounds reasonable.
    After all both models predict the same magnitude but for
    different theoretical reasons.

    You don't have to crunch any numbers to UNDERSTAND
    that blue shift is an inevitable consequence of
    that clock B runs faster than clock A relative to UTC.

    In relativity yes. But not in a classical model.
    Gravitational Blueshifting is a relativist assumption, but in a classical model Clock gains are modelled using resonance.

    But you just said that in a "classical model" (I'll assume you meant
    Newtonian model here, but see below) time lengths are the same.

    You forgot. In a classical model....Satelittes and ground clocks do not have different lengths of days.

    So how can there be any clock gains if the times are always the same?
    You contradict yourself!

    No gravitational
    blueshifting is needed. Both sat and ground clocks run at the same time.

    There, you said it again, the clocks run at the same rate, so clock
    gains are impossible.

    Make up your mind!

    I understand you clearly. You think that a classical model incorporates Relativity.
    It doesn’t.

    You are forgetting, relativity *is* a classical model! It's not quantum
    and well over 100 years old as well, so classical.

    --- SoupGate-Win32 v1.05
    * Origin: fsxNet Usenet Gateway (21:1/5)
  • From Paul B. Andersen@21:1/5 to All on Thu Jan 18 14:51:38 2024
    Den 18.01.2024 00:14, skrev Lou:
    On Wednesday 17 January 2024 at 21:15:47 UTC, Paul B. Andersen wrote:
    You don't have to crunch any numbers to UNDERSTAND
    that blue shift is an inevitable consequence of
    that clock B runs faster than clock A relative to UTC.

    In relativity yes. But not in a classical model.
    Gravitational Blueshifting is a relativist assumption, but in a classical model Clock gains are modelled using resonance. No gravitational
    blueshifting is needed. Both sat and ground clocks run at the same time.
    Just the resonant frequencies of the atoms varies with altitude.


    Since you don't even try to give a rational reason for why
    the consequence of the fact that a clock runs fast depend
    on the theory that predicts that the clock runs fast,
    let's start again and take it step by step.

    Please give yourself a chance and think before you respond.

    I will use the example with a geostationary satellite
    because that's the only way an observer on the ground
    can have a satellite continuously in sight.

    So we have an observer with clock A at equator at longitude 0⁰,
    and a geostationary satellite with clock B at longitude 0⁰.
    This means that the satellite always will be at the zenith
    as viewed from the observer on the ground.

    Clock A is at the geoid and is running synchronously with UTC.
    This means that clock A will measure the duration one rotation of
    the Earth to be:
    τ₀ = 86164.09050000000 s (this is one sidereal day measured with UTC)

    We agree that clock B in the satellite runs at the rate (1+5.3915E-10)
    relative to UTC. Clock B will measure the duration of one orbit to be:
    τ₁ = τ₀⋅(1+5.3915E-10) = 86164.09054645538 s

    If you don't agree in everything so far, we stop here. ======================================================

    If you do agree, we go on:

    The satellite has a radio sender which according to clock B sends
    the frequency f₁ = 10.0000000000000 GHz.
    This means that the number of cycles sent during one orbit is:
    N = τ₁⋅f₁ = (86164.09054645538s)⋅10E9 Hz = 861640905464553.8 cycles

    Now we know that the observer on the ground will receive all the cycles
    that are sent from the satellite. This means that he during one rotation
    will receive the same number of cycles as the satellite sent during one
    orbit.

    If you don't agree in everything so far, we stop here. ======================================================

    If you do agree, we go on:

    The observer will receive N cycles during one rotation of the Earth.
    So the frequency as measured by clock A is:
    f₀ = N/τ₀ = 861640905464553.8/86164.0905 Hz = 10.0000000053915 GHz

    Note that f₀ = (1+5.3915E-10)⋅f₁ = 10.0000000053915 GHz
    This means that the received frequency is higher than the sent frequency
    by the factor (1+5.3915E-10).
    This is called gravitational blue shift.


    If you don't agree in everything so far, we stop here. ======================================================

    If you do agree, we go on:

    A logically inevitable consequence of the fact that
    clock B runs fast by (1+5.3915E-10) relative to UTC is that
    the gravitational blue shift is the same factor (1+5.3915E-10).

    Do you agree?

    --
    Paul

    https://paulba.no/

    --- SoupGate-Win32 v1.05
    * Origin: fsxNet Usenet Gateway (21:1/5)
  • From Paul B. Andersen@21:1/5 to All on Fri Jan 19 20:36:40 2024
    Den 19.01.2024 12:48, skrev Lou:
    On Thursday 18 January 2024 at 13:48:04 UTC, Paul B. Andersen wrote:
    Den 18.01.2024 00:14, skrev Lou:
    On Wednesday 17 January 2024 at 21:15:47 UTC, Paul B. Andersen wrote:
    You don't have to crunch any numbers to UNDERSTAND
    that blue shift is an inevitable consequence of
    that clock B runs faster than clock A relative to UTC.

    In relativity yes. But not in a classical model.
    Gravitational Blueshifting is a relativist assumption, but in a classical >>> model Clock gains are modelled using resonance. No gravitational
    blueshifting is needed. Both sat and ground clocks run at the same time. >>> Just the resonant frequencies of the atoms varies with altitude.

    Since you don't even try to give a rational reason for why
    the consequence of the fact that a clock runs fast depend
    on the theory that predicts that the clock runs fast,
    let's start again and take it step by step.

    Please give yourself a chance and think before you respond.

    I will use the example with a geostationary satellite
    because that's the only way an observer on the ground
    can have a satellite continuously in sight.

    So we have an observer with clock A at equator at longitude 0⁰,
    and a geostationary satellite with clock B at longitude 0⁰.
    This means that the satellite always will be at the zenith
    as viewed from the observer on the ground.

    Clock A is at the geoid and is running synchronously with UTC.
    This means that clock A will measure the duration one rotation of
    the Earth to be:
    τ₀ = 86164.09050000000 s (this is one sidereal day measured with UTC)
    We agree that clock B in the satellite runs at the rate (1+5.3915E-10)
    relative to UTC. Clock B will measure the duration of one orbit to be:
    τ₁ = τ₀⋅(1+5.3915E-10) = 86164.09054645538 s
    If you don't agree in everything so far, we stop here.
    ======================================================

    I don’t agree at all. In a classical model a second up there is the same time
    as a second down here. The only thing that happens is up there every second the caesium atoms beat slightly faster due to atomic resonance responding
    to lesser G force. (GM/r) As I’ve repeatedly said. But the divisor in the sat
    compensate by counting 9192631774.1 beats a second.
    Your calculation above is incorrect. You have time passing at a different rate up there. That’s relativity. Not classical theory.
    Correct your above calculation so both sat and ground clock are both counting the
    same amount of seconds every second. And then maybe I’ll talk.


    OK. So we stop here and take a closer look at what is measured
    in the satellite.

    Both clock A and B are clocks which ticks out seconds as defined by SI.
    It doesn't matter if the atomic clocks are based on Cs, Rb or H,
    they all ticks out seconds as defined by SI. And so does your
    wristwatch and all clocks which use second as the time unit.
    There is nothing special about atomic clocks other than their
    extremely high precision.

    So let's call a normal clock an SI-clock to point out that
    its rate is not adjusted like the clocks in GNSS satellites.

    We know that normal SI-clocks on the geoid (ground) runs
    synchronous with UTC by definition.

    We know that a normal SI-clock in a geostationary satellite
    runs at the rate (1+5.3915E-10) relative to UTC.
    You say:
    "The only thing that happens is up there every second
    the caesium atoms beat slightly faster due to atomic
    resonance responding to lesser G force."

    So we agree that the clock "up there" runs faster than UTC
    by the factor (1+5.3915E-10). WHY the clock runs fast is irrelevant.

    The length of a sidereal day can be measured by observing
    a specific star close to the equatorial plane through
    a fixed vertical telescope. You simply measure the time
    between each time the star is at the same position in
    the telescope.

    This is done every day for centuries at Greenwich observatory,
    so we know that a sidereal day measured with a UTC clock is
    86164.0905 seconds.

    So clock A will measure τ₀ = 86164.0905

    The geostationary satellite is rotating once per orbit
    so it always face the same side to the Earth. (This
    is normal because it will have directive antennae which
    always shall point at the same place on Earth. TV?)
    So a telescope (operated by a human or a computer) pointing
    away from the Earth can observe a star, and with clock B
    measure the time between each time the star is at the same
    position in the telescope.
    And since we know that clock B runs slow this must be measured:
    τ₁ = τ₀⋅(1+5.3915E-10) = 86164.09054645538 s

    Do you still not agree?
    If not, specify exactly what is wrong above


    --
    Paul

    https://paulba.no/

    --- SoupGate-Win32 v1.05
    * Origin: fsxNet Usenet Gateway (21:1/5)
  • From Paul B. Andersen@21:1/5 to All on Fri Jan 19 20:44:34 2024
    Den 19.01.2024 20:36, skrev Paul B. Andersen:

    The geostationary satellite is rotating once per orbit
    so it always face the same side to the Earth. (This
    is normal because it will have directive antennae which
    always shall point at the same place on Earth. TV?)
    So a telescope (operated by a human or a computer) pointing
    away from the Earth can observe a star, and with clock B
    measure the time between each time the star is at the same
    position in the telescope.
    And since we know that clock B runs slow this must be measured:

    Typo:
    And since we know that clock B runs fast this must be measured:

     τ₁ = τ₀⋅(1+5.3915E-10) = 86164.09054645538 s

    Do you still not agree?
    If not, specify exactly what is wrong above



    --
    Paul

    https://paulba.no/

    --- SoupGate-Win32 v1.05
    * Origin: fsxNet Usenet Gateway (21:1/5)
  • From Paul B. Andersen@21:1/5 to All on Sat Jan 20 13:49:46 2024
    Den 20.01.2024 11:17, skrev Lou:
    On Friday 19 January 2024 at 19:32:57 UTC, Paul B. Andersen wrote:
    Den 19.01.2024 12:48, skrev Lou:
    On Thursday 18 January 2024 at 13:48:04 UTC, Paul B. Andersen wrote:
    I will use the example with a geostationary satellite
    because that's the only way an observer on the ground
    can have a satellite continuously in sight.

    So we have an observer with clock A at equator at longitude 0⁰,
    and a geostationary satellite with clock B at longitude 0⁰.
    This means that the satellite always will be at the zenith
    as viewed from the observer on the ground.

    Clock A is at the geoid and is running synchronously with UTC.
    This means that clock A will measure the duration one rotation of
    the Earth to be:
    τ₀ = 86164.09050000000 s (this is one sidereal day measured with UTC) >>>> We agree that clock B in the satellite runs at the rate (1+5.3915E-10) >>>> relative to UTC. Clock B will measure the duration of one orbit to be: >>>> τ₁ = τ₀⋅(1+5.3915E-10) = 86164.09054645538 s
    If you don't agree in everything so far, we stop here.
    ======================================================


    I don’t agree at all. In a classical model a second up there is the same time
    as a second down here. The only thing that happens is up there every second >>> the caesium atoms beat slightly faster due to atomic resonance responding >>> to lesser G force. (GM/r) As I’ve repeatedly said. But the divisor in the sat
    compensate by counting 9192631774.1 beats a second.
    Your calculation above is incorrect. You have time passing at a different >>> rate up there. That’s relativity. Not classical theory.
    Correct your above calculation so both sat and ground clock are both counting the
    same amount of seconds every second. And then maybe I’ll talk.


    OK. So we stop here and take a closer look at what is measured
    in the satellite.

    Both clock A and B are clocks which ticks out seconds as defined by SI.
    It doesn't matter if the atomic clocks are based on Cs, Rb or H,
    they all ticks out seconds as defined by SI. And so does your
    wristwatch and all clocks which use second as the time unit.
    There is nothing special about atomic clocks other than their
    extremely high precision.

    The above are simple facts.
    We are talking about real clocks in the real world.
    So exactly what is your problem with that?


    Your logic is impeccably screwed as usual. If you agree that both A and
    B clocks must have the same length of seconds in a *classical model* ... regardless of how many times their atoms beat per second. And regardless
    of their altitude in a classical non relativistic model.
    Then you can’t then contradict yourself and say that time passes faster
    up there in orbit in a classical model. Or that a sat clock B has a different length of a second than a ground clocks second.
    You forgot that in a classical model time and the passage of time is the
    same EVERYWHERE in the universe.
    You deliberately ignored the fact that under a classical model clock B is just ticking at a faster rate per second up there because under a classical model resonant frequencies of atoms (and clock atoms) speed up under
    a lower force of gravity as per GM/r.

    We are not "in a classical model", or in "a relativistic model."
    We are in the real world!

    In the real geostationary satellite in the real world
    there is a real, normal atomic clock ticking out seconds
    as defined by SI. The only way to measure time in the real
    geostationary satellite is to use this real clock.
    And when this satellite sends a signal with frequency f₁= 10GHz
    then there are 10E9 cycles per second measured by the real normal clock.

    Since we agree that this real clock "up there" will run
    fast by the factor (1+5.3915E-10) relative to UTC,
    you must also agree to the following:

    Clock B in the satellite runs at the rate (1+5.3915E-10) relative
    to UTC. Clock B will measure the duration of one orbit to be:
    τ₁ = τ₀⋅(1+5.3915E-10) = 86164.09054645538 s
    The number of cycles sent per orbit is:
    N = τ₁⋅f₁ = 861640905464553.8 cycles

    Clock A is at the geoid and is running synchronously with UTC.
    This means that clock A will measure the duration one rotation of
    the Earth to be:
    τ₀ = 86164.09050000000 s (this is one sidereal day measured with UTC)
    And since A will receive all the N cycles emitted by B during one
    rotation of the Earth, he will measure the received frequency to be:
    f₀ = N/τ₀ = 10.0000000053915 GHz

    f₀ = (1+5.3915E-10)⋅f₁
    This is called gravitational blue shift which is an inevitable
    consequence of the fact that clocks "up there" run fast relative to UTC.
    AND it is observed in the real world.

    --------------------

    Or have you changed your mind?
    Doesn't your "classical model" predict that clock B will run fast
    relative to UTC?


    --
    Paul

    https://paulba.no/

    --- SoupGate-Win32 v1.05
    * Origin: fsxNet Usenet Gateway (21:1/5)
  • From Volney@21:1/5 to Lou on Sat Jan 20 13:45:39 2024
    On 1/20/2024 5:17 AM, Lou wrote:

    If you agree that both A and
    B clocks must have the same length of seconds in a *classical model* ... regardless of how many times their atoms beat per second.

    That sentence is self-contradictory. Atomic clocks are designed to have
    a certain constant number of transitions per second. It is impossible
    for a properly operating atomic clock to have their atoms 'beat' a
    different number of times per second. Particularly for Cs clocks, since
    the second is DEFINED as the time taken for 9192631770 Cs atom
    transitions. DEFINED. Not 5, not 3333333333, not 9192631774.1, but
    9192631770 transitions is exactly 1 second.

    --- SoupGate-Win32 v1.05
    * Origin: fsxNet Usenet Gateway (21:1/5)
  • From Volney@21:1/5 to Lou on Sun Jan 21 01:19:49 2024
    On 1/20/2024 3:41 PM, Lou wrote:
    On Saturday 20 January 2024 at 18:45:43 UTC, Volney wrote:
    On 1/20/2024 5:17 AM, Lou wrote:
    Paul wrote
    Both clock A and B are clocks which ticks out seconds as defined by SI. >>>> *It doesn't matter if the atomic clocks are based on Cs, Rb or H,
    they all ticks out seconds as defined by SI.* And so does your
    wristwatch and all clocks which use second as the time unit.
    There is nothing special about atomic clocks other than their
    extremely high precision.
    Your logic is impeccably screwed as usual. If you agree that both A and
    B clocks must have the same length of seconds in a *classical model* ... >>> regardless of how many times their atoms beat per second. And regardless >>> of their altitude in a classical non relativistic model.
    That sentence is self-contradictory. Atomic clocks are designed to have
    a certain constant number of transitions per second. It is impossible
    for a properly operating atomic clock to have their atoms 'beat' a
    different number of times per second.

    Notice Paul was admitting above that any clock with any different frequencies can still beat out the same length of a second.

    Yes and no. The second is defined as 9192631770 Cs cycle times. All
    other clocks need to be calibrated against the defined second, meaning something or other needs to be measured. For example a Rb clock needs
    the Rb transition time to be measured. I know scientists have measured
    that to many sig figs so a Rb clock is nearly as good as a Cs clock.

    After
    having said the opposite earlier in that he had tried to say a second
    cannot be comprised of any amount of beats. Just 9192631770.! :-D

    By definition, 9192631770 Cs cycles is the defined second.

    So by the same token there should be no problem with a caesium clock
    beating out 9192631770 beats per second on earth. And 9192631774.1
    up there in space.

    Except 9192631774.1 cycles isn't a second. It's a tiny bit longer.

    And this is confirmed by observations.

    The blueshift of the signal sent to earth was calculated using GR, and 9192631774.1 was calculated as the amount needed to counteract the
    blueshift. That's why the satellite '10.23 MHz' carrier, which is
    generated by using the divisor 9192631774.1 in the Cs clock (instead of 9192631770) actually transmits at 10.2299999954326 MHz.

    GPS sat clocks do
    indeed best faster per second up there.

    Nope. A second is a second is a second, and 1 second aboard the
    satellite is still 9192631770 Cs cycles. Despite the fact a signal is
    generated and used using a cycle time of 9192631774.1 Cs cycles.

    You admitted it yourself
    when you admitted the divisor has to be set to count 9292631774.1
    beats per second to match the second on earth.

    I admitted no such thing, the "beats per second" is added by YOU.

    Particularly for Cs clocks, since
    the second is DEFINED as the time taken for 9192631770 Cs atom
    transitions. DEFINED. Not 5, not 3333333333, not 9192631774.1, but
    9192631770 transitions is exactly 1 second.

    You forgot. The second is defined as a 9192631770 beats a second
    *on the geoid*.

    NO IT IS NOT. Quit inventing garbage and pretending that it's the truth!
    A second is defined as 9192631770 Cs cycles on a (local) Cs clock.

    But at higher altitudes the ceasium clock beats faster.

    No, it does not. A second is still 9192631770 Cs cycles local to the
    satellite clock. Remember, the earth's surface is NOT local to the
    satellite's clock, so a second on earth will not be 9192631770 cycles of
    the satellite's clock, but it will be 9192631770 cycles of a local Cs
    clock. That's because of the blueshift of the signal.

    That’s why the second is defined as 9192631770 at the geoid and
    not at the top of Everest or any other altitude.

    It is NOT defined 'at the geoid'. Quit making up garbage. The geoid and altitude are NOT mentioned in the definition of the second. They did not
    want the second to depend on anything other than the nature of Cs atoms,
    and that includes the geoid, not available in space for example.

    If you look up atomic clocks
    etc you will see that the most accurate “second” has to be derived by comparing atomic clocks at different potentials to determine what the official geoid second is.

    Nope. They average multiple Cs clocks to avoid failure of one "master"
    clock. They've since added compensation for the various different
    altitudes of individual clocks so they have a super-accurate reference
    for TAI or UTC times for example, which DO depend on being defined at
    the geoid.

    Why? Because the same atomic clock will have
    a different frequency at different altitudes.

    Nope. An observer at the geoid will measure an atomic clock in Quito as
    running a touch fast (while the Quito observer will measure a London
    clock as running a touch slow) only because of the altitude difference
    between Quito and the geoid/London. If the Quito clock participates in
    the multi-clock timebase, its signal will be compensated for the
    altitude difference, to generate a geoid-accurate TAI or UTC time. (and
    yes, a Quito observer may complain that a TAI clock runs too slow)

    And thus give a different second
    length at different altitudes.

    Nope. A second is DEFINED as 9192631770 Cs transition times of a Local
    Cs clock. Regardless of altitude. Altitude is not part of the definition
    of the second.

    And it is therefore the frequency of the atomic clock *at the geoid* that defines the internationally accepted standard for a second.

    Again, no. The Cs cycle time on a LOCAL clock.

    Not the frequency of the same atomic clock at the top of Everest.

    The frequency at the top of Everest is correct -- for an observer on
    Everest. Introduce a blueshift for the signal from Everest to sea level
    and a difference will be seen.

    So yes ......the observations confirm that caesium atoms DO beat
    at faster rates at higher altitudes as predicted by a non relativistic classical
    model. Contrary to your above false claim that they dont!!!

    Nope. A second is still 9192631770 Cs transition times, regardless of
    altitude. Remember, the whole idea between the "new" (1967) second
    definition was to reduce dependence on physical objects (like the earth)
    as much as possible. As it is, anyone with some cesium and the
    technology to make it into an atomic clock will get an exact second.
    Regardless of altitude or even if 1000 light years away from earth.

    You simply don't understand the concept of signals being redshifted or blueshifted when going between different gravitational potentials or speeds.

    --- SoupGate-Win32 v1.05
    * Origin: fsxNet Usenet Gateway (21:1/5)
  • From Paul B. Andersen@21:1/5 to All on Sun Jan 21 11:17:31 2024
    Den 20.01.2024 15:09, skrev Lou:
    On Saturday 20 January 2024 at 12:46:03 UTC, Paul B. Andersen wrote:
    We are not "in a classical model", or in "a relativistic model."
    We are in the real world!


    Great! And in this real world 1 second down here is equivelent to one second up there. Unless you are a delusional relativist.

    Right!

    Here is the definition of a second we use in the real world:
    "The second is defined by taking the fixed numerical value
    of the cesium frequency ∆ν_Cs, the unperturbed ground-state
    hyperfine transition frequency of the cesium-133 atom, to
    be 9,192,631,770 when expressed in the unit Hz, which is
    equal to s⁻¹."

    All normal clocks which use second as the time unit
    are build according to this definition.
    The difference between your wristwatch and an atomic
    clock is only the precision of the clock.

    So yes, all normal clocks run at their normal rate
    and ticks out a second per second everywhere and always.

    This is the DEFINITION of "time" as used by engineers
    and physicist in the real world.


    In the real geostationary satellite in the real world
    there is a real, normal atomic clock ticking out seconds
    as defined by SI. The only way to measure time in the real
    geostationary satellite is to use this real clock.
    And when this satellite sends a signal with frequency f₁= 10GHz
    then there are 10E9 cycles per second measured by the real normal clock.

    Since we agree that this real clock "up there" will run
    fast by the factor (1+5.3915E-10) relative to UTC,
    you must also agree to the following:


    This real clock up there isn’t running fast relative to UTC.

    So you have changed your mind? :-D

    Clock B in the satellite runs at the rate (1+5.3915E-10) relative to UTC


    Wrong. You forgot. The GPS clock has a divisor. And it measures
    One second to be 9192631774.1 beats a second.

    No, I haven't forgot that the clock in a GPS SV
    is not a normal clock.
    It ticks out only 0.99999999946085 seconds per seconds.
    It runs too slow to be a normal clock.

    We are talking about a NORMAL clock in geostationary orbit!


    The rate relative to UTC of a NORMAL clock in circular orbit is: ----------------------------------------------------------------
    Δf/f = dτ/dt_utc - 1 = - 1.5⋅GM/r⋅c² + δutc

    where:
    GM = 3.986004418E14 m³/s²
    c = 299792458 m/s
    δutc = 6.96927E-10
    (1 + δutc) is the rate of UTC relative to Schwarzschild coordinate time

    Normal clock in GPS orbit:
    ---------------------------
    p = 43082.04525 s (orbital period half a sidereal day)
    r = cbrt(GM⋅p²/4π²) = 26561763 m.

    A normal clock in GPS orbit runs fast by
    Δf/f = 4.4647E-10
    relative to UTC.

    Normal clock in Galileo orbit:
    ------------------------------
    r = 29600000 m

    A normal clock in Galileo orbit runs fast by
    Δf/f = 4.7218E-10
    relative to UTC.

    Normal clock in GLONASS orbit:
    -------------------------------
    Equatorial radius of the Earth R = 6378137 m
    Altitude of orbit h = 19100000 m
    r = h + R = 25478137 m

    A normal clock in GLONASS orbit runs fast by
    Δf/f = 4.3582E-10
    relative to UTC.

    Normal clock in geostationary orbit:
    ------------------------------------
    p = 86164.0905 s (orbital period one sidereal day)
    r = cbrt(GM⋅p²/4π²) = 42164169.6241 m

    A normal clock in geostationary orbit runs fast by
    Δf/f = 5.391498E-10
    relative to UTC.


    Note that these predictions by GR are thoroughly confirmed
    for GPS, Galileo and GLONASS, so there is no reason to doubt
    that it also will be correct for a clock in geostationary
    orbit.
    _________________________________________________________________


    So clock B in the geostationary orbit runs at the rate
    (1+5.3915E-10) relative to UTC.

    Previously you claimed that "the classical model" predicted
    the same as GR for the rate of clocks "up there".

    Now you seem to have changed your mind.

    So please tell me:
    What is the rate relative to UTC of a NORMAL clock
    in geostationary orbit?

    Is it (1+5.3915E-10) relative to UTC as we know is correct,
    or is it something else?


    Hint: Look up UTC.

    --
    Paul

    https://paulba.no/

    --- SoupGate-Win32 v1.05
    * Origin: fsxNet Usenet Gateway (21:1/5)
  • From Paul B. Andersen@21:1/5 to All on Sun Jan 21 12:06:02 2024
    Den 20.01.2024 15:09, skrev Lou:

    The GPS clock has a divisor. And it measures
    One second to be 9192631774.1 beats a second.
    At which point the sat sends this data back to earth ground.
    And guess what..,,! It matches ground seconds. It ISNT runnimg
    faster.
    Right!

    To stay in sync with UTC the clock in a GPS SV is adjusted
    to run slower than a normal SI-clock by the factor (1-4.4647E-10).

    It is thoroughly confirmed that it then IS in sync with UTC,
    or the GPS wouldn't work.

    Which is a beautiful confirmation of the fact that a normal SI-clock
    in GPS orbit runs fast relative to UTC by the factor (1+4.4647E-10).

    Thanks for reminding me of this confirmation of the GR-prediction.

    --
    Paul

    https://paulba.no/

    --- SoupGate-Win32 v1.05
    * Origin: fsxNet Usenet Gateway (21:1/5)
  • From Tom Roberts@21:1/5 to Prokaryotic Capase Homolog on Sun Jan 21 09:17:38 2024
    Everybody in this thread is over-thinking this and getting bogged down
    in irrelevancies. In particular, the meaning of "time" is not important
    -- everything can be, and should be, expressed in terms of clock
    readings, because they are what is measured.

    On 1/20/24 11:50 PM, Prokaryotic Capase Homolog wrote:
    The second does not change with altitude of a clock above or below
    the geoid.

    Yes.

    The words "one second" are defined to mean "the duration of 9192631770
    cycles of the unperturbed hyperfine ground-state transition of the
    Cs-133 atom". This definition applies anywhere and anywhen, but must
    only be used locally (i.e. the Cs-133 atom must be co-located and
    co-moving with whatever is being timed).

    Our *observations* of a clock's readings, however, *does* depend on
    the gravitational potential difference between the clock's position
    and our own.

    Note we simply cannot observe a clock that is not co-located with us.
    What we observe are SIGNALS from the clock, usually EM signals (light,
    radio). It should be obvious that one must account for how those signals
    are measured, and how that varies with distance, location, and relative
    motion, as the signals are NOT the distant clock.

    It should also be obvious that a clock can only measure time intervals
    between events on its worldline. So to model how a clock measures the
    intervals between signals from a distant clock, those signals must be geometrically projected onto the measuring clock's worldline.

    [This geometrical projection is in spacetime -- one cannot
    avoid that when discussing clocks located at different
    positions in space, or moving relative to each other.]

    For the case of a standard clock on a GPS satellite emitting radio
    signals at 10.22999999543 MHz, those signals are measured on earth's
    geoid to arrive at 10.23000000000 MHz (by a standard clock located
    there). General Relativity models such measurements very accurately, as
    due solely to that geometrical projection.

    [In GR, clocks, signals, and all other physical processes
    are not affected by gravity (spacetime curvature). But
    geometry is affected, as are geometrical projections.]

    Tom Roberts

    --- SoupGate-Win32 v1.05
    * Origin: fsxNet Usenet Gateway (21:1/5)
  • From Volney@21:1/5 to Lou on Sun Jan 21 14:40:58 2024
    On 1/21/2024 6:27 AM, Lou wrote:

    Typical Blarney from Volney. You spend the entire post admitting that the
    GPS clock divisor is set to measure more beats per second (9192631774.1
    as opposed to 9192631770 on ground) when up in orbit to make its second length matches the ground second.

    No, I did not. There is only one second, 9192631770 periods of the Cs
    frequency on a local Cs clock.

    And that Cs clocks run at different speeds
    at different altitudes due to gravitational effects.

    Again, no I did not. All Cs clocks tick at 9192631770 periods per
    second. By definition.

    But then in complete contradiction you then say that all these Cs clocks aren’t beating at different frequencies or running at different speeds at different altitudes.

    They are not. They tick at 9192631770 periods per second.

    And that they only magically appear to be running at
    different speeds.

    Blueshift of signals (not the clocks themselves) are physics, not magic.

    I think you will find it hard to disprove my claim that the GPS atomic clock resonant frequency is beating slightly faster up there at around 9192631774.1 beats per second due to the effects of gravity.

    No, because BY DEFINITION they tick at 9192631770 periods per second.

    And needs to be corrected by an
    onboard divisor to make sure it’s second matches the earth clocks second.

    No, the correction is to compensate for the blueshift of the signal to
    earth so that the 10.23 MHz really is received at 10.23 MHz. That can
    only happen if the signal is transmitted at 10.2299999954326 MHz at the altitude of the GPS satellites.

    --- SoupGate-Win32 v1.05
    * Origin: fsxNet Usenet Gateway (21:1/5)
  • From Paul B. Andersen@21:1/5 to All on Mon Jan 22 12:15:29 2024
    Den 21.01.2024 12:41, skrev Lou:
    On Sunday 21 January 2024 at 10:13:46 UTC, Paul B. Andersen wrote:
    Den 20.01.2024 15:09, skrev Lou:

    Great! And in this real world 1 second down here is equivelent to one second
    up there. Unless you are a delusional relativist.

    When a normal clock say a second has passed, then
    the time one second has passed irrespective of the clock's
    position and state of motion.

    Read the following again:

    Here is the definition of a second we use in the real world:
    "The second is defined by taking the fixed numerical value
    of the cesium frequency ∆ν_Cs, the unperturbed ground-state
    hyperfine transition frequency of the cesium-133 atom, to
    be 9,192,631,770 when expressed in the unit Hz, which is
    equal to s⁻¹."

    All normal clocks which use second as the time unit
    are build according to this definition.
    The difference between your wristwatch and an atomic
    clock is only the precision of the clock.

    So yes, all normal clocks run at their normal rate
    and ticks out a second per second everywhere and always.

    This is the DEFINITION of "time" as used by engineers
    and physicist in the real world.

    < snip irrelevant talk about adjusted GPS clocks>

    Because:

    We are talking about a NORMAL clock in geostationary orbit!


    The rate relative to UTC of a NORMAL clock in circular orbit is:
    ----------------------------------------------------------------
    Δf/f = dτ/dt_utc - 1 = - 1.5⋅GM/r⋅c² + δutc

    where:
    GM = 3.986004418E14 m³/s²
    c = 299792458 m/s
    δutc = 6.96927E-10
    (1 + δutc) is the rate of UTC relative to Schwarzschild coordinate time

    Normal clock in geostationary orbit:
    ------------------------------------
    p = 86164.0905 s (orbital period one sidereal day)
    r = cbrt(GM⋅p²/4π²) = 42164169.6241 m

    A normal clock in geostationary orbit runs fast by
    Δf/f = 5.391498E-10
    relative to UTC.


    You snipped my question:

    Previously you claimed that "the classical model" predicted
    the same as GR for the rate of clocks "up there".

    Now you seem to have changed your mind.

    So please tell me:
    What is the rate relative to UTC of a NORMAL clock
    in geostationary orbit?

    Is it (1+5.3915E-10) relative to UTC as we know is correct,
    or is it something else?



    Note that the predictions by a non relativistic classical model that
    resonant frequencies of atomic clocks will be faster at higher altitudes
    are also thoroughly confirmed by GPS,Galileo and GLONASS.

    I take this to mean that your answer to my question above is:

    The "non relativistic classical model" predicts that the rate relative
    to UTC of a normal atomic clock at higher altitudes will be the same
    as the thoroughly confirmed prediction of GR.

    We can then continue where we previously stopped :

    Since you agree that the clock in geostationary orbit will
    run fast by the factor (1+5.3915E-10) relative to UTC,
    you must also agree to the following:

    Clock A is at the geoid and is running synchronously with UTC.
    This means that clock A will measure the duration of one rotation
    of the Earth to be:
    τ₀ = 86164.09050000000 s (this is one sidereal day measured with UTC)

    Clock B in the satellite runs at the rate (1+5.3915E-10) relative
    to UTC. Clock B will measure the duration of one orbit to be:
    τ₁ = τ₀⋅(1+5.3915E-10) = 86164.09054645538 s

    The geostationary satellite sends a signal with frequency
    f₁ = 10 GHz as measured by clock B.
    The number of cycles sent during one orbit is:
    N = τ₁⋅f₁ = 861640905464553.8 cycles

    Since A during one rotation of the Earth will receive all the N
    cycles emitted by B during one orbit, he will measure the received
    frequency to be:
    f₀ = N/τ₀ = 10.0000000053915 GHz

    f₀ = (1+5.3915E-10)⋅f₁
    This is called gravitational blue shift which is an inevitable
    consequence of the fact that clocks "up there" run fast relative to UTC.
    AND it is observed in the real world.

    --------------------

    Do you now agree to the above?
    Or have you changed your mind again?
    Doesn't your "classical model" predict that clock B will run fast
    relative to UTC?


    --
    Paul

    https://paulba.no/

    --- SoupGate-Win32 v1.05
    * Origin: fsxNet Usenet Gateway (21:1/5)
  • From Paul B. Andersen@21:1/5 to All on Mon Jan 22 12:48:11 2024
    Den 21.01.2024 12:46, skrev Lou:
    On Sunday 21 January 2024 at 11:02:17 UTC, Paul B. Andersen wrote:
    Den 20.01.2024 15:09, skrev Lou:

    The GPS clock has a divisor. And it measures
    One second to be 9192631774.1 beats a second.
    At which point the sat sends this data back to earth ground.
    And guess what..,,! It matches ground seconds. It ISNT runnimg
    faster.
    Right!

    To stay in sync with UTC the clock in a GPS SV is adjusted
    to run slower than a normal SI-clock by the factor (1-4.4647E-10).

    It is thoroughly confirmed that it then IS in sync with UTC,
    or the GPS wouldn't work.

    Which is a beautiful confirmation of the fact that a normal SI-clock
    in GPS orbit runs fast relative to UTC by the factor (1+4.4647E-10).

    Thanks for reminding me of this confirmation of the GR-prediction.


    And thanks for agreeing with me that this is also a confirmation of
    the non relativistic classical model prediction. Seeing as you just admitted that
    the GPS orbit clocks atoms frequency is faster per second then when on the ground
    and needs to be corrected by an onboard divisor.


    It is interesting to see that you now claim that your
    "non relativistic classical model" predicts the same as
    "the relativistic model GR".

    If your "non relativistic classical model" predicts exactly
    the same as GR for an experiment, then a confirmation of GR will
    also be a confirmation of your "non relativistic classical model".
    Obviously!

    I will not ask you to show how you from your "non relativistic
    classical model" can calculate that a clock in a geostationary
    satellite will run at the rate (1+5.3915E-10) relative to UTC.
    We both know that you can't do that.

    Or maybe you claim that the GR derivation from the Schwarzschild
    metric really is your "non relativistic classical model" because
    it contains the Newtonian gravitational potential GM/r? :-D

    --
    Paul

    https://paulba.no/

    --- SoupGate-Win32 v1.05
    * Origin: fsxNet Usenet Gateway (21:1/5)
  • From Volney@21:1/5 to Laurence Clark Crossen on Mon Jan 22 13:23:13 2024
    On 1/21/2024 11:18 PM, Laurence Clark Crossen wrote:
    On Sunday, January 21, 2024 at 3:02:17 AM UTC-8, Paul B. Andersen wrote:
    Den 20.01.2024 15:09, skrev Lou:

    The GPS clock has a divisor. And it measures
    One second to be 9192631774.1 beats a second.
    At which point the sat sends this data back to earth ground.
    And guess what..,,! It matches ground seconds. It ISNT runnimg
    faster.
    Right!

    To stay in sync with UTC the clock in a GPS SV is adjusted
    to run slower than a normal SI-clock by the factor (1-4.4647E-10).

    It is thoroughly confirmed that it then IS in sync with UTC,
    or the GPS wouldn't work.

    Which is a beautiful confirmation of the fact that a normal SI-clock
    in GPS orbit runs fast relative to UTC by the factor (1+4.4647E-10).

    Thanks for reminding me of this confirmation of the GR-prediction.

    --
    Paul

    https://paulba.no/
    There is a lot of talking past each other, e.g.:

    "No, I haven't forgot that the clock in a GPS SV
    is not a normal clock.
    It ticks out only 0.99999999946085 seconds per seconds.
    It runs too slow to be a normal clock."

    No, it is set at a lower frequency to run exactly the same in orbit as a clock on Earth.

    Well, yeah, that's the gravitational blueshift of the transmitted
    signal. A blueshift increases the received frequency. They design the
    system so that the lower frequencies used on the satellite are EXACTLY compensated by the blueshift.

    Skeptics know that but you think you have to explain it:

    You seem unable to understand it, so we try to explain it.

    "To stay in sync with UTC the clock in a GPS SV is adjusted
    to run slower than a normal SI-clock by the factor (1-4.4647E-10).

    It is thoroughly confirmed that it then IS in sync with UTC,
    or the GPS wouldn't work."

    That's been known since the NTS-2 prototype was launched. They ran for
    20 days with no compensation (Newton mode, I call it) and it didn't work correctly. The received frequencies were too high, for one. Then they
    ran it in "Einstein mode" (clock frequency reduced to what GR predicts, 1-4.4647E-10) and it worked fine.

    So, why adjust again for the gravitational blue shift? Or adjust for the radio signal as Roberts asserts below?

    What do you mean "again"? It is adjusted once and only once. If you ask
    that question, you don't understand what's going on.

    --- SoupGate-Win32 v1.05
    * Origin: fsxNet Usenet Gateway (21:1/5)
  • From Tom Roberts@21:1/5 to Laurence Clark Crossen on Mon Jan 22 22:14:24 2024
    On 1/22/24 9:39 PM, Laurence Clark Crossen wrote:
    How would the clock in orbit be left to run fast without calibrating
    it?

    The Cs-133 oscillators run at 9,192,631,770 Hz, and NOTHING can change
    that, because it is a DEFINITION. Such Cs-133 clocks are
    self-calibrating (because of that definition).

    So the designers and implementers of GPS satellites modified the "clock"
    to cancel the gravitational blueshift of signals to the ground. This is
    not a standard clock.

    Surely, it must be calibrated to run the same as on Earth.

    The Cs-133 oscillators run at 9,192,631,770 Hz. The timekeeping
    mechanism on the satellite has been modified so SIGNALS TO THE GROUND
    arrive at the correct frequency. In doing so they also canceled the
    difference in elapsed proper time of the satellite clock compared to
    clocks on the ground.

    [Here ground means geoid.]

    That modification is 38 microseconds per day. The GPS system also
    corrects the clock parameters daily, which includes the overall offset,
    but these are more than a thousand times smaller than the modification;
    they account for deviations from the ideal orbit, effects of planets and
    the sun, clock drift, etc.

    If the clock is left to run fast in orbit, that is not a
    gravitational blue shift.

    Hmmmm. The "clock" in the GPS satellites is NOT "left to run fast", it
    has been MODIFIED. The SIGNALS TO THE GROUND experience gravitational blueshift, not the "clock".

    Gravitational blue shift is caused by gravitational attraction and
    not reduced gravity.

    This is just plain not true. Gravitational blueshift is generated by differences in gravitational potential [#], not "gravitational
    attraction" as you falsely claim.

    [#] In weak fields, with speeds << c, such as in the GPS.

    You have been told this MANY times in MANY different ways. The fact that
    you are unable to understand this indicates that physics is just not for
    you -- find some other hobby for which you are more suited.

    Tom Roberts

    --- SoupGate-Win32 v1.05
    * Origin: fsxNet Usenet Gateway (21:1/5)
  • From Paul B. Andersen@21:1/5 to All on Tue Jan 23 10:53:59 2024
    Den 23.01.2024 04:39, skrev Laurence Clark Crossen:
    On Monday, January 22, 2024 at 3:11:42 AM UTC-8, Paul B. Andersen wrote:

    The clock B in geostationary orbit will run fast
    by the factor (1+5.3915E-10) relative to UTC.
    The following is an inevitable logical consequence:

    Read it!

    Clock A is at the geoid and is running synchronously with UTC.
    This means that clock A will measure the duration of one rotation
    of the Earth to be:
    τ₀ = 86164.09050000000 s (this is one sidereal day measured with UTC)
    Clock B in the satellite runs at the rate (1+5.3915E-10) relative
    to UTC. Clock B will measure the duration of one orbit to be:
    τ₁ = τ₀⋅(1+5.3915E-10) = 86164.09054645538 s

    The geostationary satellite sends a signal with frequency
    f₁ = 10 GHz as measured by clock B.
    The number of cycles sent during one orbit is:
    N = τ₁⋅f₁ = 861640905464553.8 cycles

    Since A during one rotation of the Earth will receive all the N
    cycles emitted by B during one orbit, he will measure the received
    frequency to be:
    f₀ = N/τ₀ = 10.0000000053915 GHz

    f₀ = (1+5.3915E-10)⋅f₁
    This is called gravitational blue shift which is an inevitable
    consequence of the fact that clocks in high orbit run fast relative to UTC. >> AND it is observed in the real world.

    (Clocks in low orbit, r < ca. 1.5 Earth radius,
    will run slow relative to UTC)

    How would the clock in orbit be left to run fast without calibrating it?

    By leaving the normal clock to run at its normal rate.

    Surely, it must be calibrated to run the same as on Earth.

    Yes.
    To run synchronously with UTC the GNSS clocks must be corrected
    by the factors:
    GPS: (1 - 4.4647E-10)
    Galileo: (1 - 4.7218E-10)
    GLONASS: (1 - 4.3582E-10)
    This is important because the time reported by the satellites
    must be in sync with UTC within few ns, and a rate error would
    make the offset increase with time.

    But not all clocks in orbit have to run synchronously with UTC.
    You probably know that there are a lot of geostationary satellites
    sending signals (TV etc.) down to Earth. The carriers will typically
    be in the order of 10 GHz, and the bandwidth of the signal will
    be in the order of MHz. So the tiny gravitational blue shift
    Δf = 10E9 x 5.3915E-10 Hz ≈ 5 Hz will be of no consequence whatsoever.
    So there is no reason to adjust the rate of the clock.

    If the clock is left to run fast in orbit, that is not a gravitational blue shift. Gravitational blue shift is caused by gravitational attraction and not reduced gravity. You seem to have conflated the two.

    I explained above what gravitational blue shift is.
    Read it!

    --
    Paul

    https://paulba.no/

    --- SoupGate-Win32 v1.05
    * Origin: fsxNet Usenet Gateway (21:1/5)
  • From Tom Roberts@21:1/5 to Laurence Clark Crossen on Tue Jan 23 22:48:49 2024
    On 1/23/24 10:17 PM, Laurence Clark Crossen wrote:
    On Monday, January 22, 2024 at 8:14:38 PM UTC-8, Tom Roberts wrote:
    On 1/22/24 9:39 PM, Laurence Clark Crossen wrote:
    How would the clock in orbit be left to run fast without calibrating
    it?
    The Cs-133 oscillators run at 9,192,631,770 Hz, and NOTHING can change
    that, because it is a DEFINITION. Such Cs-133 clocks are
    self-calibrating (because of that definition).

    So the designers and implementers of GPS satellites modified the "clock"
    to cancel the gravitational blueshift of signals to the ground. This is
    not a standard clock.
    Surely, it must be calibrated to run the same as on Earth.
    The Cs-133 oscillators run at 9,192,631,770 Hz. The timekeeping
    mechanism on the satellite has been modified so SIGNALS TO THE GROUND
    arrive at the correct frequency. In doing so they also canceled the
    difference in elapsed proper time of the satellite clock compared to
    clocks on the ground.

    [Here ground means geoid.]

    That modification is 38 microseconds per day. The GPS system also
    corrects the clock parameters daily, which includes the overall offset,
    but these are more than a thousand times smaller than the modification;
    they account for deviations from the ideal orbit, effects of planets and
    the sun, clock drift, etc.
    If the clock is left to run fast in orbit, that is not a
    gravitational blue shift.
    Hmmmm. The "clock" in the GPS satellites is NOT "left to run fast", it
    has been MODIFIED. The SIGNALS TO THE GROUND experience gravitational
    blueshift, not the "clock".
    Gravitational blue shift is caused by gravitational attraction and
    not reduced gravity.
    This is just plain not true. Gravitational blueshift is generated by
    differences in gravitational potential [#], not "gravitational
    attraction" as you falsely claim.

    [#] In weak fields, with speeds << c, such as in the GPS.

    You have been told this MANY times in MANY different ways. The fact that
    you are unable to understand this indicates that physics is just not for
    you -- find some other hobby for which you are more suited.

    Tom Roberts
    So the clocks are modified by means of an oscillator that makes them run the same rate in space as on Earth BY DEFINITION.

    No! You need to LEARN HOW TO READ.

    The nonstandard clock in a GPS satellite runs at a slower rate than a
    clock on the geoid. This is done so SIGNALS from the satellite arrive on
    the geoid with the correct frequency.

    Tom Roberts

    --- SoupGate-Win32 v1.05
    * Origin: fsxNet Usenet Gateway (21:1/5)
  • From Tom Roberts@21:1/5 to Maciej Wozniak on Wed Jan 24 09:37:49 2024
    On 1/24/24 12:51 AM, Maciej Wozniak wrote:
    On Wednesday 24 January 2024 at 05:49:02 UTC+1, Tom Roberts wrote:
    The nonstandard clock in a GPS satellite runs at a slower rate
    than a clock on the geoid.

    9 192 631 774 is slower rate than 9 192 631 770, poor halfbrain?

    Yes, of course. The Cs-133 oscillator inherently oscillates at
    9,192,631,770Hz. So since the nonstandard clock in the GPS satellite
    waits until 9,192,631,774 oscillations have passed between its 1-second
    ticks, it does indeed tick more slowly than a standard clock.

    [That number is approximate.]

    Of course, there is nothing exceptional or nonstandard there.

    You really need to LEARN HOW TO READ. 9,192,631,770 oscillations between 1-second ticks is standard, 9,192,631,774 is not.

    They're completely ordunary clocks,

    No, they are not. Look above and READ.

    Tom Roberts

    --- SoupGate-Win32 v1.05
    * Origin: fsxNet Usenet Gateway (21:1/5)